首页 [英语考试]03-11年08由于格式原因除外全国大学生英语竞赛c类初赛试卷及答案完全整理个人收录_保证全!

[英语考试]03-11年08由于格式原因除外全国大学生英语竞赛c类初赛试卷及答案完全整理个人收录_保证全!

举报
开通vip

[英语考试]03-11年08由于格式原因除外全国大学生英语竞赛c类初赛试卷及答案完全整理个人收录_保证全![英语考试]03-11年08由于格式原因除外全国大学生英语竞赛c类初赛试卷及答案完全整理个人收录_保证全! 2003年全国大学生英语竞赛初赛试题 Part I Listening Comprehension(30 minutes,30 points) Section A Dialogues(10 points) Directions:In this section ,you will hear 10 short dialogues(At the end of each dialogue,a question...

[英语考试]03-11年08由于格式原因除外全国大学生英语竞赛c类初赛试卷及答案完全整理个人收录_保证全!
[英语考试]03-11年08由于 格式 pdf格式笔记格式下载页码格式下载公文格式下载简报格式下载 原因除外全国大学生英语竞赛c类初赛试卷及 答案 八年级地理上册填图题岩土工程勘察试题省略号的作用及举例应急救援安全知识车间5s试题及答案 完全整理个人收录_保证全! 2003年全国大学生英语竞赛初赛试题 Part I Listening Comprehension(30 minutes,30 points) Section A Dialogues(10 points) Directions:In this section ,you will hear 10 short dialogues(At the end of each dialogue,a question will be asked about what was said(Both the dialogue and the question will be read only once(After each question there will be a pause(during the pause,you must read the four choices marked A,B,C and D,and decide which is the best answerThen m ark the corresponding letter on the Answer Sheet with a single line ( through the center( 1(A(A sales clerk( B(A police officer( C(A tailor( D(A nurse( 2(A(By train( B(She walks( C(By car( D(By bus( 3(A(Fish is the only dish left( B(Chicken is the only dish left( C(Vegetarian meals are not offered( D(There aren't any vegetarian meals left( 4(A(He starts work next weekend( B(He'll be away( C(He'll be in the mountains( D(He's moving to Florida( 5(A(In an elevator( B(At a dress store( C(On the seventh floor( D(At a department store( 6(A(They felt it was disorganized( B(They were pleased with its Asian content( C(They felt it lacked Asian content( D(They felt it ignored recent events( 7(A(He doesn't have enough time( B(He doesn't have a watch( C(The library doesn't have the articles he wants( D(He can't find the library( 8(A(He wants the woman to dine out with them( B(He wants to work tomorrow( C(He wants the woman to finish dinner first( D(He wants to pay for the dinner( 9(A(Twice a day( B(Twice a week( C(Once a week( D(Daily( 10(A(At two o'clock( B(At four o'clock( C(At three thirty( D(At eight o'clock( Section B News Items(10 points) Directions:In this section,you will hear 10 pieces of short news from BBC or VOA(There will be a question following each piece of news(Write down the answer to each question in no more than 15 words( 11(_______________________________________ 12(_______________________________________ 13(_______________________________________ 14(_______________________________________ 15(_______________________________________ 16(_______________________________________ 17(_______________________________________ 18(_______________________________________ 19(_______________________________________ 20(_______________________________________ Section C Compound Dictation(10 points) Directions:In this section,you will hear a passage three times(When the passage is read for the first time,you should listen carefully for its general idea(Then listen to the passage again(When the passage is read for the second time,you are required to fill in the blanks numbered from 21 to 28 with the exact words you have just heard(For blanks numbered from 29 to 30,you are required to fill in the missing information(You can either use the exact words you have just heard or write down the main points in your own words(Finally,when the passage is read for the third time,you should check what you have written and rewrite the correct answers on the Answer Sheet( Although general Motors and General Electric are large multinational companies with operations around the globe,there are numerous smaller companies that engage in international trade(Because 95percent of the world's population and two-thirds of its (21)_____ power are located outside the United States,it is important for American (22)_____to be present in foreign markets(However, before we explain the different methods by which a company may (23)_____in international trade,we might first consider some important (24)_____that U.S(companies often fail to study before they sell products in a foreign country(These factors are (25)_____with differences in language,in values and attitudes,and in political (26)_____( When (27)_____Coca-Cola into the Chinese market in 1920,the company used a group of Chinese symbols that,when spoken,sounded like Coca-Cola(However,when read, these symbols meant, “a female horse fattened with wax”(Upon reentering the Chinese market in the 1970s,Coca-Cola used a series of Chinese (28)_____that translates into“happiness in the mouth”((29)_________________________( Culture is the total pattern of human behavior that is practiced by a particular group of people((30)_________________________( Part II Vocabulary and Structure(15 minutes,30 points) Section A Multiple Choice(20 points) Directions:Questions 31-50 constitute a complete passage(There are 20blanks in the passage(For each blank there are four choices marked A,B,C and D(Choose the one that best completes the sentence(Then mark the corresponding letter on the Answer Sheet with a single line through the centre( 31(Senior Metropolitan police officers tried to dismiss the Noting Hill race riots which raged for five nights over the August bank holiday in 1958 as the work of“ruffians ,both colored and white” hell-bent on hooliganism ,according to _____ official files( A(recent revealed B(newly released C(previous disclosing D(earlier exposing 32(But police eyewitness reports in the secret papers_____ that they were overwhelmingly the work of a white working class mob out to get the“niggers”( A(contain B(convince C(consist D(confirm 33(The ferocity of the Noting Hill“racial riots”,as the press called them at the time,shocked Britain into_____ for the first time that it was not above the kind of racial conflict then being played out in the American deep south( A(realizing B(witnessing C(watching D(identifying 34(The carnival,which will_____ the streets of west London _____more than 1.5 million people this weekend,was started in 1959 as a direct response to the riots( A(crowd;of B(pour;for C(fill;with D(emerge;in 35(While senior officers tried to play down the racial aspects of the riots,the internal Metropolitan police files released this month at the public record office confirm that the disturbances were overwhelmingly _____ by 300 to 400 strong“Keep Britain White” mobs ,many of them Teddy boys armed with iron bars ,butcher's knives and weighted leather belts,who went“nigger-hunting”among the West Indian residents of Noting Hill and Noting Dale( A(erupted B(commenced C(triggered D(inaugurated 36(The first night left five black men _____ on the pavements of Noting Hill( A(lying unconscious B(there died C(feel faint D(serious hurt 37(The battles raged over the bank holiday weekend as the black _____responded in kind with counterattacks by large groups of“men of color”similarly armed( A(column B(army C(brigade D(community 38(Thomas Williams was stopped by the police as he came out of Bluey's Club on Talbot Road, Noting Hill(He _____a piece of iron down his left trouser leg,a petrol bomb in his right pocket and a razor blade in his inside breast pocket:“I have to protect myself,”he told the arresting officer( A(found to have B(was found to have C(found having D(was found having 39(The _____ files,which were sealed under the 75-year rule but have been released early,show that senior officers tried to convince the then home secretary,“Rab”Butler,that there was not a racial element to the rioting( A(forbidden B(confidential C(incredible D(strict 40(In his official report,Detective Sergeant M.Walters of the Notting Hill police said the national press had been wrong to portray the“widespread series of street disturbances”as“racial”riots: “Whereas there certainly was some _____ feeling between white and colored residents in this area, it is abundantly clear much of the trouble was caused by ruffians,both colored and white,who seized on this opportunity to indulge in hooliganism (” A(ill B(sick C(painful D(hurt 41(But the police witness statements and private statistics _____ ( A(told differently B(interpreted in a different way C(existed m any differences D(told a different story 42(The Met com missioner was told that _____ the 108people who were charged with offences ranging from grievous bodily harm to affray and riot and possessing offensive weapons,72 were white and 36 were “colored”( A(for B(from C(of D(in 43(It is popularly believed that the riot began on the night of Saturday,August 20,when a 400-strong crowd of white men,_____“Teds”,attacked houses occupied by West Indians( A(they are all B(many of them C(some were D(most of them belong to 44(Among the _____ was Majbritt Morrison ,a young white Swedish bride of a Jamaican( A(offenders B(rioters C(victims D(residents 45(She was pelted with stones,glass and wood,and _____ in the back with an iron bar as she tried to get home( A(bruised B(struck C(patted D(scratched 46(The internal police witness statements provide graphic evidence of the motives of the mobs—at one point crowds several thousand strong roamed the streets of Notting Hill,_____ homes and attacking any West Indian they could find( A(plunging into B(breaking into C(seeking for D(searching for 47(PC Richard Bedford said he had seen a mob of 300 to 400 white people in Bramley Road _____:“We will kill all black bastards(Why don't you send them home,” A(shouting B(to cry C(utter D(announced 48(PC Ian McQueen on the same night said he was told:“Mind your own _____,cops(Keep out of it(We will settle these niggers our way(We'll murder the bastards(” A(matters B(affair C(things D(business 49(The disturbances continued night after night until they finally petered out on September 5(At the Old Bailey Judge Salmon later handed down exemplary _____ of four years each on nine white youths who had gone“nigger hunting”( A(decisions B(statements C(trials D(sentences 50(While those dealt with by the courts were overwhelmingly white ,the large number of black people also arrested and the official _____ there had not been a racial motive ensured a legacy of black mistrust of the Metropolitan police that has never really been eradicated( A(persistence B(perseverance C(insistence D(instance Section B Error Correction(10points) Directions:The following passage contains 9 errors(In each case only one word is involved(You should proofread the passage on the Answer Sheet and correct it in the following way: EXAMPLE One night,quite late,I was still awake in the room I am shared with 1. am my husband(I was lying on my right side and can hear a child crying( 2. could Getting up,I went ? see if our son was all right( 3. to He was sleeping soundly,breathing deeply and gently( 4. ? The Zipper Whatever did we do before the invention of the zipper, In 1893 the world's first zipper was produced in Chicago( Although the inventor claimed that it was a reliable fastening for clothing,this was not the case(The Chicago zipper sprang 51(______ open without warning,or jammed shut,and it swiftly lost popularity(Twenty years ago a Swedish-born engineer called 52(______ Sundback solved the problem(He attached tiny cups to the backs of the interlocked teeth,and this meant that the teeth 53(______ could be enmeshed more firmly and reliably( At first zippers were made of metal(They were heavy,and if they got stuck it was difficult to free(Then came nylon 54(______ zippers which were lighter and easier to use,and had smaller teeth(The fashion industry liked the new zippers far better because they didn’t distort the line of the garment or weighing 55(______ down light fabrics(They were also easier for the machinists to fit into the garment( Meanwhile a new fastening agent made its appearance at the end of the twenty century: velcro. Velcro is another product 56(______ made from nylon(Nylon is a very tough synthetic fibre first developed in the 1930s,and bearing a name to mind the wearer 57(______ of the two places where it was developed:NY for New York and LON for London(Velcro is made with very small nylon hooks on one side of the fastening which caught tiny looped whiskers on the 58(______ other side of the fastening(It is strong and durable( Velcro is used on clothing,luggages and footwear(It is quick 59(______ and easy to fasten and unfasten,and has taken a large part of the zipper's share of the market(It is also used in ways a zipper cannot be used—for instance as an easily changed fastening on plaster casts,and to hold furnishing fabrics in a position( 60(______ Part III Situational Dialogues(5 minutes,10 points) Directions:Complete the following dialogues by choosing the best answer(Then mark the corresponding letter on the Answer Sheet with a single line through the centre( 61(Rob:Hey Jill,you're looking great( Jill:Thanks,Rob(____________ Rob:Well,you did it(How, Jill: I jog every morning,and I go to aerobics every other day( A(I bought this dress yesterday(Really smart( B(You are looking fine too( C(I'm recovering my strength after the flu( D(My New Year's resolution was to get in shape( 62(Bob:Hi Jane(How are you, Jane:____________I didn't sleep a wink last night(The people next door were making a lot of noise again till very late at night( A(I'm feeling a bit out of sorts this morning( B(Fine,thank you(And you, C(I slept like a log and didn't want to get out of bed( D(It seems a bit unusual,you know( 63(Ann :Aah~He's gorgeous~Look at those big,golden paws(When did you get him, Roger:Yesterday(____________ Ann :Oh,right(What kind is she, Roger:A Labrador( A(Susan's got a more beautiful one( B(What's up, C(It's a she actually( D(Isn't it right, 64(Tina:Wow,look at all the things on sale(____________ Andrew:Yes,look,this shirt is 50 ,off( Tina:And look at these shoes(They are 30 ,off the normal price( A(I'd like to buy a skirt(B(There are some real bargains( C(Are the prices reasonable,D(These shoes are the same as mine( 65(Woman:Have you finished the packaging, Man :____________ Woman:Good(Because the truck will be coming soon,this is a rush job( A(Don't hurry m or I'll break the glass( B(Almost(I just have to wrap the glass and put it into boxes( C(No,I haven't(Why didn't you help me with it, D(Yes,I have(What else can I do for you, 66(Customs Officer :________________________ Mrs(John son :No,nothing at all( Customs Officer :No perfume,alcohol or cigarettes, Mrs(John son :Well,I have 200 cigarettes;that's all( A(Do you have anything in the bag,ma'am, B(Do you have anything to declare,ma'am, C(Do you want to buy something,ma'am, D(Is there anything I can do for you,ma'am, 67(Linda:Hello(I'd like to send this package,please( Clerk:____________________________________ Linda:First class(How long will that take, Clerk:About three days( A(How would you like to send it, B(Which class are you in, C(Where do you want to send it to,D(Which class is it in, 68(Assistant:Can I help you, Colin :Yes,it's about this sports shirt(I washed it the other day(The colour ran and it shrank( Assistant:Oh dear,I see(________________________ Colin :I'm afraid not( Assistant:I'm sorry,but I'm not allowed to change anything without a receipt( A(Did you buy it here, B(Would you want to change it, C(Do you have the receipt, D(Could you tell me who sold it to you, 69(James:Could I have my bill,please,Can I pay by credit card or eurocheque, Receptionist:____________ James:I'll pay by credit card,then( Receptionist:That's fine(I hope you enjoyed your stay here( A(Here's your bill(B(Sorry,we don't take credit card( C(You can pay by eurocheque(D(Yes,we take both( 70(Husband:When is our anniversary, Wife:________________________ Husband:No,it's just that I bought these flowers for you and I was hoping today was the day( A(Hmm (((I can't remember either(Why, B(Hey,are these flowers for me, C(Who cares,Do you want to give me a surprise, D(Are you joking,Have you really forgotten again, Part IV Reading Comprehension(25 minutes,40 points) Section A Multiple Choice(10 points) Directions:There is one reading passage in this part(The passage is followed by 5 questions or unfinished statements(For each of them there are four choices marked A,B,C and D(You should decide on the best choice and mark the corresponding letter on the Answer Sheet with a single line through the centre( Questions 71 to 75 are based on the following passage: Taking a peep at what's going on in your head CARL Filer,18,a star salesman at a B,Q hardware store in the UK,was called up for promotion within one week of starting work(But,instead of being made supervisor,he was sacked—after his employers saw the results of his psychometric test( You might think that anyone who answers that he“strongly disagreeshe is an over-achiever is asking for trouble,but Mr Filer already thought he had proved himself more than capable( This year,nearly half of UK firms—46 percent—will use psychometric tests to select trainees, compared with just 17 percent in 2000,according to a report for GTI,a publisher of graduate career guides( These tests,which rate candidates’ ability and gauge their personality,have been used in the UK since the 1980s(But assorted studies have shown most people—graduates in particular—are wholly cynical about the idea of their personality being“measured( “People tend to see them as either too silly or too clever,says Clive Fletcher,professor of occupational psychology at University of London(“But all the evidence indicates the tests do have some value( The first personality test as we know it,was developed by the American army in 1917 to filter out weak recruits( But it was not until the 1980s that the tests became popular in Britain(With a rising number of graduates going for a decreasing number of jobs,organizations began to see psychometric testing as a cheap,reliable alternative to the expensive,time-consuming interview( But today the tests are becoming alarmingly sophisticated and are edging towards probing the“dark side:pathology and personality disorders(Increasingly,tests are being used to try to detect promising young graduates who may,later in life,fly off the rails(go crazy);or to stop psychopaths(having mental disorder)getting recruited( In the future,interviewees could even be given a mouth swab to reveal the genetic and biological markers of personality(“We are heading for the era of genetic screening,”warns Carolyn Jones,of the Institute for Employment Rights(“I think these tests are very flawed( And there are other problems with the tests(For starters,it is possible to fake it—even the test producers agree on this(But they have made it as hard as possible(For example, look at whether you agree or disagree with the following two statements:“New ideas come easily to meand“I find generating new concepts difficult(How long did it take you to realize they both could mean the same thing, The main argument,however,is that the tests are invalid and cannot quantify(put a numerical value on)something as changeable as personality( The golden rule is then,that a psychometric test should never be used as the sole basis of selection,but should always be followed by interviews( 71(Most people's attitude towards the psychometric test is ______( A(contemptuous B(favorable C(tolerant D(confounded 72(Which of the following is one of the reasons why psychometric testing wins an advantage over interviews, A(It doesn't cost any money( B(It requires no equipment( C(It is time-saving( D(It can be done within seconds( 73(Which of the following statements is the author's idea, A(Psychometric tests are defective( B(Psychometric tests should not be the only way to recruit promising young graduates( C(Psychometric tests are invalid and cannot quantify something changeable as personality( D(Psychometric tests are golden rules( 74(The test producers make the tests very complicated to ______( A(avoid cheating B(improve genetic screening C(find out the best ideas D(generate new concepts 75(Which of the following is not true according to the passage, A(The American army developed the first personality test to screen out weak recruits( B(In the future,interviewers could give a mouth swab to reveal interviewees’ symptoms( C(There are possibilities for starters to cheat in the psychometric tests( D(Interviews still play an important role in evaluating interviewees( Section B Short Answer Questions(30 points) Directions:In this part there are 3 passages with 15 questions or incomplete statements(Read the passages carefully(Then answer the questions in the fewest possible words(not exceeding 10 words)(Remember to rewrite the answers on the Answer Sheet( Questions 76 to 80 are based on the following passage: The 8 Steps of Social Invention 1(Get ready to play( Like other types of creativity,social inventiveness flourishes when you begin thinking outside conventional boundaries(Charlie Girsch,a St(Paul,Minnesota-based creativity consultant,suggests that you start by playing with obviously absurd explanations for everyday events(“If traffic is slow, you'll be tem pted to say,‘Hmm(Must be an accident up ahead(’ Instead,try saying,‘Must be a family of turtles crossing the highway’or‘I expect there's some kind of alien abduction going on(’ You'll be amazed how soon you will be looking at familiar problems in new ways(”Girsch's book, Fanning the Creative Spirit(Creativity Central,1999)has scores of other exercises for limbering up the inventive part of your brain( 2(Generate a zillion far-fetched ideas( Concerned about the homeless in your neighborhood,Imagine a Homeless Parliament,a Homeless Circus,homeless families forming an orchestra,a homeless museum (((and on and on(Generate like mad with no regard for feasibility in order,as social invention pioneer Nicholas Albery advises, to“overcome e worthy-but-dull ideas(”Eventually the two or three best ideas will begin to stand out( 3(Take your wildest idea and bring it down to earth( How about that Homeless Circus,Could it turn into a forum for homeless people to display their creative talents,A performance series about homelessness,A neighborhood carnival with the homeless as guests of honor,Your flakiest idea may have a germ of brilliance that actually makes it more attractive,and thus more feasible(and fundable),than its worthy-but-dull cousins( 4(Look for in venations that solve more than one problem( The Slow Food Movement,born in Italy,boosts local farmers and regional cuisine traditions and restaurateurs and the same time that it“feeds”our hunger for authentic tastes, healthy eating,and a more leisurely,saner style of life( 5(Accentuate the positive( “A very common question that I get when I work with people in communities is‘Why doesn't anybody care about our problems,’”notes M chael Patterson,a social inventor and activist in Massachusetts(“What a worthless question(‘Why’,questions are for philosophers(Ask‘How’, and‘What’,questions—they are a lot more practical(”For instance, Patterson asks,“What would you do if you knew you couldn't fail,” 6(Give it a rest( Walk away from your favorite idea for a while,forget about it,let it sleep(With your conscious mind out of the way,your subconscious gets to fiddle with the concept for a while, and you just might have an unexpected insight or breakthrough( 7(Practice“yes and”in stead of“yes but”( No matter how tempted you are to say“Yes,but this will be hard because,”or“Yes,but a million other people are doing this,”shift the conjunction to“and”and see what sort of positive refinement or change emerges(“Yes,and we could concentrate on immigrants(”“Yes,and we can make it open to all ages(” 8(Get your idea into the world( This is the tough part(You might seek out the help of activists who will take a shine to your ideas(Or become an organizer yourself(Paul Glove,a New York social inventor,coun-sels:“If you have an idea you believe in,write a pamphlet with your phone number on it and post it in Laundromats and bookstores(If three people call you,have lunch with them and call yourselves an organization(If five people call,meet with them and issue a press release(” Presto,you're launched( 76(To generate far-fetched ideas helps to ______( 77(Michael Patterson wants us to come up with“How”,and“What”,instead of“Why”,questions because he considers they are more practical than ______( 78(The purpose to practice“yes and”instead of“yes but”is to make yourself more ______( 79(According to the article,when one has difficulty developing his favorite idea,he should ______( 80(One should not only generate far-fetched ideas but also ______ because the latter step is the nearest to reality( Questions 81 to 85 are based on the following passage: Thin Slice of TV Has Big Market It is too early to write an obituary for bulky picture tubes,which will remain the most affordable TV sets for years to come( But,analysts and industry executives insist that thin screens already have started to become the dominant format for TV sets in the digital era( Sharp price cuts have brought plasma sets and other thin,flat televisions out of high-end electronic boutiques and into thousands of mass-market outlets such as Cosco,a wholesale buying club in the US,best known for offering members bulk items and big discounts( The least expensive plasma sets still cost a hefty US ,3,000or more ,yet sales are growing so rapidly that many manufacturers are racing to boost production( That increase,combined with expanding production capacity and improved technology, could push the price of plasma sets down by one-third next year,according to analyst Richard Doherty of Envisioneering Group,a US research firm( But manufacturers are not just competing with each other;they are also trying to fend off challenges from competing thin-screen technologies,such as liquid crystal displays(LCD)( The demand for thin screens is fuelled in part by the advent of DVDs and digital TV broadcasts, which offer more detailed pictures and more lifelike colors than conventional analog TV signals( To see the difference,consumers need a set that can pack more information onto the screen than their current TVs can( This sharpness is most vivid on screens that are 40inches diagonal or larger(At that size,however, traditional direct view and projection TVs are so bulky that many consumers have trouble finding a place for them at home( Hence the interest in thin screens—models slender and light enough to hang on a wall( The glass panels at the heart of plasma and LCD sets come mainly from about a dozen companies with factories in Japan,South Korea and,increasingly,China(About 800,000 plasma panels will be shipped this year around the world,analysts say( That is a tiny amount compared with the overall market for TVs,which was about 140 million sets last year(But,industry experts said 2003would be a“breakout year” or plasma because shipments should double( Helping drive the growth are new or expanded manufacturing facilities(For example, Japanese electronics giant NEC last year doubled the capacity of its Japanese factory—reaching 300,000to 400,000 plasma panels(And it plans to double it again in 2003,officials said( As competition has heated up during the last four years,prices have fallen more than 50 percent(According to“NPD Tec world”,the average price of a plasma display sold in the US dropped from US ,12,700in January 1999 to US ,6,100in October 2002( The best markets for plasma screens have been in Asia,and about half of the sets have gone to businesses instead of homes( LCD TVs carry a premium price—they can be 10 times as expensive as a comparable tube-driven television—that knocks them out of most buyers’ budgets( But LCD panels are quickly taking over the market for computer monitors,and the tens of millions of panels being produced for that segment will help push down prices for LCD TVs,analysts predicted( Sharp Electronics,for one,is betting heavily on LCDs(Its chairman,Toshiaki Urushisako, has predicted that Sharp will switch completely from conventional tube sets to LCD TVs in Japan by 2005( Flat-panel refers to wafer-thin(3 inches or less)TVs,whereas flat-screen may actually describe traditional cathode-ray-tube sets(CRTs)whose glass front lacks the distorting curve that TVs have had for 50 years( Be aware of two things:One,flat-panel technology may not be high-definition TV;for eventual HDTV reception,some of these sets will require a separate HD tuner(Two,some flat-panel TVs are just the panel and lack speakers and sometimes a built-in tuner( Price range:US ,700,2,000 LCD vs plasma In general,LCD technology is used for smaller screens because of the enormous number of transistors needed to turn the glasslike liquid crystals into color images(The larger the display,the more transistors,the more chance of failed connections( A plasma screen is found in TV sets larger than 20 inches(Color is comparable to an LCD's(LCDs do not deteriorate over time,while a plasma display averages 30,000 hours(a traditional TV screen can go for 20,000),after which it fades over a period of years(Earlier problems with the quality of plasma's contrast have been addressed,and current screens are cleaner and better defined( Price range:US ,600,2,800 HDTV Simply put,high-definition TV is 10 times as sharp as traditional TV,and the sound is digital,like CD sound,not FM ,which is what traditional TV provides(HD technology achieves its visual clarity with more immage lines on the screen(Where analog TVs have 480 horizontal lines,HDTV has 720 or 1, 080lines(Be aware :m any HDTV sets being sold now are in fact only HDTV monitors ,offering a crisp picture (To receive genuine high-definition television signals,owners must buy a separate HDTV receiver( Price range:US ,1,000,6,000 LOS ANGELEST IMES 81(According to the article,TV sets with _____will still be the most popular in the coming years( 82(The factors that stimulate the thin screens to be more and more popular include _____,_____, and_____( 83(The rapidly expanding market for LCD panels and their large-scale production will help lower _____( 84(When a wealthy customer wants to buy a very large TV,he should select _____and _____according to the passage( 85(We can infer from the passage that among all kinds of TV sets _____is of the best quality( Questions 86 to 90 are based on the following passage: Dell Does Domination Over the years I've spent a fair amount of time hanging out with Michael Dell,and what I noticed during my latest visit with him in Austin is how things have changed(Yes,he is still unflappable(And yes,he greets me in his new glossy offices with the same Stamford Wife-like grin he has always had(But he appears thinner now,as if he's lost baby fat(While he's still slow-moving,as if he's conserving energy,he now cuts to the quick in conversation(And when he zeroes in on the point he wants to make,when he reiterates why Dell Computer is in a better position than any other PC maker in the world,you realize that the 36-year-old has lost what was once one of his greatest advantages: no one underestimate ates him anymore( Instead,Michael Dell looms over the PC landscape like a giant,casting a shadow over all his unfortunate com petitors(This is a terrible time in a difficult business(PC sales were down for the first time last year(Dell's sales will be down,too,also for the first time(Yet even with that,even with recession ,even with the threat of a Hewlett-Packard ,Com paq Goliath,this is the only PC maker you can count on to grow and grow and grow(Almost single-handedly,Dell is forcing this industry to consolidate(Could this mean“game over”in the PC biz,“Game over,”he looks back at me incredulously(“No way(We only have 14, global market share(” The Dellites may not admit to“game over”aspirations,but clearly they are thinking of a kind of domination never seen before among PC makers(“We think 40,market share is possible,”says Dell's No(2,Kevin Rollins(That's a remarkable goal;what's more remarkable is that it really is attainable(Don't look for Dell to hit that kind of number anytime soon(Rather,the company's growth will come from grinding out gains on several existing fronts,while shrewdly expanding into new target markets( The reason is simple:there's no better way to make,sell,and deliver PCs than the way Dell does it, and nobody executes that model better than Dell(By now most business people can recite the basic tenets of Dell's direct-sales model(Dell machines are made to order and delivered directly to the customer(There is no middleman(The customer gets the exact machine he wants cheaper than he can get it from the competition(The company gets paid by the customer weeks before it pays suppliers(Given all that,the company that famously started in Austin out of a University of Texas dorm room now dominates the northern side of this city the way giant steelworks once lorded over old mill towns(Dell has some 24 facilities in and near Austin and employs more than 18,000 local workers(Dell did over ,30billion in sales in 2000,ranking 48th on the FORTUNE 500,ahead of names like Walt Disney,and Du Pont(Michael is the richest man under 40 in the world,worth ,16 billion( Two facts show how well the Dell model is working,even in tough times:Dell is on track to earn over ,1.7 billion in 2001,taking almost every single dollar of profit among makers of Windows-based PCs(And Dell is gaining market share(That's not true for any other major PC maker( Quite the contrary(The others are going splat for the same reason that Dell is succeeding: commoditization(The desktop PC has become a commodity(That's great for consumers,who get standardized,easy-to-use,cheap PCs( Commoditization has been going on in the industry for years(Dell,as master of the direct model, spent most of the 1990s operating in techno-Nirvana(The PC market was growing by 15,-plus per year(For its quarter ended January 2000,Dell did a record ,6(8 billion in sales,up 31, from the previous year's quarter(In a sign of things to come,sales growth slowed later in 2000(Then the growth disappeared in 2001( The economic slowdown was bad news for everyone,but Michael Dell and Kevin Rollins,who is increasingly his equal partner in running this business,made sure it was terrible news for Dell's com petitors(In late 2000they decided to slash prices(“It was advantageous for us,actually,because in periods of slow demand component prices drop,and, unlike our competition,we can pass those savings on immediately to customers,”explains Rollins,a fine violinist who grew up in a hard-charging Utah family—his father was an engineering professor at Brigham Young—and came to Dell from the Bain consulting firm (Dell could make more money selling more computers at lower prices than it could selling fewer computers at higher prices(The low prices wreaked havoc on competitors(Compaq,HP,and Gateway all lost market share for the 12 months that ended Sept(30, 2001,while Dell's share of the U(S(market climbed 31,( 86(_____was once one of Dell's greatest advantages( 87(The passage attributes the success of Dell to_____( 88(Why did Dell's share of the U.S(market climb 31, in 2001 while the economy slowed down, 89(Dell hopes to increase its global market share by _____according to the passage( 90(If one wants to buy a Dell computer,who do you think he is supposed to pay, Part V Word Guessing and IQ Test(5 minutes,10 points) Section A Word Guessing(5 points) 91(In the western countries,a person who serves the guests in a restaurant expects a gratuity in appreciation for good service( A(good remark B(letter of thanks C(tip D(promotion 92(In Greek mythology,the hero usually finds himself in a terrible predicament in which he has to face a fate which has been foretold by the Oracle at Delphi( A(strange phenomenon B(unpleasant situation C(unusual circumstance D(huge monster 93(A first-class business establishment will fully recompense its customers if there is ever any dissatisfaction with the service or product( A(compensate B(understand C(inform D(award 94(Our family was truly a matriarchy,because my grandmother had to raise her children and provide for them all by herself;my grandfather had died when his sons and daughter were quite young( A(family governed by women B(family without the father C(family depending on the mother D(family with many young children 95(I'd love to help you look for all those things but I'm a bit pushed for time today(Why don't we split up and meet back in a couple of hours for a quick cuppa and then I can have a look at what you've bought( A(put it aside B(go separately C(save the time D(do it one by one Section B IQ Test(5 points) 96(What number should replace the question mark, A(10 B(9 C(8 D(7 97(If someone could take a sheet of paper 0(1mm thick,tear it in half,put the pieces on top of each other,tear these in half,then repeat the process until the paper had been torn 25 times,what would be the final height of the pile of paper, A(As thick as a book( B(As tall as a man( C(As high as a house( D(As high as a mountain( 98(If the pyramid were flattened out would it look like A,B,C or D, 99(Only one of the sets of six letter below can be rearranged into a six-letter word in the English language(Can you find that set, A(LO RIDM B(ETNMIU C(TUBLID D(MIRCEL 100(These are millennium celebrations taking place in 4 different places(Which one occurred first, A. PARIS B. SYDNEY C. NEW YORK D. GREAT WALL OF CHINA Part VI Translation(10 minutes,10 points) Directions:Translate the underlined sentences of the following passage into Chinese on the Answer Sheet( TASTE A WORLD OF DIFFERENCE Wherever you are in the world,it seems,the billboards and supermarket shelves are saturated with the same brand names(There are few places left on the planet where you can't buy a Diet Coke,a packet of Kellogg's cornflakes,or Oxo cubes( But just try tasting them((101)The packaging and the product may look identical,but the flavour maybe far from familiar(Even the most famous brand names are specially for-mulated to appeal to individual national palates(Heinz,for instance,insists that its tomato ketchup is“the same recipe worldwide”,but admits that“there maybe very subtle variations in the spicing”( Even the ultimate global brand,Coca-Cola,accepts that not all its products are what they appear to be(“We go to great lengths to ensure that Coca-Cola is the same wherever you drink it,”says a spokesman(“But Diet Coke may change slightly from country to country,because we use different sweeteners in different places(” Americans prefer many products,particularly chocolate,to be far more sugary than Europeans do((102)The French perceive strawberry flavour in a different way from the British,so the artificial flavouring in France will not have a“proper”strawberry taste to Britons(Preferences for saltiness and color differ as well,and most Japanese consumers dislike the taste of milk( The French in general prefer stronger,fuller-tasting coffee than the British(There is a higher cream content in a Magnum in Italy than in Britain,because the Italians expect ice-cream to have a much richer taste(Tea sold in Europe tends to be weaker than the strong tea preferred by Britons((103) Taste preferences can vary even within countries:at least one well-known soft drink is made according to a sweeter recipe in the southern states of the United States than that on sale in the north( Local water and soil will affect the taste of home-grown natural ingredients(Some key constituents may not be available,meaning that alternatives must be found(Sometimes,especially in developing countries,premium grade components,such as high-grade flour,are replaced with lower-quality equivalents( (104)Extremes in climate will require different additives and preservatives to be used( Chocolate calls for an alternative recipe in hot countries if it is to maintain its texture and taste( And multinationals must conform to national laws and regulations on additives, flavorings, colorings,and artificial low-calorie sweeteners(Mars and Snickers bars taste slightly different in Australia,since laws there state that vegetable fat must not be used in chocolate( Local religious sensibilities must also be observed((105)Even McDonald's,which proclaims the homogeneity of its Big Macs(巨无霸)around the globe,has to serve lamb rather than beef burgers in India,because the sacred status of cows means that the majority of its Indian customers can't eat beef( Part VII Writing(30 minutes,20 points) Directions:The chart below shows the amount of leisure time enjoyed by men and women of different employment status(Write a report for a university lecturer describing the information shown below(You should write at least 150 words( Leisure time in a typical week:by sex and employment status,1998,99 2003年全国大学生英语竞赛初赛赛卷答案 Part I Listening Comprehension (30 分) Section A Dialogues (10 分, 每题1分) 1. C. 2. C 3. C 4. B 5. D 6. C 7. A 8. A 9. B 10. B Section B News Items (10 分, 每题1分) 11. The UN Security Council’s resolution. 12. $100 million. 13. About a deadly shooting. 14. 14. 15. A way to reduce early births among women at the risk of premature delivery. 16. The Bush administration. 17. A fire in a night club. 18. To seek UN approval for war. 19. Murder and injury. 20. A gunshot. Section C Compound Dictation (10 分,21—28题,每题0.5分;29、30题,每题3分) 21. purchasing 22. firms 23. engage 24. factors 25. concerned 26. climate 27. introducing 28. characters 29. This critical problem of proper translating is only one of many cultural differences facing American corporations overseas 30. Because of their cultural heritage, businesspersons in each country conduct their activities differently. Part II Vocabulary and Structure (30 分) Section A Multiple Choice (20 分, 每题1分) 31. B 32. D 33. A 34. C 35. C 36. A 37. D 38. B 39. B 40. A 41. D 42. C 43. B 44. C 45. B 46. B 47. A 48. D 49. D 50. C Section B Error Correction (10 分, 每题1分) 51. ? 52. ago? later 53. interlocked ? interlocking 54. free?? them 55. weighing? weigh 56. twenty ? twentieth 57. mind ? remind 58. caught ? catch 59. luggages ? luggage 60. a ? 删除a Part III Situational Dialogues (10 分, 每题1分) 61. D 62. A 63. C 64. B 65. B 66. B 67. A 68. C 69. D 70. D Part IV Reading Comprehension ( 40 分) Section A Multiple Choice (10分, 每题2分) 71. A 72. C 73. B 74. A 75. B Section B Short Answer Questions (30分, 每题2分) 76. increase your social inventiveness 77. theoretical/philosophical 78. determined 79. give it a rest 80. get them into the world / make them known 81. bulky picture tubes 82. price cuts, convenience, the demands for high definition 83. prices for LCD TVs 84. LCD, HDTV 85. HDTV 86. That he was once underestimated / That someone once underestimated him 87. The direct sales model / commoditization 88. They slashed the prices. / They lowered the prices. 89. 26% 90. The Dell company. Part V Word Guessing and IQ Test (10分) Section A Word Guessing (5 分,每题1分) 91. C 92. B 93. A 94. A 95. B Section B IQ Test (5 分,每题1分) 96. C 97. D 98. A 99. B 100. B Part VI Translation (10 points) 101. 包装和产品也许看起来是完全一样的,但是味道也许跟所熟悉的相差甚远。 102. 法国人对草莓口味的感觉与英国人不同,所以法国产的草莓人工调味剂不合英国人的胃 口。 103. 甚至在一国之内,口味偏好也会有极大差别:至少一种指明软饮料,在美国南部地区是用 比在北方销售的饮料甜味更重的配方制成的。 104. 气候的巨大差别要求使用不同的添加剂和防腐剂。 105. 即使宣称在全球销售同一种类的巨无霸的麦当劳也不得不在印度提供羊肉汉堡而不是牛 肉汉堡,原因是牛的神圣地位意味着大多数印度顾客不能食用牛肉。 Part VII Writing (20分) (参考范文) The chart shows the number of hours of leisure enjoyed by men and women in a typical week in 1998-99, according to gender and employment status. Among those employed full-time, men on average had forty-seven hours of leisure, whereas women had approximately thirty-seven hours. There were no figures given for male part-time workers, but female part-timers had thirty-nine hours of leisure time, only slightly more than women in full-time employment, perhaps reflecting their work in the home. In the unemployed and retired categories, leisure time showed an increase for both sexes, as might have been expected. Here too, men enjoyed more leisure time---over eighty hours, compared with less than seventy hours for women, perhaps once again reflecting the fact women spend more time working in the home than men. Lastly, housewives enjoyed approximately fifty-four hours of leisure, on average. There were no figures given for househusbands! Overall, the chart demonstrates that in the categories for which statistics on male leisure time were available, men enjoyed at least ten hours of extra time. 2004年全国大学生英语竞赛初赛试题 Part I Listening Comprehension (30 minutes, 30 points) Section A Dialogues (10 points) Directions: In this section, you will hear 10 short dialogues. At the end of each dialogue, a question will be asked about what was said. Both the dialogue and the question will be read only once. After each question ,there will be a pause. During the pause, you must read the four choices marked A, B, C and D, and decide which is the best answer. Then mark the corresponding letter on the Answer Sheet with a single line through the centre. 1. A. In San Francisco. B. At an airport. C. At a travel agency. D. In a post office. 2. A. The woman is going out to lunch. B. The woman wants to eat some chocolate. C. The woman will go to a convenience store. D. The woman will be back in 30 minutes. 3. A. By car. B. By plane. C. By train. D. By ferry. 4. A. She had lost her job. B. She didn’t know the mayor. C. She was mistaken. D. The man misunderstood her. 5. A. He needs some tomato juice. B. His shirt is stained. C. He needs his shirt by tomorrow. D. His shirt is missing. 6. A. To a meeting. B. To the office. C. To a restaurant. D. To a bowling class. 7. A. Give the woman some medicine. B. Find out more about the woman’s injury. C. Test the strength of the woman’s shoulder. D. Go skiing with the woman. 8. A. Excited. B. Thankful. C. Somewhat disappointed. D. Somewhat bothered. 9. A. When her family celebration is over. B. After the man graduates from school. C. After they have some pictures taken together. D. When she has bought her cap and gown. 10. A. By continuous assessment. B. By giving a per cent. C. By giving grade. D. By means of exams. Section B News Items (10 points) Directions: In this section, you will hear 10 short pieces of news from BBC or VOA. After each news item and question,there will be a pause. During the pause, you must read the three choices marked A, B and C, and decide which is the best answer. Then mark the corresponding letter on the Answer Sheet with a single line through the centre. 11. A. Under the age of four. B. Under the age of five. C. Under the age of six. 12. A. Beijing’s successful bid for the 2008 Olympic Games. B. The Organizing Committee of the 2008 Olympic Games. C. The large market of the Olymic brand. 13. A. To keep the code for its Windows operating system a secret. B. To design some new computer software. C. To persuade more PC users to adopt the Windows operating system. 14. A. One. B. Ten. C. Thirty. 15. A. No. B. Yes. C. Not mentioned. 16. A. More than 500 dollars. B. A little more than three dollars. C. Less than three dollars. 17. A. Because the Iraqi economy has gradually risen after the war. B. Because Iraqi people trust the new dinar more. C. Both A and B. 18. A. Five. B. Six. C. Seven. 19. A. Low fruit and vegetable intake. B. Smoking and little exercise. C. Unhealthy diet. 20. A. The euro has risen in value. B. The US dollar has risen in value. C. German economy has slided into recession. Section C Passages (10 points) Directions:In this section, you will hear 2 passages. At the end of each passage, you will hear 5 questions. After you hear a question, you must choose the best answer from the four choices marked A, B, C and D. Then mark the corresponding letter on the Answer Sheet with a single line through the centre. Passage One 21. A. Jazz. B. Indian. C. Country. D. Pop. 22. A. Their hairstyles. B. Their humor. C. Their clothing. D. All of the above. 23. A. America. B. England. C. Italy. D. Canada. 24. A. The Beatles were formed in England. B. The Beatles had a successful movie career. C. The Beatles are regarded as one of the finest jazz groups. D. The Beatles first recorded music in 1962. 25. A. Seventeen. B. Twenty-eight. C. Twenty-two. D. Twelve. Passage Two 26. A. Because he regarded the bear as his friend. B. Because the bear was beautiful. C. Because he considered it poor sportsmanship to shoot a tied-up animal. D. Because bears are not dangerous animals. 27. A. Because Teddy is the nickname for Theodore Roosevelt. B. Because it was then the usual practice to do so. C. Because it was the first time to name toy bears Teddy Bears. D. Because the toy bear was made to look a bit like the President. 28. A. People collect Teddy Bears. B. Teddy Bears can be found in museums. C. The first Teddy Bear was made by Mr. Mitchtom’s wife. D. President Roosevelt shot the black bear in 1902. 29. A. Seven. B. Six. C. Two. D. Five. 30. A. He moved to Florida. B. He became President. C. He drew cartoons. D. He started a toy company. Part II Vocabulary and Structure (10 minutes, 20 points) Section A Multiple Choice (10 points) Directions:There are 7 incomplete sentences and 3 incomplete dialogues in this section. For each blank there are four choices marked A, B, C and D. Choose the one that best completes the sentences and dialogues. Then mark the corresponding letter on the Answer Sheet with a single line through the centre. 31. Never________the power of your actions. With one small gesture you can change a person’s life. A. underestimate B. overvalue C. misuse D. dismiss 32. Scientists have warned that penguins in the Antarctic could be very________to changes in climate and could be threatened by any long-term temperature shifts. A. superstitious B. acceptable C. suspicious D. susceptible 33. Since settling in Scotland I ________ golf as a hobby. A. have taken up B. took up C. have taken in D. took in 34. She often thinks that her six years in Italy were wasted, ________she________ that time learning more Italian. A. but that; might have taken B. for that; should have found C. in that; could have spent D. with that; would have used 35. He constantly________his proposal that________of the budget surplus be used to offer a voluntary prescription drug benefit to seniors. A. views; many B. reiterates; a part C. complains; a great amount D. thinks; lots 36. ________Alan’s amazement, the passport office was closed when he arrived. A. With B. For C. To D. Of 37. I was asked the other day whether high and low pressure systems were________the central pressure. A. maintained to B. determined by C. generated within D. preserved to 38. Bob: What are you reading, Frank? Tom: It’s this week’s New Scientist, why? Bob: I was just wondering—________, but I’ve never actually read it myself. Is it aimed at real scientists or can ordinary people like me understand it? A. it’s for anyone really B. where I can buy it C. it seems very expensive D. it looks interesting 39. Girl: Hi Paul—looking forward to your holiday? Boy: Oh, yeah—it’s going to be great. Though I’m a bit worried that I’ve packed the wrong clothes. I don’t think the weather’s going to be as good as I hoped. Girl:________ Boy: That’s right—my first flight. A. Everything will be OK, isn’t it? B. You’re flying on Saturday, aren’t you? C. It’s far from here, as everybody knows. D. That’s a good idea, anyway. 40. John: What plastic products do you have in mind that are easy to recycle? Tom: Shampoo bottles, detergent bottles, medicine bottles, food containers, etc. They are all easily collectable and reusable. John: ________, but actually I think you are missing the point of recycling. It doesn’t just mean using old bottles again and again for the same purpose. What it means these days is melting the plastics down and building them up again into some completely new product. A. Not too bad B. Something is wrong C. You’re right there D. It’s a new idea Section B Cloze-Test (10 points) Directions: There are 10 blanks in the passage. For each blank there are four choices marked A, B, C and D. Choose the one that best completes the sentence. Then mark the corresponding letter on the Answer Sheet with a single line through the centre. Ask most people for their list of Top Ten fears, and you’ll be sure to find being burgled fairly high on the list. An informal survey I carried out among friends at a party last week (41)________that eight of them had their homes broken into more than twice, and two had been burgled five times.To put the record (42)________, none of my friends owns valuable paintings or a sideboard full of family silverware. Three of them are students, in fact. The most (43)________burglary, it seems, involves the theft of easily transportable items—the television, the video, even food from the freezer. This may have something to do with the fact that the average burglar is(44)________his (or her) late teens, and probably wouldn’t know what to do with a Picasso, (45)________selling a Walkman or a vacuum cleaner is a much easier matter. They are perhaps not so much (46)________criminals as hard-up young people who need a few pounds and some excitement. (47)________that this makes having your house turned upside down and your favourite things stolen any easier to accept. In most cases, the police have no luck (48)________any of the stolen goods. Unless there is any (49)________evidence, they are probably unable to do anything at all. And alarms or special locks don’t seem to help either. The only advice my friends could (50)________up with was “Never live on the ground floor” and “Keep two or three very fierce dogs”. 41. A. released B. revealed C. reclaimed D. redeemed 42. A. straight B. clear C. apparent D. correct 43. A. typical B. abnormal C. hazardous D. vicious 44. A. near B. in C. beyond D. out of 45. A. whereas B. whenever C. however D. once 46. A. serious B. professional C. efficient D. perfect 47. A. Given B. Even C. Not D. Despite 48. A. seizing B. withdrawing C. seeking D. recovering 49. A. distinguishable B. obscure C. outstanding D. definite 50. A. come B. catch C. keep D. put Part III Word Guessing and IQ Test (5 minutes, 10 points) Section A Word Guessing (5 points) 51. Social capital has become a mantra for politicians and policy makers: they see it as a bulwark against society’s ills and a means of multiplying the effects of financial investment in social projects. A. measurement B. defense C. treatment D. complaint 52. Her desire for anonymity soon became apparent when she refused to answer questions about her identity. A. recognition B. concealment C. vanity D. success 53. “Gentlemen,” replied Candide, with a most engaging modesty, “you do me much honor, but upon my word I have no money.” A. you help me a lot B. you are great C. it’s very generous of you to say so D. that’s a great honor for me 54. When it comes to listening to the opinions of members of your school community, do you think you are already “all ears?” A. in full strength B. bearing ideas in mind C. ready to listen attentively D. having enough preparation 55. That extremely indolent student will clean out his desk when pigs fly or I am much mistaken as to his character. A. soon B. never C. sometime D. often Section B IQ Test (5 points) 56. Sally had a third again as many as David, who had a third as many again as Francis. Altogether they had 111. How many did David have? A. 27 B. 32 C. 36 D. 48 57. What letter should replace the question mark? A. T B. S C. I D. N 58. BONA FIDE is to genuine as DE FACTO is to________. A. together B. actual C. reason D. assumed 59. How many revolutions must the largest cog make in order to bring the cogs back to their original positions? A. 56 B. 48 C. 36 D. 12 60. The diagram shows a small village church. There is a door in the west end, seen in the diagram. There is a tower at the east end of the church with a window set in its east wall. This wall is hidden in the diagram.There is also a door in the tower. Which of these is most likely to be the view of the eastern end of the church? Part IV Reading Comprehension (25 minutes,30 points) Directions:In this part there are 5 passages with 30 questions or incomplete statements. Read the passages carefully. Then answer the questions in the fewest possible words(not exceeding 10 words). Remember to rewrite the answers on the Answer Sheet. Questions 61 to 66 are based on the following passage: Centuries ago, man discovered that removing moisture from food helps to preserve it, and that the easiest way to do this is to expose the food to sun and wind. Fruit is sun-dried in Asia Minor, Greece, Spain and other Mediterranean countries, and also in California, South Africa and Australia. The methods used vary, but in general, the fruit is spread out on trays in drying yards in the hot sun. In order to prevent darkening, pears, peaches and apricots are exposed to the fumes of burning sulphur before drying. Plums, for making prunes, and certain varieties of grapes for making raisins and currants, are dipped in an alkaline solution in order to crack the skins of the fruit slightly and remove their wax coating, so increasing the rate ofdrying. Nowadays most foods are dried mechanically. The conventional method of such dehydration is to put food in chambers through which hot air is blown at temperatures of about 110? at entry to about 43? at exit. This is the usual method for drying such things as vegetables, minced meat, and fish. Liquids such as milk, coffee, tea, soups and eggs may be dried by pouring them over a heated horizontal steel cylinder or by spraying them into a chamber through which a current of hot air passes. In the first case, the dried material is scraped off the roller as a thin film which is then broken up into small, though still relatively coarse flakes. In the second process it falls to the bottom of the chamber as a fine powder. Where recognizable pieces of meat and vegetables are required, as in soup, the ingredients are dried separately and then mixed. Dried foods take up less room and weigh less than the same food packed in cans or frozen, and they do not need to be stored in special conditions. For these reasons they are invaluable to climbers, explorers and soldiers in battle, who have little storage space. They are also popular with housewives because it takes so little time to cook them. Usually it is just a case of replacing the dried-out moisture with boiling water. Questions: 61. Fruit is sun-dried generally on________. 62. Why are sulphur fumes used before drying some fruits? 63. Where are vegetables commonly dried nowadays? 64. If soup requires recognizable pieces of meat, they are________. 65. Dried foods are often used by________, ________and________. 66. Why do housewives like dried foods? Questions 67 to 72 are based on the following passage: Hollywood writers honor Coppola, “Splendor” Sunday, February 22, 2004 Posted: 9:56 AM EST (14:56 GMT) LOS ANGELES, California (Reuters) —Hollywood’s screenwriters Saturday snubbed the final installment of the highly acclaimed Lord of the Rings trilogy and instead awarded a key prize to a low-budget film based on a comic book writer. American Splendor, which revolves around the travails of comics connoisseur Harvey Pekar, won the Writers Guild of America Award for best adapted screenplay, while writer / director Sofia Coppola’s Lost in Translation, about a pair of mismatched Americans languishing in Tokyo, nabbed the trophy for original screenplay. The American Splendor screenplay was written by the film’s directors, Robert Springer and Shari Springer Berman, who were not present at the awards. The Writers Guild of America Awards were handed out simultaneously in Los Angeles and New York. Coppola said she was excited to be honored by the union. “I find it difficult to write, so it’s very encouraging and exciting to get an award,” Coppola told Reuters after the event. Coppola’s competition was Gurinder Chadha, Paul Mayeda Berges and Guljit Bindra for Bend It Like Beckham, Steven Knight for Dirty Pretty Things, Irish director Jim Sheridan and his daughters Naomi and Kirsten for In America, and first-time writer / director Tom McCarthy for The Station Agent. The other adapted screenplay nominees were director Peter Jackson, Fran Walsh and Philippa Boyens for The Lord of the Rings: The Return of the King, Anthony Minghella for Cold Mountain, Brian Helgeland for Mystic River, and writer / director Gary Ross for Seabiscuit. Coppola, Knight and the Sheridans will vie for the Academy Award next week, along with the writers of The Barbarian Invasions and Finding Nemo. Apart from Cold Mountain, all the Writers Guild of America adapted screenplay contenders will compete for the Oscar, along with the Brazilian drama City of God. In the last 12 years, eight of the Writers Guild of America adapted screenplay winners and seven of its original screenplay winners have gone on to Oscar glory. Lost in Translation has already picked up three Golden Globes—an Oscar bell-wether—including best screenplay. The Lord of the Rings: The Return of the King, which has 11 Oscar nominations, has collected four Golden Globes and prizes from Hollywood’s producers and directors guilds. Questions: 67. What does American Splendor mainly write about? 68. Sofia Coppola’s Lost in Translation won the Writers Guild of America Award for________. 69. The Writers Guild of America Awards were offered at the same time in________and________. 70. How did Coppola feel about her getting the award? 71. List at least 3 movies that competed with Lost in Translation. 72. How many Writers Guild of America screenplay winners have gone on to Oscar glory in the last 12 years? Questions 73 to 78 are based on the following passage: The need for a surgical operation, especially an emergency operation, almost always comes as a severe shock to the patient and his family. Despite modern advances, most people still have an irrational fear of hospitals and anaesthetics. In the early years of last century there was little specialization in surgery. A good surgeon was capable of performing almost every operation that had been devised up to that time. Today the situation is different. Operations are now being carried out that were not even dreamed of fifty years ago. The heart can be safely opened and its valves repaired. Clogged blood vessels can be cleaned out, and broken ones mended or replaced. A lung, the whole stomach, or even part of the brain can be removed and still permit the patient to live a comfortable and satisfactory life. The scope of surgery has increased remarkably in 20th century. Its safety has increased too. Deaths from most operations are about 20% of what they were in 1910 and surgery has been extended in many directions,for example to certain types of birth defects in newborn babies, and, at the other end of the scale, to life-saving operations for the octogenarian.The hospital stay after surgery has been shortened to as little as a week for most major operations. Many developments in modern surgery are almost incredible. They include the replacement of damaged blood vessels with simulated ones made of plastic; the replacement of heart valves with plastic substitutes; the transplanting of tissues such as the lens of the eye. One of the most revolutionary areas of modern surgery is that of organ transplants. Until a few years ago, no person, except an indentical twin, was able to accept into his body the tissues of another person without reacting against them and eventually causing death. Recently, however, it has been discovered that with the use of x-rays and special drugs, it is possible to graft tissues from one person to another which will survive for periods of a year or more. Kidneys have been successfully transplanted between non-identical twins. Heart and lung transplants have been reasonably successful in animals, though rejection problems in humans have yet to be solved. “Spare parts” surgery, the simple routine replacement of all worn-out organs by new ones, is still a dream of the distant future. As yet, surgery is not ready for such miracles. In the meantime, you can be happy if your doctor says to you,“Yes, I think it is possible to operate on you for this condition.” Questions: 73. Most people are afraid of being operated on in spite of________. 74.A patient can still live a comfortable and satisfactory life even after the removal of________. 75. Today deaths from most operations are about ________of what they were in 1910. 76. What’s the main difficulty in organ transplanting? 77. Is “spare parts” surgery possible now? 78. You can be happy if your surgeon can operate because it means________. Questions 79 to 84 are based on the following passage: Sales of anti-ageing skin treatments have reached a new high as British women try to stay younger longer (writes Joanna Bale). But according to a recent survey of those aged between 35 and 55 there are significant regional variations in annual spend on these cosmetics. While the average woman thinks costs of ,200 a year acceptable—almost treble the ,75 of three years ago—some fork out ,500, according to the survey of over 2,000 women nationwide by the beauty company Olay. Brows will wrinkle at the differences in yearly spend on anti-ageing treatments across major cities: the ladies of Edinburgh spent least, just ,50 a year, while those in Leeds spend most, at a staggering ,500. London women most commonly spend ,200 annually, and those in Manchester give themselves a modest yearly budget of ,100. Equally surprising are the results among 40-somethings. Single women spend the least on indulging their desire for facial rejuvenation, with only 25 per cent forking out on skincare products or treatments. This figure rises to just over 31 per cent of married women and a similar figure for those who are unmarried with partners. By far the biggest spenders are the 50 per cent of divorcees who feel the pressure to invest heavily in their facial futures. The survey highlighted two groups who spend significant sums on enhancing their appearances—“Sindies” (single income now divorced), and women in their 40s who use their looks to get ahead. The survey also found that although women wanted to “de-age” they had a holistic view of beauty and embraced a realistic and natural approach to looking good. Questions: 79. Compared with that of three years ago, the average British woman’s annual spend on cosmetics has almost________. 80. Women in________spend most on cosmetics according to the survey. 81. The ladies of Birmingham as well as those in ________spend ,100 a year on antiageing treatments. 82. ________and________spend more on indulging their desire for facial rejuvenation than single women. 83. Enhancing appearances plays an important role in the life of________and________. 84. After reading the passage, what do you learn about the sales of anti-ageing skin treatments now in Britain? Questions 85 to 90 are based on the following passage: It is hard to get any agreement on the precise meaning of the term “social class”. In everyday life, people tend to have a different approach to those they consider higher or lower than themselves in the social scale. The criteria we use to “place” a new acquaintance, however, are a complex mixture of factors. Dress, way of speaking, area of residence in a given city or province, education and manners all play a part. In ancient civilizations, the Sumerian, for example,social differences were based on birth,status or rank,rather than on wealth. Four main classes were recognized. These were the rulers, the priestly administrators, the freemen (such as craftsmen,merchants or farmers) and the slaves. In Greece, after the sixth-century B.C., there was a growing conflict between the peasants and the landed aristocrats, and a gradual decrease in the power of the aristocracy when a kind of “middle class” of traders and skilled workers grew up. The population of Athens,for example, was divided into three main classes which were politically and legally distinct. About one-third of the total were slaves, who did not count politically at all, a fact often forgotten by those who praise Athens as the nursery of democracy. The next main group consisted of resident foreigners, the “metics”, who were freemen, though they too were allowed no share in political life. The third group was the powerful body of “citizens”, who were themselves divided into subclasses. In ancient Rome, too, a similar struggle between the plebs, or working people, and the landed families was a recurrent feature of social life. The medieval feudal system, which flourished in Europe from the ninth to the thirteenth centuries, gave rise to a comparatively simple system based on birth.Under the king there were two main classes—lords and “vassals”, the latter with many subdivisions. The vassal owed the lord fidelity, obedience and aid, especially in the form of military service. The lord in return owed his vassal protection and an assured livelihood. In the later Middle Ages, however, the development of a money economy and the growth of cities and trade led to the rise of another class, the “burghers” or city merchants and mayors. These were the predecessors of the modern middle classes. Gradually high office and occupation assumed importance in determining social position, as it became more and more possible for a person born to one station in life to move to another. This change affected the towns more than the country areas, where remnants of feudalism lasted much longer. Questions: 85. List at least three common criteria for telling a person’s social position. 86. What were the four main classes in the Sumerian civilization? 87. Slaves in Greece in the sixth century B.C. were not________significant. 88. The struggle between the plebs and the landed families was a________feature of social life. 89. The metics,one of the three classes of Greece, consisted mainly of________. 90. What did the development of a money economyand the growth of cities and trade lead to? Part V Error Correction (5 minutes,10 points) Directions:The following passage contains 9 errors. In each case only one word is involved. You should proofread the passage on the Answer Sheet and correct it in the following way: EXAMPLE One night,quite late,I was still awake in the room I am shared with 1. am my husband. I was lying on my right side and can hear a child crying. 2. could Getting up,I went ? see if our son was all right. 3. to He was sleeping soundly,breathing deeply and gently. 4. ? If the air in New York seems a little less grimy this spring, thank Rudolph Giuliani. On January 10th, after months of burning debate, the city’s non-smoke mayor 91.________ signed the Smoke-Free Air Act. From April 10th smoking will be stubbed out(碾灭) in restaurants catering for more than 35 people, a move that will hit about half the city’s 11,000 eating places. Nicotine addicts will also smoked out at work, except 92.________ in ventilated smoking rooms or offices occupied by no more than three consenting adults. More radically, outdoor seating areas will also become smoke zones. 93.________ Come the new baseball season, fans at Yankee Stadium will be breaking the law if they light up. New York joins well over 100 American cities—and four states—that have passed laws banned smoking 94.________ in public places. More than a third of American companies now forbid smoking in the workplace, up to 95.________ a mere 20% in 1986. And the tobacco industry, which in America alone has annual sales of close to ,50 billion, is watching its profits go down in smoke. 96.________ The industry may never recover. Polls suggest that nine out of ten Americans are irritated by cigarette smoke. With good reason. In 1993 the Environmental Protection Agency (EPA) has classified 97.________ “second-hand” smoke as a health hazard—one that,according to the EPA, causes 3,000 non-smokers to die from lung cancer each year. 98.________ New Yorkers must now wait and see if the pro-smoking lobby’s alarming predictions of citywide economical collapse come true. Tobacco 99.________ company Philip Morris may show the way. Last year it threatened to move its 2,000 head-office employee out 100.________ of the city if the smoking ban became law. Part VI Translation (10 minutes, 20 points) Section A English-Chinese Translation (10 points) Directions: Translate the underlined sentences of the following passage into Chinese on the Answer Sheet. What is a brand anyway? A brand isn’t just a logo on a shirt or an ice-cream van. It’s much more than that. To be successful a brand has to have rational characteristics—(101)ie, it has to be competitive on price or quality or service, and it has to have a “personality” that charms and seduces. It is a mix of emotional factors,such as “Do I like it?” and “Is it me?” and rational factors such as,“Is it cheaper or better or quicker?”(102)Getting things in balance is tricky,and that’s why so many brands don’t succeed. Just let’s look at all the ways brands can fail. Brands are vulnerable to fashion. Fizzy drinks such as Coke and Pepsi are now being attacked by stimulation drinks such as Red Bull. Fast food brands are threatened by salads and other “lite” foods. McDonald’s has recently been in real trouble. Brands are also vulnerable because they get cocky,arrogant and out of touch. They think they know best and don’t change with the market—like the Gap, Levi’s and Marks & Spencer brands who learnt their lesson the hard way. (103)In fact, even the people who create brands can’t really control them or even predict how people will use them. Not one single mobile phone company anywhere in the world anticipated the growth of texting.The companies involved in the new 3G phone technology are still holding their breath because they don’t know whether it will take off or not. And if it does take off, they won’t know how and in what direction until the market tells them. It’s a huge gamble.Range Rover was the first Sport Utility Vehicle, but Land Rover never completely understood the concept that it had inspired. Range Rover has spent much of its life span trying to catch up with the trend that it serendipitously created. (104)Organic foods are a great success, but not one single major manufacturer or retailer originally promoted them. Little companies started the trend and we consumers just decided we didn’t want our food mucked around with. So at first slowly, hesitantly and ponderously, the retailers and the manufacturers followed. They did what we asked. (105)In other words, brands are nothing like as powerful as they look. The people who manage them often get things wrong, muck things up, look in the wrong direction and generally act just like most organisations directed by human beings—messily. So despite all the huffing and puffing, the reality is that brands are more or less completely in our power. When we like them we buy them, when we don’t, we just buy something else. And what’s more, as customers we’re unpredictable. We can be loyal or fickle, extravagant or stingy, serially or simultaneously. Section B Chinese-English Translation (10 points) 106. 许多遭到洪水侵害的农场主说,他们别无选择只得解雇一些工人。 107. 有时候对一个人来说需要终生的时间才能懂得活着就是为了奉献。 108. 在美国,由于道路畅通,开车上下班很方便,现在许多在城市里工作的人,喜欢住在乡下。 109. 收音机及电视机使得做广告的人有可能用这种方法吸引千百万人的注意。 110. 纽约劳动力市场近来显现出从二战以来最长时间的低迷中复苏的迹象。 Part VII Writing (30 minutes, 30 points) Writing Task I (10 points) Directions: Yesterday you lost your student identification card. You need it to get discounts on public transport and for the cinema, and to use the college library. Even more importantly, you need it as proof of identity to withdraw money from the bank. Write to the Director of Student Services explaining the situation and requesting a new card as soon as possible. You should write about 120 words. You do NOT need to write your own address. Write your letter on the Answer Sheet. Begin your letter as follows: Dear ________, Writing Task II (20 points) Directions: The following is an advertisement for English teachers.Suppose you are a university graduate,and your future plan is to be an English teacher. After reading this advertisement, you decide to write a statement of interest to GEOS. You should write at least 150 words. Write your statement on the Answer Sheet. 2004年全国大学生英语竞赛初赛 听力录音原文及参考答案 Part I Listening Comprehension (30 minutes, 30 points) Section A Dialogues (10 points) Directions: In this section, you will hear 10 short dialogues. At the end of each dialogue, a question will be asked about what was said. Both the dialogue and the question will be read only once. After each question,there will be a pause. During the pause, you must read the four choices marked A, B, C and D, and decide which is the best answer. Then mark the corresponding letter on the Answer Sheet with a single line through the centre. 1. W: Hi, I’d like to send this package by express mail to San Francisco and I would like to buy a sheet of stamps, please. M: Here are your stamps, and just put the package on the scale. Q: Where did the conversation take place? (D) 2. M: I’m going out to lunch. Do you need anything while I’m out? W: Yes, if you pass a convenience store, get me some chocolate—a Snickers bar, please. Q: What do you learn from this conversation? (B) 3. W: If we go by car, how do we cross the river? M: There’s a ferry that will take your car. There’s even one for trains. Q: How will they cross the river? (D) 4. W: I heard that the mayor is closing the cheese factory. M: Yes, but it is only temporary. W: Oh, I’m surprised. I thought it was going to shut down for good. Q: Why was the woman surprised? (C) 5. M: I spilled tomato juice on my new white shirt. Do you think it will come out? W: That’s too bad. Leave it there and I’ll see what I can do. Q: What is the man’s problem?(B) 6. W: I’m going to lunch with my bowling instructor. M: What about the committee meeting? W: Don’t worry. I’ll be back at the office before then. Q: Where is the woman probably going now? (C) 7. M: How long have you had this problem with your shoulder? W: It started last week after my skiing accident. M: Let’s try some tests to determine the nature of the injury. Q: What is the man going to do? (B) 8. W: Are you having a good time? M: Sure. Thanks again for inviting me. W: No problem. I just wish more people could have come. Q: How does the woman feel? (C) 9. M: We finally made it, Mary! W: I can’t believe graduation is tonight. M: Can you come to my graduation party? W: Sure, after I finish the family celebration. M: I want to be sure we get pictures of us together. W: In our caps and gowns! Q: When will the woman go to the man’s graduation party? (A) 10. M: Hi, did you pass your geography exam? W: Yeah, I did quite well in fact, I got 76%. M: Oh,well done! So they gave you a per cent? I thought they gave grades. W: Yeah, they gave both. Mine was an “A”. So how about you? M: Well, we don’t have exams.We have continuous assessment, so you just have to do coursework, and you get a mark for each essay. Q: How does the school evaluate the man’s progress in geography? (A) Section B News Items (10 points) Directions: In this section, you will hear 10 short pieces of news from BBC or VOA. After each news item and question,there will be a pause. During the pause, you must read the three choices marked A, B and C, and decide which is the best answer. Then mark the corresponding letter on the Answer Sheet with a single line through the centre. 11. Tens of thousands of health workers will go house to house over the next three days in an effort to immunise 63 million children under the age of five in sub-Saharan Africa. The campaign is the start of monthly national immunisation days during the low season for polio. It’s hoped that vaccinating children now—when the virus is at its weakest—will be the best way of stopping transmission. Question:How old are the children to be immunised?(B) 12. Amid pomp and ceremony, China launched the 2008 Olympics. Together with a Chinese counterpart, the president of the International Olympic Committee, Jacques Rogge, used a giant golden key to symbolically open what he called the most important market in the world. In his speech, he emphasised the power of the Olympic brand in China’s emerging market. Question:What does the giant golden key symbolize?(C) 13. Microsoft tries to keep the code for its Windows operating system a closely guarded secret. It’s the equivalent of computer DNA and the firm fears if it falls into the wrong hands it could be used to infiltrate millions of computers worldwide. More than 90 percent of the world’s PCs run Windows. Question:What action does Microsoft intend to take?(A) 14. Before he set off in November, there were fears that Francis Joyon would be unable to control his huge boat, named IDEC. With its three hulls slicing through the water and a massive rotating mast that reached 30 metres into the sky, the boat was built in 1986 for a crew of ten. It was feared that such a boat would be too powerful for one man in the rough seas of the Southern Ocean. Question:How many people can the boat carry?(B) 15. Over timescales of thousands of years, the Earth goes through a natural cycle of warmer and colder periods, driven by changes in heat coming from the Sun. Professor William Ruddiman from the University of Virginia has now calculated that if the Earth had followed its natural cycle over the last ten thousand years, it ought to have got steadily colder. It hasn’t,because, he believes, human activities have been keeping the temperature steady. Question:Has the Earth got steadily colder over the last ten thousand years?(A) 16. Inequality of health care is still paramount, says the WHO’s latest report. Industrialised countries account for less than 20 percent of the world’s population but take 90 percent of health spending. In Japan more than 500 dollars is spent on drugs per person per year. This compares to just three dollars in Sierra Leone. Only slightly more is spent in many sub-Saharan countries. Question: How much do many sub-Saharan countries spend on drugs per person per year?(B) 17. The Iraqi dinar has risen a third or so in value against the dollar since the new banknotes began to circulate. One factor has been the gradual pick up of the Iraqi economy after the devastation of the war. There are simply more transactions taking place, which has supported the value of the currency. And it seems Iraqis trust the new dinar banknotes more than they did the old ones, which featured pictures of Saddam Hussein. Question:Why did the Iraqi new dinar rise in value?(C) 18. The list of countries known to have the relatively new and deadly strain of bird flu is rapidly growing. The focus now is on Indonesia where tests will soon confirm whether or not the bird flu which killed several million chickens there is the often fatal H5N1, already confirmed in 5 other countries in the region. Reports of an outbreak in Laos are also being investigated. Question:What is the number of countries mentioned in this news report?(C) 19. An unhealthy diet together with little exercise and smoking are the key preventable risks of non-communicable diseases and it’s estimated that low fruit and vegetable intake alone causes more than two and a half million deaths each year. Question:What causes more than two and a half million deaths each year?(A) 20. Around Europe interest rates are at their lowest levels in half a century. But businesses are pressing for even cheaper borrowing costs amid signs of continued economic weakness. A big drop in German manufacturing announced earlier this week is cited as evidence that Europe’s most important economy may even be sliding into recession. And the rise of the euro to a four-year high against the dollar in currency dealing is a major worry for many European exporters. Question:What is the key problem for European exporters?(A) Section C Passages (10 points) Directions:In this section, you will hear 2 passages. At the end of each passage, you will hear 5 questions. After you hear a question, you must choose the best answer from the four choices marked A, B, C and D. Then mark the corresponding letter on the Answer Sheet with a single line through the centre. Passage One The world of music will never be the same since the formation of a band in Liverpool, England in 1956. The Beatles were formed by George Harrison, Ringo Starr, Paul Mc-Cartney, and John Lennon. Their first hit song Love Me Do was recorded in 1962. The Beatles quickly became the world’s best-known pop music group and many people today still regard them as the finest band in the history of pop music. Lennon and McCartney were the authors of most of the songs the group recorded. Harrison also wrote songs, often using ideas from Indian music. The drummer of the group was the famous Ringo Starr and he occasionally sang. For six years the Beatles had hit after hit song. Twenty-eight of their songs were on the Top Twenty record charts and seventeen of these songs reached number one on the charts. The group also had a successful movie career. The comedies A Hard Day’s Night and Yellow Submarine became very successful movies. People imitated their long hairstyles, clothing, and humor. Almost all later pop bands learned from the Beatles. Beatlemania is the word used to describe how strong and loyal the fans were. Questions 21 to 25 are based on the passage you have just heard: 21. What kind of music did the Beatles play?(D) 22. What did many people copy from the Beatles?(D) 23. Where were the members of the Beatles group from?(B) 24. Which of the following is NOT true?(C) 25. How many of the Beatles’ songs reached number one on the record charts?(A) Passage Two Have you ever wondered where these cute little teddy bears came from? They were named for President Theodore Roosevelt in 1902. President Roosevelt was on a hunting trip in Mississippi when members of the hunting party caught a black bear and tied him to a tree. President Roosevelt was called to the area to shoot the bear, which he refused to do and said it was unsportsmanlike and showed poor manners. The Washington Post newspaper ran a cartoon showing the President refusing to shoot the bear and people all over America saw the cartoon. Morris Michtom, a shopkeeper in Brooklyn, New York, placed two toy bears in the window of his shop. Mr. Michtom requested permission from the President to call them “Teddy Bears” as Teddy is the nickname for Theodore Roosevelt. The sweet little bears with shiny button eyes were a delight with children everywhere. The Teddy Bears were made by Mr. Michtom’s wife. Mr. Michtom formed a new business called the Ideal Novelty and Toy Corporation. Today, Teddy Bears are treasured toys of children all over the world. They are also collected by people and many are displayed in museums. Teddy Bears are sold by many companies and you can find them in almost any toy store, dressed in costumes or with a ribbon around the neck. Questions 26 to 30 are based on the passage you have just heard: 26. Why did President Roosevelt refuse to shoot the bear?(C) 27. Why did Mr. Michtom ask for the President’s permission to call the toy bears “Teddy Bears”?(A) 28. Which of the following is NOT true?(D) 29. How many Teddy Bears were made by Mrs. Mitchtom and placed in the window of their shop?(C) 30. What did Mr. Mitchtom do after he sold the Teddy Bears in 1902?(D) Part II Vocabulary and Structure (10 minutes, 20 points) Section A Multiple Choice (10 points) 31. A 32. D 33. A 34. C 35. B 36. C 37. B 38. D 39. B 40. C Section B Cloze-Test (10 points) 41. B 42. A 43. A 44. B 45. A 46. B 47. C 48. D 49. D 50. A Part III Word Guessing and IQ Test (5 minutes, 10 points) Section A Word Guessing (5 points) 51. B 52. B 53. D 54. C 55. B Section B IQ Test (5 points) 56. C 57. A 58. B 59. A 60. A Part IV Reading Comprehension (25 minutes,30 points) 61. trays 62. To preserve their colours. (or: To prevent darkening.) 63. In hot-air chambers. 64. dried separately and then mixed 65. climbers, explorers, soldiers 66. Because it takes so little time to cook them. 67. The travails of comics connoisseur Harvey Pekar. 68. original screenplay 69. Los Angeles, New York 70. Encouraged and excited. 71. Bend It Like Beckham, Dirty Pretty Things, In America, The Station Agent. ( Any three of them.) 72. 15. 73. modern advances in surgery 74. the stomach or one lung 75. 20% 76. The body’s tendency to reject alien tissues. 77. No, it has yet to become a reality. 78. your illness may be curable 79. tripled 80. Leeds 81. Manchester 82. Married women, those unmarried with partners 83. “Sindies”, women in their 40s 84. The sales have reached a new high, with regional variations. 85. Dress, way of speaking, area of residence education and manners. (Any three of them.) 86. Rulers, administrators, freemen and slaves. 87. politically 88. recurrent 89. resident foreigners 90. The rise of the burghers. Part V Error Correction (5 minutes,10 points) 91. non-smoke?non-smoking 92. also ? smoked?be 93. smoke?smokeless / non-smoking 94. banned?banning 95. to?from 96. down?up 97. has?has 98. ? 99. economical?economic 100. employee?employees Part VI Translation (10 minutes, 20 points) Section A English-Chinese Translation(10 points) 101. 即它必须在价格或质量或服务方面具有竞争力,并且还应具有能够吸引人们购买的“个性特 点”。 102. 保持诸方面因素的平衡是很需要技巧的,这也正是如此多的品牌不成功的原因所在。 103. 事实上,即使是设计品牌的人也无法左右品牌,他们甚至预测不出人们将如何利用它们。 104. 有机食品是很成功的, 但在最初却没有一个较大规模的生产商或零售商(愿意)推销它们。 105. 换句话说,品牌本身远没有他们看起来那样有影响力。 Section B Chinese-English Translation (10 points) 106. Many flood-ravaged farmers say they will have no choice but to lay off some employees. 107. Sometimes it takes a lifetime for one to realize that to live is to give. 108. Many people in America who work in the cities now enjoy living in the country because of good roads and the ease of commuting by automobile. 109. Radio and television have made it possible for advertisers to capture the attention of millions of people in this way. 110.The labor market of New York has recently shown signs of emerging from its longest slump since World War II. Part VII Writing (30 minutes, 30 points) Writing Task I (10 points) One possible version: Dear Director, I have to report the loss of my student identification card yesterday, Tuesday, September 3rd. I am a student in Business Studies. I believe I lost my card at the swimming pool when I dropped my backpack and many items fell to the ground. I need my card to get money from the bank. The bank will only accept my student card as ID. I asked the bank if they would accept my passport, but they said that my student ID card was used to open the account, and that I should bring it with me when I want to withdraw money. I also want to go to the cinema, but I will not get a concession without my student card. Could you please issue me a new card as soon as possible? I will come to your office this afternoon to see if I can get one immediately. Thank you very much. Yours sincerely, ### Writing Task II (20 points) One possible version: Dear Sir or Madam: I am currently a student at # # University. After reading your message in # # newspaper, I am very interested to apply for a teaching position with GEOS. English has become a world language, spoken and understood everywhere in the world. Therefore, to be an English teacher in foreign countries is to experience different cultures. Some of my friends, who are from the UK and are teaching English here, regard teaching English not only as a career but also a chance to learn about people and culture. They set good examples for me, and they have encouraged me to apply for a chance to join you. English is one of my favorite subjects. To be an English teacher always has been my dream. After four years of study in this university, I have passed the College English Test Band 6. Most of the courses I have taken are specially focused on teaching methods and skill training. If you are looking for an energetic, outgoing and motivated university graduate, I am sure you have found one. Sincerely yours, ### 2005年全国大学生英语竞赛初赛试题(A级,本科生) Part I Listening Comprehension (25 minutes, 30 points) Section A Dialogues (10 points) Directions:In this section, you will hear 10 short dialogues. At the end of each dialogue, a question will be asked about what was said. Both the dialogue and the question will be read only once. After each question, there will be a pause. During the pause, you must read the four choices marked A, B, C and D, and decide which is the best answer. Then mark the corresponding letter on the Answer Sheet with a single line through the centre. 1. A. The man got the wrong window. B. The woman's window will not be open until 3:00. C. The man didn't make a reservation. D. There's no postal order at presen. 2. A. On Monday. B. On Tuesday. C. On Wednesday. D. On Thursday. 3. A. Manager and staff member. B. Sales Manager and customer. C. Salesman and customer. D. Teacher and student. 4. A. Pleasant. B. Unhappy. C. Indifferent. D. Hesitating. 5. A. It's going to snow. B. It's going to rain. C. It's going to be windy. D. It will be sunny. 6. A. At 3:00. B. At 3:30. C. At 4:00. D. At 4:30. 7. A. They use up too much energy. B. They are not efficient. C. They are too expensive. D. She doesn't believe what it says on the box. 8. A. She is offended by the front cover. B. The magazine is too expensive. C. Her friend wants to cancel it. D. She has no time to read it. 9. A. Restaurant food. B. Home-cooked food. C. Takeaway food. D. Snack food. 10. A. They will go Dutch. B. The man will pay for the bill. C. The woman will pay for the bill. D. The woman will cook for them. Section B News Items (10 points) Directions:In this section, you will hear 10 short pieces of news from BBC or VOA. After each news item and question, there will be a pause. During the pause, you must read the three choices marked A, B and C, and decide which is the best answer. Then mark the corresponding letter on the Answer Sheet with a single line through the centre. 11. A. Russia and Germany. B. China and Japan. C. China and Russia. 12. A. Ukraine government. B. Members of Parliament from the opposition. C. President of Ukraine. 13. A. A library. B. A park. C. An underground museum. 14. A. $8 million. B. $18 million. C. $80 million. 15. A. In the USA. B. In Uganda. C. In the Sudan. 16. A. Because some bacteria are drug-resistant. B. Because the infection is untreatable. C. Because some bacteria can be spread by physical contact. 17. A. Washington D.C. B. Baghdad. C. Pennsylvania. 18. A. The Airbus A380 can offer more seats than the Boeing 747. B. The Airbus has to change the traffic movements a lot to enable more passengers to travel. C. The Airbus company is exercising the most influence in air travel now. 19. A. North Africa. B. South America. C. Hong Kong. 20. A. Oil prices got lower. B. Oil prices got higher. C. More than 24.5 million barrels will be produced every day. Section C Passages (10 points) Directions:In this section, you will hear 2 passages. At the end of each passage, you will hear 5 questions. After you hear a question, you must choose the best answer from the four choices marked A, B, C and D. Then mark the corresponding letter on the Answer Sheet with a single line through the centre. Passage One Questions 21 to 25 are based on the first passage you have just heard. 21. A. A lot of things remind them of their home country. B. They can have new experiences. C. They will make a lot of friends there. D. They already know a lot about the foreign country. 22. A. It makes people feel uncomfortable. B. It makes people become more open to new experiences. C. They will not visit the country again. D. They will go back to their own countries immediately. 23. A. When they first arrive in a new country. B. When they have been used to a new country. C. After a few weeks or months. D. After a friendly talk with a trained staff of the health center. 24. A. Having more sleep. B. Eating less. C. Trying to put on weight. D. Seeing a doctor. 25. A. They will feel pain all over. B. They may think about killing themselves. C. They will not go to school or work. D. No one can help these people. Passage Two Questions 26 to 30 are based on the second passage you have just heard. 26. A. People waste a lot of time watching it everyday. B. It encourages people to possess guns and knives through advertisements. C. It shows violence, which influences younger people especially. D. People will be willing to put up with violence. 27. A. The influence of passive viewing of TV on the development of Children's brain. B. The subject matter it shows to parents. C. The active way of living it promotes. D. The possible harm TV viewing has on the health of old people. 28. A. By allowing no time for the family to spend together. B. By preventing children from having contact with their parents. C. By encouraging children to depend on their parents. D. By taking over an essential part of the parents' work. 29. A. Recognize the educational merits of the medium. B. Ban TV advertising aimed at younger people. C. Prohibit teenagers from watching TV. D. Help older people survive the “television experience”. 30. A. It should be done because TV shows too much violence. B. It is neither necessary nor possible. C. It might not be workable, but we should not overlook the danger of TV. D. It should be done as soon as possible. Part II Vocabulary and Structure (5 minutes, 10 points) Directions: There are 10 incomplete sentences in this part. For each blank there are four choices marked A, B, C and D. Choose the one that best completes the sentence. Then mark the corresponding letter on the Answer Sheet with a single line through the centre. 31. Everyone knew that _________ the project would be completed and we'd all have to return to our own separate departments. A. primarily B. ultimately D. precisely 32. The planes were delayed and the hotel was awful, but _________ we still had a good time. A. on the contrary B. by the same token C. on top of all that D. for all that 33. It's freezing outside; you'd better _________ black ice when you drive home. A. look down on B. check up on C. watch out for D. keep out 34. At that time no one's even studied the problem, _________ tried to do anything about it. A. not to say B. far more C. in no way D. much less 35. It was odd that you _________ for Margie's address. I just got a letter from her—the first one since her family moved on August 1 1983. A. should have asked B. would have asked C. shall ask D. might ask 36. The women ran down to assist them in bringing up their seals; but no sooner _________ the house than a voice was heard in the passage, and a man entered. A. after they re-enter B. had they re-entered C. were they to re-enter D. when they had re-entered 37. Several of the administrative participants recognized the support and leadership from their institution's chief executive officers as a serious motivating factor at those moments _________. A. if the problem was challenging B. as the situation became serious C. when the challenges seemed overwhelming D. which the counterpart grew manipulating 38. Well, really _________ now is start learning what to do with this software and read a lot of tutorials, learn and practice _________ you can. A. what you should do; as much as B. that you need do; whatever C. how you should start; the most D. as should you do; the longest hours 39. _________, and members of the congregation having returned, this church too will start out with _________ effort in the work that lies before them, which will be taken up by the different departments. A. The summer vacation is over; great B. The summer vacation being over; renewed C. As the summer vacation over; refreshing D. Had the summer vacation been over; sacred 40. Although there is some truth to the fact that Linux is a huge threat to Microsoft, predictions of the Redmond company's demise are, _________, premature. A. saying the most B. to say the most C. to say the least D. to the least degree Part III Situational Dialogues (5 minutes, 10 points) Directions: There are 10 incomplete dialogues in this part. For each blank there are four choices marked A, B, C and D. Choose the one that best completes the dialogue. Then mark the corresponding letter on the Answer Sheet with a single line through the centre. 41. Simon: I'm a reporter from the Hendon Standard.______________ Celia: Yes, I was standing at the end of Elm Avenue, by the park. Simon: What happened? Celia: There was a red van traveling west, and several cars and vans behind it. A. Were you present when the accident happened? B. Would you tell me what you have seen? C. What a surprise to see you here! D. Did you do anything special just now? 42. Stan: Thank you from the bottom of my heart for saving my little girl's life. John: ______________ Stan: I can't tell you how much I appreciate what you've done. John: I'm just happy I could help. A. There's nothing to be afraid of. B. This is a wonderful day. C. Anybody would have done the same. D. I am glad to save her. 43. Harry: I hear you are going to get married soon. Ron: That's right, next June. June 21st. Can you come to the wedding? Harry: Oh! ______________ That's when we're away on vacation. Ron: Never mind, we'll send you some wedding cake. A. That's great! B. What a pity~ C. You have my deepest sympathy. D. You certainly can. 44. Mother: You're watching too much TV, Emily. Emily: Oh, come on, Mom. Mother: ______________ And you're eating too many snacks. Emily: I only had some popcorn and some potato chips. A. I mean it. B. You're excused. C. Where's your radio? D. I don't want to know it. 45. Karen: I'm not optimistic about finding a job after I finish college. Joseph: Oh? Why not? Karen: The economy is going downhill-fast. Joseph: I know. ______________ It's getting to the point where even a degree won't help you anymore. Karen: That's right. And I'll be lucky to even move out of my parents' house. A. My friend could offer me a job. B. I am not sure whether I agree or not. C. How do you feel about the economy? D. What is this world coming to? 46. Ivan: I brought you the new Groove People CD. Dale: What good is a CD if I haven't got a CD player? Ivan: I can bring you a CD player. Dale: What good is a CD player if I don't even have electricity? Ivan: ______________ Dale: What good is calling the Electric Company if I haven't got a house? A. Why do you have so many questions? B. What are you going to buy tomorrow? C. Why don't you call the Electric Company? D. Where did you buy the new CD? 47. Doreen: We've been waiting for an hour and a half. Trevor: Say your aunt is very ill. A doctor ought to see her at once. Doreen: There isn't a doctor available. They're all busy. Trevor: ______________ Doreen: I've asked her over and over again. The more I ask, the longer I wait. A. You're starting to calm down now. B. Ask the receptionist to hurry up. C. There will be a difficult period for your aunt. D. I'm sorry to hear that. 48. Bruce: Good morning. Welcome to Supervacation Travel Agency. Can I help you? Mandy: Yes, I hope so. I'm interested in a short holiday soon. I'd like some information. Bruce: Yes, certainly. ______________ Mandy: Somewhere with some sunshine. Bruce: Here's a Supervacation brochure. It gives information about lots of holidays. Mandy: Thanks for the information. I expect I'll see you soon. A. What about New York? B. Will you be traveling alone? C. What sort of holiday interests you? D. How are things going with you? 49. Mavis: Did you see how close that finish was? It took my breath away! Roger: Wait, they're announcing the winner now. It's Don Jensen! Mavis: What a surprise! He's never won a major race before. Roger: I'm just astounded to see how much he's improved. Mavis: ______________ Roger: I'm just floored that my best friend won such an important race. A. Why can't he improve quickly? B. What happened to him in the past two years? C. I'm going to take a break. I'm so tired! D. Let's go down on the track and congratulate him. 50. Careth: Good afternoon. I've just joined the library. How many books can I take out? Eunice: You can take two books, and keep them for ten days. If you haven't finished, you can renew them. Careth: How do I do that? ______________ Eunice: No, you can telephone. Tell us the titles of the books, and the date they are due for return. Careth: Splendid. I would be A Tale of Two Cities, or David Copperfield. Eunice: These two books have been lended. A. Why can't I keep them for a longer time? B. Must I visit the library? C. How much could I be fined? D. Can I lend them to my friends? Part IV IQ Test (5 minutes, 5 points) Directions: There are 5 IQ Test questions in this part. For each question there are 4 choices marked A, B, C and D. Mark your answer on the Answer Sheet with a single line through the centre. 51. A postman delivers mail round a housing estate. He does not want to visit the same street more than once, but can pass over the same street corners. On which housing estate is this possible? 52. Here is a riddle. I am suspended in the air, I touch nothing, and I am surrounded by lights. Now I dress myself afresh, and now I am naked, and I am in the heat and the cold, by night and by day. Everyone amuses himself by tramping upon me, even the animals abuse and scorn me, and yet I have such treasures hidden in my bosom that he who finds them I can make full of happiness. What is the answer to the riddle? A. The Sun. B. The Moon. C. The Earth. D. The Sky. 53. Which of the following numbers should replace the question mark? A. 417. B. 926. C. 138. D. 172. 54. This bar chart shows the heights of a class of pupils Which statement must be true? A. 2 children are 125 cm tall or less. B. 8 children are at least 140 cm tall, but less than 145 cm tall. C. 8 children are more than 144 cm tall, but less than 150 cm tall. D. No children are taller than 158 cm. 55. Can you guide the robot along the white squares through this grid? It starts on the square marked „Begin? and finishes on the square marked „End?. You can only programme it to move FORWARD, TURN LEFT 90? or TURN RIGHT 90?. Which of the instructions below will guide the robot through the grid? A. FORWARD 4, TURN RIGHT 90?, FORWARD 4, TURN LEFT 90?, FORWARD 4, TURN RIGHT 90?, FORWARD 2. B. FORWARD 4, TURN RIGHT 90?, FORWARD 3, TURN RIGHT 90?, FORWARD 4, TURN RIGHT 90?, FORWARD 3. C. FORWARD 3, TURN RIGHT 90?, FORWARD 3, TURN RIGHT 90?, FORWARD 4, TURN RIGHT 90?, FORWARD 2. D. FORWARD 3, TURN RIGHT 90?, FORWARD 3, TURN LEFT 90?, FORWARD 4, TURN RIGHT 90?, FORWARD 2. Part V Reading Comprehension (25 minutes, 35 points) Section A Multiple Choice (5 points) Directions: There is 1 passage in this section with 5 questions. For each of them, there are four choices marked A, B, C and D. You should decide on the best choice. Then mark the corresponding letter on the Answer Sheet with a single line through the centre. Questions 56 to 60 are based on the following passage. New Findings in Archaeology ?South America and Madagascar were joined 70 million years ago Picture: Reuters An old croc: The discovery in Brazil of a nearly complete fossilised skeleton of the prehistoric crocodile Ubera-basuchus terrificus, above, supports theories that South America and Madagascar were joined 70 million years ago. Picture: Reuters An old croc: The discovery in Brazil of a nearly complete fossilised skeleton of the prehistoric crocodile Ubera-basuchus terrificus, above, supports theories that South America and Madagascar were joined 70 million years ago. ?We've all just got 40,000 years older Two skulls found in Africa have been identified as the oldest human remains known to science. The fossilised bones found in 1967 were originally thought to be 130,000 years old. But a re-dating of rock layers near those which yielded the fossils showed they are actually about 195,000 years old—from the time modern humans emerged. The skulls, known as Omo I and II, push back the known presence of Homo sapiens in Africa by 40,000 years. The previous oldest Homo sapiens skulls, dated to between 154,000 and 160,000 years old, were found near a village called Herto in the afar region of eastern Ethiopia. Omo I and II were unearthed by famous palaeontologist Richard Leakey along the Omo River in southern Ethiopia, near the town of Kibish. Omo I's more modern features led to disagreement among experts over whether they were the same age. The rocks in which they were found show they are, said Australian archaeologist Prof Ian McDougall, who made the discovery. He told scientific journal “Nature”: “Omo I and Omo II are relatively securely dated to 195,000 years old, making them the oldest anatomically modern human fossils yet recovered.” Questions: 56. By which means could the scientists tell the age of fossils they have found? A. The bones of the fossils. B. The place where they were found. C. The history of humans. D. Identifying the rock layers of the fossils. 57. How many years older has our species become by the new discovery? A. 130,000. B. 40,000. C. 195,000. D. Between 154,000 and 160,000. 58. Where were the Omo I and II found? A. Near Herto. B. Along the Omo River. C. Near Kibish. D. In Australia. 59. What does the word “ homo sapiens” mean in this passage? A. Human being. B. A sample of fossils. C. A river. D. A country. 60. Which of the following statements are NOT true according to the passage? A. The discovery of an old crocodile skeleton in Brazil makes people believe that South America and Madagascar were joined 70 million years ago. B. Prof Ian McDougall thought Omo I and II were the oldest human fossils discovered. C. Prof Ian McDougall discovered that Omo I and II were not the same age. D. Omo I and II were discovered in southern Ethiopia. Section B Short Answer Questions (20 points) Directions: In this section, there are 2 passages followed by 10 questions or unfinished statements. Read the passages carefully, then answer the questions in the fewest possible words (not exceeding 10 words). Remember to write the answers on the Answer Sheet. Questions 61 to 63 are based on the following passage. King James Before the last shot of Michael Jordan's final game with the Chicago Bulls, he left behind a moment that will hang in the minds of sports fans the way he did in the air. His quick crossover dribble, with the help of a tiny shove, sent Utah Jazz defender Byron Russell to the floor. Wide open, Jordan nailed the game-winning jump shot, and he and the Bulls clinched their sixth—and Jordan's last—NBA title. At the Rose Garden Arena in Portland, Ore., last Wednesday, Cleveland Cavaliers star LeBron James, 20, became the youngest player to reach one of the toughest performance milestones in basketball, the “triple double” (double digits in points, rebounds and assists in a single game—he did it again three nights later). During one play, his quick crossover dribble, sans shove, sent a Portland defender veering to the sideline. Wide open, James shot a three pointer. Swish. Nothing but Mike. Ever since Jordan's first retirement a dozen years ago, the NBA has searched from Chi-town to China for a star as dominant and marketable as His Airness. The race to replace Jordan may end without a winner, but LeBron James, in just his second year out of St. Vincent—St. Mary High School in Akron, Ohio, is already far ahead of the pretenders. His game is spectacular. James combines Jordan's ease at filling up the box score (and the stands) with a ruthless instinct to win, a trait that Hill, a gentleman, and Carter, a diffident performer, surely lack. And like Jordan, James knows he can't do it alone. In fact, he is a complete team player who loves to pass the ball and make his teammates better: Iverson and Bryant can't claim much there. The biggest shock of all: the most hyped teen athlete of this century, who single-handedly put high school basketball on national television, has actually exceeded the ungodly expectations set before him. “The King James era is here”, says Milwaukee Bucks head coach Terry Porter, who played against Jordan for much of his 17-year pro career. “You could argue that LeBron is kind of carrying the league right now.” Questions: 61. What does the sentence “Nothing but Mike.” indicate? 62. Hill, a gentleman, and Carter, a diffident performer lack _________, compared with Jordan. 63. Can you guess what Terry Porter was before he was a coach? Questions 64 to 70 are based on the following passage. Good news: Olympic chiefs visit London. Bad news: they'll be traveling by Tube It's crunch week for London's bid to host the 2012 Games BY ANDREW JOHNSON and JONATHAN THOMPSON Senior government ministers and British sporting heroes will be on hand to extol the capital's virtues when the 12-member International Olympic Committee (IOC) team visits this week to assess the London 2012 bid. Although much of their four-day visit is expected to be spent in the exclusive Four Seasons Hotel in Canary Wharf grilling bid organisers in question and answer sessions, the evaluation commission will spend one day visiting London's proposed venues—and will take a trip on the Tube. It could prove a costly journey. London 2012 organisers, who have spent up to ,20m on the project, have pencilled in trips to the proposed sites, including the planned stadium at Stratford, the Millennium Dome at Greenwich and the ExCel centre in Docklands. However, the commission can insist on being taken anywhere, including the new Wembley stadium, which will host the football, or to the archery venue at Lord's, or even to Wimbledon or Hackney. Earlier this month, the Madrid bid suffered a setback over lack of accommodations, which was criticised by IOC members. Will transport prove Londons' Achilles' heel? On Friday The Independent on Sunday sent four reporters on various journeys across London to put the transport infrastructure to the test. Questions: 64. How many people can the London's planned Olympic stadium hold? 65. How many International Olympic Committee(IOC) members will visit London and where are they going to have the question and answer sessions? 66. What does “tube” mean in American English? 67. London 2012 organisers have spent up to ,20m on the project. The proposed sites include _________ at Stratford, _________ at Greenwich and _________ in Docklands. 68. Why was Madrid criticised by International Olympic Committee members? 69. The Independent on Sunday sent four reporters on four journeys across London. The four journeys are from Canary Wharf to the following destinations: ? , ? , ? , ? . 70. What is the problem which the authors worried about London's bid to host the 2012 Olympic Games? Section C True (T) or False (F) (10 points) Directions: In this section, there is 1 passage with 10 statements. Read the passage and decide which of the statements at the end of the passage are true and which are false. Remember to write the answers on the Answer Sheet. Questions 71 to 80 are based on the following passage. Statements: 71. The Oxford University Press are looking for some enthusiastic and hard-working higher education sales representatives to fill UK field sales positions based in South West, South London & West Midlands. 72. The Higher Education Sales Representatives are required to manage their territories in a way that maximises sales growth. 73. As one of the higher education sales representatives, he will get competitive salary plus car and bonus scheme and he will not work at home. 74. Excellent organizational, presentation, and proven people management skills are essential to be a Sales Support Co-ordinator. 75. HTML skills are essential to be a Sales Support Co-ordinator currently. 76. You have to work in Oxford if you are the Sales Support Manager or the Sales Support Co-ordinator. 77. You can contact with either Sue Smith or Chris Tatton if you are interested in the post of Marketing Systems Executive. 78. The closing date for all the mentioned posts is 4 October 2004. 79. You can contact the Oxford University Press by post or email if you want to apply the posts. 80. You can get more information about all the posts from oup.com.uk/recruit. Part VI Cloze-Test (10 minutes, 10 points) Directions: There are 10 blanks in the passage. For each blank, the first letter of the word has been given. Read the passage below and think of the word which best fits each blank. Use only one word in each blank. Remember to write the answer on the Answer Sheet. Farmers, as we all know, have been having a hard time of it in Britain, and have been turning to new ways of earning income from their land. This involves not only p 81 new kinds of crops, but some strange ways of making money, the most u 82 of which has got to be sheep racing. Yes, you heard me correctly! A farmer in the West of England now holds sheep races on a regular basis, and d 83 the past year over 100,000 people have turned up to w 84 the proceedings. “I was passing the farm on my way to the sea for a holiday,”one punter told me, “and I thought I'd have a look. I didn't believe it was serious, to tell you the truth.” A 85 to a regular visitor, betting on sheep is more interesting than betting on horses. “At proper horse races everyone has already studied the form of the horses in advance, and there are clear f 86 . But nobody has heard anything about these sheep! Most people find it d 87 to tell one from another in any case. ”I stayed to watch the races, and I must admit that I found it quite exciting. In a typical race, half a dozen sheep race downhill o 88 a course of about half a mile. Food is waiting for them at the other end of the track, I ought to add! The sheep run s 89 fast, although presumably they haven't eaten for a while just to give them some m 90 . At any rate, the crowd around me were obviously enjoying their day out at the races, judging by their happy faces and the sense of excitement. Part VII Translation (15 minutes, 20 points) Section A English-Chinese Translation (10 points) Directions: Translate the underlined sentences of the following passage into Chinese. Remember to write the answers on the Answer Sheet. The symbolic importance of the burger cannot be underestimated. (91) Under its beefburger guise, it was the first of the new range of “convenience” foods which were about to make the world a better place and begin the liberation of women from the drudgery of home-cooking and housework. The older generation did not approve, which made it all the better. In the Sixties the hamburger was a symbol of the techno age — perfectly circular and streamlined. It was as uniform and relentlessly predictable as only the latest technology could make it. (92) True, there were those who rebelled against it, but to most the hamburger was a reflection of the national love affair with Americana(美国文化). It was a phenomenon which was made fresh in Seventies London with the trendy burgers of the Great American Disaster and the Hard Rock Café, and in many other cities round the world. In the Eighties another subtle shift occurred. People became aware that America was no longer another place but a culture which had spread throughout the world. And the hamburger became globalized, too, in the form of McDonald?s. With its US home market, like the fat in its burgers, heavily saturated, McDonald?s looked abroad. (93) By the end of the Eighties it had grown to such a size that every day 28 million global citizens ate there and the Big Mac became omnipresent. (94) McDonald?s stormed the world, but its successes also drew upon it in the Nineties the criticisms which were levelled at that era. (95) Food experts began to see the world?s changing culinary (烹调的) tastes as a symbol of what is wrong with the new consumerism. “The hamburger is a metaphor for our times — cheap, convenient and an indication that we have given up any real interest in what we eat,” said the leading food writer Frances Bissell, lamenting the trends of our increasingly obese society towards snacking on the hoof or before the TV instead of eating proper meals. Then along came “Mad Cow Disease” and even though the average person was told they had more chance of winning the National Lottery than contracting “Mad Person Disease”, with it came the dreadful realization that the cheap, convenient, easy way out might, in the end, turn out to be none of these things. Section B Chinese-English Translation (10 points) Directions: Translate the following sentences into English. Remember to write the answers on the Answer Sheet. 96. 文化是一切。文化是我们着装的方式,是我们走路的方式,是我们系领带的方式。 97. 健康专家警告,近年来由于过量饮食而引起的疾病的发生率在不断攀升,特别是在节假日期间, 人们往往吃喝很多,却很少锻炼。 98. 时间是个多才多艺的 关于同志近三年现实表现材料材料类招标技术评分表图表与交易pdf视力表打印pdf用图表说话 pdf 演者。它能展翅飞翔,能阔步前进,也能治愈创伤。 99. 如果你有天赋,勤勉会使其更加完善;如果你能力平平,勤勉会补足缺陷。 100. 电影《音乐之声》上映40年后,新编舞台剧《音乐之声》将首次在奥地利首都上演。 Part VIII Writing (30 minutes, 30 points) Task I (10 points) Directions: Here is an advertisement about the University of Warwick. Suppose you are going to apply one of the MA programmes. You should write a letter to the secretary to inquire something about the MA programme, tuition fee and accommodation. Your letter should be about 120 words. Now write the letter on the Answer Sheet. Task II (20 points) Directions: You should write a composition on the topic Distance Learning Programme—the Best Way? You should write about 150 words, and base your composition on the outline (given in Chinese) below. Now write the composition on the Answer Sheet 参考答案 Part I Listening Comprehension (25 minutes, 30 points) Section A Dialogues (10 points) 1.A 2. D 3. C 4. B 5. C 6. D 7. C 8. A 9. B 10. C Section B News Items (10 points) 11. C 12. B 13. A 14. B 15. C 16. A 17. C 18. A 19. B 20. B Section C Passages (10 points) Passage One 21. B 22. A 23. C 24. D 25. B Passage Two 26. C 27. A 28. D 29. B 30. C Part II Vocabulary and Structure (5 minutes, 10 points) 31. B 32. D 33. C 34. D 35. A 36. B 37. C 38. A 39. B 40. C Part III Situational Dialogues (5 minutes, 10 points) 41. A 42. C 43. , 44. A 45. D 46. C 47. B 48. C 49. D 50. B Part IV IQ Test (5 minutes, 5 points) 51. D 52. C 53. D 54. B 55. D Part V Reading Comprehension (25 minutes, 35 points) Section A Multiple Choice (5 points) 56. D 57. B 58. C 59. A 60. C Section B Short Answer Questions (20 points) 61. It indicates that fans thought that James almost reached Jordan. 62. a ruthless instinct to win 63. A basketball player. 64. 80,000. 65. 12, Four Seasons Hotel in Canary Wharf. 66. Subway. 67. the planned stadium, the Millennium Dome, the ExCel centre 68. Lack of accommodation. 69. ? Wembley Stadium ? Lord's Cricket Ground ? Wimbledon ? Hackney Wick 70. The transport infrastructure. Section C True (T) or False (F) (10 points) 71.F 72. T 73. F 74. F 75. F 76. T 77. F 78.T 79. T 80. F Part VI Cloze-Test (10 minutes, 10 points) 81. planting 82. unusual 83. during 84. watch 85. According 86. favourites 87. difficult 88. over 89. surprisingly 90. motivation Part VII Translation (15 minutes, 20 points) Section A English-Chinese Translation (10 points) 91(牛肉馅饼便是接着问世的一系列新的“便利”食品的第一个产品。“便利”食品使世界变 得更加美好,也使妇女从单调乏味的家庭烹饪和家务事中解放出来。 92(的确,有些人反对它,可对更多的人来说,汉堡包却是全民族迷恋美国文化的反映。 93(到80年代末,麦当劳已拥有相当大的规模,每天全球就有2800万人在麦当劳就餐, 巨无霸汉堡包变得无处不在。 94. 麦当劳已风靡全球,可到了90年代,它成功的业绩也招致了针对那个时代的种种批评。 95(食品专家开始注意到,世界上正在改变的烹调口味意味着新的消费模式出了什么问题。 Section B Chinese-English Translation (10 points) 96(Culture is everything. Culture is the way we dress, the way we walk, the way we tie our ties. 97(Health experts have warned that the incidence of diseases related to eating and drinking too much has risen in recent years, especially during holiday seasons when people tend to eat and drink more but take less exercise. 98(Time is a versatile performer. It flies, marches on, and heals all wounds. 99(If you have great talents, industry will improve them; if you have but moderate abilities, industry will supply their deficiency. 100(The stage production of The Sound of Music is to be performed for the first time in the Austrian capital, 40 years after the film was released. 2006年全国大学生英语竞赛初赛试题(C类) Part I Listening Comprehension (25 minutes, 30 points) Section A Short Conversations (6 points) 1. A. Keys and a purse. B. Glasses and keys. C. Glasses and a bag. D. Glasses, keys and a purse. 2. A. A plate. B. A table. C. A lamp. D. A phone. 3. A. They are under the chair by the television. B. They are under the chair with the rubbish. C. They are beside the television. D. They are by the door with the rubbish. 4. A. Grapes and oranges. B. Grapes and apples. C. Bananas and grapes. D. Bananas and oranges. 5. A. Five to three. B. Five past three. C. Twenty five to three. D. Twenty five past three. 6. A. The restaurant. B. The market. C. The cinema. D. The sports center. Section B Long Conversation (4 points ) Section C News Items (10 points) Directions: In this section, you will hear 10 short pieces of news from BBC or VOA. After each news item and question, there will be a pause. During the pause, you must read the three choices marked A, B and C, and decide which is the best answer. Then mark the corresponding letter on the Answer Sheet with a single line through the centre. 11. A. 6. B. 16. C. 60. 12. A. To bring more foreign language speakers into government service. B. To promote trade with foreign countries. C. To make people learn foreign languages at an early age. 13. A. An hour. B. More than an hour. C. Less than an hour. 14. A. To get publicity for their cause. B. To get some money for their group. C. To get more rights for their group. 15. A. Ticket prices have been falling and their incomes rising. B. Aviation fuel is becoming less expensive. C. British people prefer to travel by plane. 16. A. More than eighty thousand. B. More than sixty-two thousand. C. More than fifty-three thousand. 17. A. 30. B. 27. C. 57. 18. A. Baton Rouge. B. Louisiana. C. Atlanta. 19. A. Reduce violations of intellectual property rights. B. Control the growing population. C. Working together to fight terrorism. 20. A. Workers and policemen. B. Farmers and the unemployed. C. Workers and farmers. Section D Passages (10 points) Directions: In this section, you will hear 2 passages. At the end of each passage, you will hear 4 or 6 questions. After you hear a question, you must choose the best answer from the four choices marked A, B, C and D. Then mark the corresponding letter on the Answer Sheet with a single line through the centre. Passage One Questions 21 to 24 are based on the passage you have just heard. 21. A. He is very thin. B. He is very tall. C. He is very short. D. He is very fat. 22. A. He met a cook from a restaurant. B. He met a man who had a farm. C. He met two fat farmers. D. He met a farmer and his wife. 23. A. He wanted to have something to eat for lunch. B. He wanted to go to a restaurant for dinner. C. He wanted to make a cup of coffee. D. He wanted to do some farm work. 24. A. He wanted some green beans. B. He wanted a big steak. C. He wanted some cake and coffee. D. He wanted lots of potatoes. Passage Two Questions 25 to 30 are based on the passage you have just heard. 25. A. Last week. B. Three weeks ago. C. Two months ago. D. Three years ago. 26. A. By coach. B. By bus. C. By car. D. By train. 27. A. 9 am to 9 pm. B. 10 am to 8 pm. C. 10 am to 9 pm. D. 10 am to 10 pm. 28. A. Get information. B. Watch a film. C. Find a bank. D. Buy some shoes. 29. A. Feed the ducks. B. Take a bus ride round the lake. C. Go swimming. D. Go sailing. 30. A. There was nowhere to put the rubbish. B. There were not enough cleaners. C. The food in the café was disappointing. D. The service in the shops was slow. Part II Vocabulary and Structure (5 minutes, l0 points) Directions: There are I0 incomplete sentences in this part. For each blank there are four choices marked A, B, C and D. Choose the one that best completes the sentence. Then mark the corresponding letter on the Answer Sheet with a single line through the centre. 31. I can't agree with my Mum. I think that such an old-fashioned dress can't cost a ________. She says 100 pounds is a real ________. A. lot of money; luck B. bargain; luck C. fortune; bargain D. big sum; fortune 32. ________ is on the rise, with over 20% of serious crimes being committed by children under the age of seventeen. A. Junior crime B. Juvenile delinquency C. Minor crime D. Senior delinquency 33. The Smiths were leaving that ________ town. Everybody wanted to escape its noise and pollution and was looking forward to a ________ country life. A. crowded; peaceful B. deserted; peaceful C. desert; thrilling D. empty; sour 34. When I first began writing poetry, I think the poets that I had studied at school ________ my approach and the things I wrote about. A. communicated B. impressed C. influenced D. discussed 35. She is an excellent teacher who has taught in four schools. ________she taught, her principals had a high opinion of her. A. Wherever B. Everywhere C. Inasmuch as D. In such schools that 36. My friend Tanya ________ Japanese for six years before she ________ Japan. I've just received a letter from her. It says she has been studying Chinese for three months and ________ for China in a month. A. had been studying; visited; is leaving B. studied; had visited; will live in C. has studied; visited; would leave D. was studying; has visited; leaves 37. At the last place Gary worked, they ________ an annual company picnic. All the employees ________ bring their families along and spend the day at a nearby park. It was great. A. had to have; had to B. used to have; couldn't C. would have; didn't have to D. used to have; would 38. They ________ such a big van ________ the price of gas would skyrocket. A. would have bought; if they knew B. wouldn't have bought; had they known C. wouldn't have bought; if had they known D. wouldn't have bought; did they know 39. We’re going to paint the town ________ to celebrate our win. A. blue B. purple C. gold D. red 40. Written in central Canada in the early part of the twentieth century, ________ depicts life in Manitoda. A. The Midnight Sun was Victor Frank's last novel, B. Victor Frank's last novel was The Midnight Sun, C. The Midnight Sun, which was Victor Frank's last novel, D. Which was Victor Frank's last novel, The Midnight Sun, Part III Situational Dialogues (5 minutes, 5 points) Directions: There are 5 incomplete dialogues in this part. For each blank there are four choices marked A, B, C and D. Choose the one that best completes the dialogue. Then mark the corresponding letter on the Answer Sheet with a single line through the centre. 41. Dan: Have you ever participated in a risky sport? Kay: Yes, I like hang gliding. It's fantastic to be able to fly like a bird. Though landing is sometimes difficult, I've always felt hang gliding is quite safe. Dan: __________________________________________________ Kay: I've never been seriously injured. Maybe I've just been lucky. Once, my glider turned upside down, and I lost control. I almost crashed, but I parachuted away just in time. A. What sports are risky? B. Do you ever get into some difficult situations? C. Have you ever hurt yourself in an accident? D. Have you ever been to a sports centre? 42. Lucy: What made you leave such a large company? Ken: My work there was so boring. I couldn't do anything myself. I always had to have my boss’ approval. So I decided to get a new job at Coricom, a small venture company. Lucy: __________________________________________________ Ken: The work here is very challenging, which suits me. We always have to cope with dynamic working conditions. And, since there are not many people in this company, we understand each other very well and feel like we are all in the same family. A. What's the hardest part of your new job? B. How do you like your new job? C. Tell me about what you liked at the large company. D. Tell me about what you liked in your university. 43. Julio: __________________________________________________ Officer: Well, first, you write and get an application form. Then, you send it in with a copy of your school records. And after that, you ask your teachers for some letters of recommendation. Julio: Are foreign students allowed to work in the States? Officer: They'll only let you work in the summer. And you'll need to get permission from the U.S. Officer of Immigration to do that. During the school year you're not allowed to work unless the work experience is part of your school program. A. Is it all right to apply to several universities at the same time? B. I'd like to get some information on how to get into an American university. C. When can I apply for that? D. I'd like to get some information on how to get a travel card. 44. Bob: __________________________________________________ Jane: My first book was Trapped in a Cave, a true story about two boys who got trapped in an underground cave for five days without food, water or light. Next I wrote the current twelve volumes of Real Kids, Real Adventures. Right now I'm working on the next two Real Kids, Real Adventures books. Bob: __________________________________________________ Jane: If I'm looking for a specific kind of story—for instance a child who survived being struck by lightning—I'll go to the library and use Newsbank, keying in on words like "lightning" and "child". Mostly, though, I get tips from kids who read the Real Kids, Real Adventures books and send notes or newspaper clippings. A. Can you tell me about the books you've written so far? ; Where do you get your stories? B. What are your favorite books that you've read? ; Where do you get your stories? C. Can you tell me about the books you've written so far? ; What's the hardest part about being a writer? D. Where do you get your stories? ; What's the hardest part about being a writer? 45. Lisa: __________________________________________________ Andy: I think people love to laugh. They want to laugh even in serious business presentations, in the classroom, seminar, and so on. When people laugh, they relax. And they can remember you and your message better. Lisa: __________________________________________________ Andy: Most people give a summary at the end of their speech. But, in my opinion, a summary at the end only distracts from a good presentation. I want to give people a chance to think about the topic, so I finish my speech with some questions. A. Could you tell me how to introduce speakers? ; How do you end your speech? B. Could you tell me how to introduce speakers? ; Do you think the title of a speech is important? C. How do you end your speech? ; Do you think the title of a speech is important? D. Why do you use so many jokes in your speech? ; How do you end your speech? Part IV IQ Test (5 minutes, 5 points) Directions: There are 5 IQ Test questions in this part. For each question there are 4 choices marked A, B, C and D. Mark your answer on the Answer Sheet with a single line through the centre. 46. What is the minimum number of matches you can remove from this diagram to leave just 2 squares? A. 2. B. 4. C. 6. D. 8. 47. Which of the bottom watches completes the sequence? 48. Which of these cubes cannot be formed from this web? 49. How many circles contain a dot? A. 12. B. 11. C. 10. D. 13. 50. Each symbol in this table has a value. The total of these values in each row and column is written at the end of the corresponding row or column. Can you find the value of each symbol? A. Triangle = 6.5, Square - 4.2, Diamond = 5.8, Circle - 11.4 B. Triangle = 7.5, Square - 5.2, Diamond -- 6.8, Circle = 12.4 C. Triangle - 8.5, Square - 5.2, Diamond - 6.8, Circle - 13.4 D. Triangle - 5.5, Square - 3.2, Diamond - 4.8, Circle = 10.4 Part V Reading Comprehension (25 minutes, 40 points) Section A Multiple Choice (5 points) Directions: There is one passage in this section with 5 questions. For each question, there are four choices marked A, B, C and D. You should decide on the best choice. Then mark the corresponding letter on the Answer Sheet with a single line through the centre. Questions 51-55 are based on the following passage. I was dirty, smelly, hungry and somewhere be Questions 51-55 are based on the following passage. I was dirty, smelly, hungry and somewhere beneath all that, suntanned. It was the end of an Inter-Rail holiday. My body couldn't take any more punishment. My mind couldn't deal with any more foreign timetables, currencies or languages. “Never again,” I said, as I stepped onto home ground. I said exactly the same thing the following year. And the next. All I had to do was buy one train ticket and, because I was under twenty-five years old, I could spend a whole month going anywhere I wanted in Europe. Ordinary beds are never the same once you’ve learnt to sleep in the corridor of a train, the rhythm rocking you into a deep sleep. Carrying all your possessions on your back in a rucksack makes you have a very basic approach to travel, and encourages incredible wastefulness that can lead to burning socks that have become too anti-social, and getting rid of books when finished. On the other hand, this way of looking at life is entirely in the spirit of Inter-Rail, for common sense and reasoning can be thrown out of the window along with the paperback book and the socks. All it takes to achieve this carefree attitude is one of those tickets in your hand. Any system that enables young people to travel through countries at a rate of more than one a day must be pretty special. On that first trip, my friends and I were at first unaware of the possibilities of this type of train ticket, thinking it was just an inexpensive way of getting to and from our chosen camp-site in southern France. But the idea of non-stop travel proved too tempting, for there was always just one more country over the border, always that little bit further to go. And what did the extra miles cost us? Nothing. We were not completely uninterested in culture. But this was a first holiday without parents, as it was for most other Inter-Railers, and in organizing our own timetable we left out everything except the most immediately available sights. This was the chance to escape the guided tour, an opportunity to do something different. I took great pride in the fact that, in many places, all I could be bothered to see was the view from the station. We were just there to get by, and to have a good time doing so. In this we were no different from most of the other Inter-Railers with whom we shared corridor floors, food and water, money and music. The excitement of travel comes from the sudden reality of somewhere that was previously just a name. It is as if the city in which you arrive never actually existed until the train pulls in at the station and you are able to see it with your own tired eyes for the first time. Questions: 51. At the end of his first trip, the writer said “Never again” because ________. A. he felt ill B. he disliked trains C. he was tired from the journey D. he had lost money 52. What does the writer mean by “this way of looking at life” in Paragraph 3? A. Worrying about your clothes. B. Throwing unwanted things away. C. Behaving in an anti-social way. D. Looking after your possessions. 53. Why did the writer originally buy an Inter-Rail ticket? A. To get to one place cheaply. B. To meet other young people. C. To see a lot of famous places. D. To go on a tour of Europe. 54. What the writer liked about traveling without his parents was that ________. A. he could see more interesting places B. he could spend more time sightseeing C. he could stay away from home longer D. he could make his own decisions 55. What does "it" in Line 3, Paragraph 6, refer to? A. A name. B. The city. C. The train. D. The station. Section B Yes / No / Not given (5 points) Directions: In this part, you will have 5 minutes to go over the passage quickly and answer the questions on the Answer Sheet. For questions 56 - 60, mark Y (for Yes) if the statement agrees with the information given in the passage; N (for No) if the statement contradicts the information given in the passage; NG (for Not Given) if the information is not given in the passage. Questions 56-60 are based on the following passage. The Outdoor Centre Opening times Water sports: 10 am - 6 pm Play Park: 10 am - 5.30 pm Entrance / Car park fees Low season: Weekdays ,2.00 per car Weekends ,3.00 per car High season: 23 July - 11 September Weekdays and weekends ,3.00 per car Fees are for cars with four people. Each extra person is 50p. Fees to be paid at the main office. The center is not a private club; it is an organization whose aim is to provide outdoor sports and recreation facilities for the public. Group visitors are requested to inform the center in advance of their intended visit. Windsurfing - One-day course Beginner windsurfing course is offered on Saturdays and Sundays when the weather is good enough. Learning to windsurf is a lot of fun. The excitement when you sail across the water for the first time is not easily forgotten. Boards with small sails are available for beginners. Course fee: ,32.50 (this includes all equipment) One-day adventure course This is an opportunity you have been waiting for. Come and try sailing, climbing, surfing and archery. This course is intended to introduce outdoor activities to adults in a fun, leisurely manner. You do not need to be extremely fit or to have had previous experience of the activities. All you need is to be interested. 22.50 Course fee: , Play Park The Play Park is suitable for children from two to ten years of age. It is one of the best of its type in the country. It has sand and water play, slides, large ball pool, play castle and much, much more. Next year the center will open a new Play Palace and Play Ship. Summer adventure holidays (for 14 - 18 years of age) Sailing Climbing Windsurfing Fun Games Statement: Safety is of primary importance at the Outdoor Center. All staff members are fully trained in First Aid, and qualified to teach the activities on offer. We also make certain that all children only take part in activities that are suitable for their age and physical abilities. For this programme children must be able to swim 25 metres and be in good physical health. Statements: 56. In August, four people visiting the centre together by car have to pay more than two people. 57. The centre has special equipment for people who learn to sail. 58. The adventure course is suitable for beginners. 59. The centre is planning to add extra facilities to the Play Park. 60. Summer adventure holidays are open to any child between eight and fourteen years who can swim. Section C Short Answer Questions (20 points) Directions: In this section, there are 2 passages followed by 10 questions or unfinished statements. Read the passages carefully, then answer the questions in a maximum of 10 words. Remember to write the answers on the Answer Sheet. Passage One Questions 61-65 are based on the following passage. Going to the beach is many Americans’ favorite activity. In the area near New York City, nine million people used to go to the beach every summer. They went swimming in the ocean without giving a thought to what was underwater. But those days are long gone. In the summer of 1988, the government was forced to shut down beaches all over America. Many of the beaches had to be closed because garbage from hospitals was found in the water. The garbage included glass bottles with samples of blood, and people were afraid they might get AIDS from the blood. Where the medical garbage came from is anybody’s guess. At some beaches, sewage(生活污水) was found in the water. Americans were shocked by this state of affairs. They had long taken for granted that oceans were big enough to stay clean, even if garbage and sewage were dumped into them. People didn’t think of the underwater garbage because it was out of sight. Some of the most polluted waters still look beautiful at first glance. San Francisco Bay is a good example of a beautiful bay that’s full of chemicals. Scientists discovered pollution in some lakes and rivers when they found fish with rotting skin. In many parts of America, people are told not to eat too much fish because of pollution. Most American cities put their garbage in the ground. But New York and a few other cities put their garbage in the ocean. Boston Harbor is so polluted that scientists say it won’t recover until the next century at best. The city of Boston puts its sewage in the water. The government has ordered the city to build a sewage treatment plant. Cleaning up oceans won’t be easy, but people can no longer ignore this challenge. Questions: 61. Most Americans used to go to the beach because of ________. 62. Ocean waters around America have become polluted by ________. 63. Some polluted waters are still beautiful because pollutants such as chemicals are hard to ________. 64. If fish live in polluted waters people should not ________. 65. The author of this passage seems to suggest that people should pay more attention to ________. Passage Two Questions 66-70 are based on the following passage. There are cockroaches (蟑螂) everywhere on Earth except the places that are covered with ice. Scientists have discovered about 3,500 different species of cockroach. There is just one human species! Cockroaches can be anything in size from about five mm to nine cm. Although five mm is very small, nine cm is as long as a large rat. It is very difficult to catch most cockroaches. They “see” with the hairs on their bodies. These hairs can feel the smallest movement in the air, so the cockroaches know immediately something moves, and run to safety. Of all the species of cockroach, fortunately only three live among humans and are a serious problem. They are the German, the Oriental, and the American. One egg case of the German cockroach can produce as many as seven million cockroaches in 12 months! Our main problem with cockroaches is that not only do they look ugly to us, but they also carry diseases. They are particularly dangerous in hospitals as they eat all kinds of hospital waste or get it on their bodies. They can then carry this waste, which may contain dangerous bacteria, on to food which is then eaten by people in the hospital. Most of the bacteria that cause food poisoning have been found in the stomachs of cockroaches, so it is important that cockroaches should be kept out of restaurants and other places where food is prepared. Many people work and try to destroy cockroaches, but as soon as they find one way of doing it, the cockroaches “learn” how to deal with it. Electricity does not always kill them and they can avoid most poisons or “learn” how to deal with others. At one time, scientists thought that radiation would kill them, but they have been on Earth for about 300 million years, and it does not harm them as much as it does us. It seems probable that when there are no longer human beings living on the Earth, cockroaches will still be here. Questions: 66. Cockroaches do not live in places where it is ________. 67. Cockroaches know that someone or something is near because ________. 68. Cockroaches can ________ because they carry bacteria. 69. Paragraph 6 says that it is very difficult to ________. 70. The passage is mainly about ________. Section D Summary (10 points) Directions: In this part, there is one passage followed by a summary. Read the passage carefully and complete the summary with the appropriate words from the passage. Remember to write the answers on the Answer Sheet. Questions 71-75 are based on the following passage. Cosmetics have been used throughout history. The ancient Greeks, the Egyptians, and the Romans all used various kinds of makeup. Some of these cosmetics were used to improve their appearance. Others were used to protect their skin. But in some cases, things used for makeup were dangerous, or even deadly! Some of the first skin care treatments started in Egypt. In fact, Cleopatra was known to use them. She thought a bath in milk and honey left her skin silky smooth. Egyptians also developed some of the earliest sunscreens. They used oils and creams for protection against the sun and dry winds. Egyptian and other ancient cultures also used various powders on their skin for beauty. Egyptians used black kohl around their eyes. Romans put white chalk on their faces. And Indians painted red henna on their bodies. Most of the ancient cosmetic powders, oils, and creams were harmless. But in the name of beauty, some people applied dangerous chemicals and poisons to their skin. During the Italian Renaissance, women wore white powder made of lead on their faces. Of course, doctors now know lead is like a poison for our bodies. Also around the time of the Renaissance, women in Italy put drops of belladonna in their eyes. Belladonna is a very poisonous plant. The poison in the plant affects the nerves in the body. By putting belladonna drops in her eyes, a woman’s pupils would become very large. People thought this made her more beautiful. Actually, this is why the plant is called belladonna. In Italian, belladonna means “beautiful woman.” When Elizabeth I was queen of England in the late 1500s, some rather dangerous cosmetics were also used by women there. Women were using rouge made with mercury. They were also using special hair dye made with lead and sulphur. The dye was designed to give people red hair, the same color as the queen’s hair. Over time, the dye made people’s hair fall out. Finally, women using this dye ended up bald, like the queen, and had to wear wigs. Summary: Although people have used cosmetics throughout history, not all of them have been safe. In fact, some of them have been quite (71) ________ to people. For example, long ago in Italian (72) ________, people thought women with big pupils were beautiful. Therefore, in the (73) ________ of beauty, women began to put (74) ________ of belladonna in their eyes to make their pupils larger. Today we know belladonna is poisonous, and it can affect the (75) ________ in the body. Part VI Cloze (10 minutes, 15 points) Directions: There are 15 blanks in the passage. For each blank, some letters of the word have been given (not exceeding 3 letters). Read the passage below and think of the word which best fits each blank, Use only one word in each blank. Remember to write the answers on the Answer Sheet. I'm really in two minds about what to do when I leave school. Should I go straight to university or should I spend a year traveling (76) a the world? First of all, there are so many (77) ben of going straight to university. The most important point is that the (78) s I get my qualifications, the quicker I'll get a job and start earning. In my opinion, starting work and making (79)m one of the most important things in life. And I'm not (80) al in this opinion. Many consider a sound (81)ca and a good salary to be an important goal. Secondly, if I go straight to university, I'll learn so many things that will help me in my future life. It is often said that knowledge is the key to power, and I cannot (82) dis with this. (83)M , I'll be able to take part in the social activities that the university offers, and meet lots of new friends who share the same interests. However, it could also be (84)ar that I would meet lots of interesting people while I was traveling. (85) Fur , if I spent a year traveling, I would learn more about the world. On the one hand, I would experience lots of different (86) c . On the other hand, I could end up suffering from culture shock, homesickness and some (87) str tropical diseases. Nevertheless, these inconveniences would be an inevitable part of traveling and would be greatly (88) out by the advantages. (89) Unf , another point is that if I spent a year traveling I would need a lot of money. But I (90)b it would be easy to make a bit while I was traveling, giving English lessons or working in hotels and shops. Part VII Translation (15 minutes, 15 points) Section A English-Chinese Translation (10 points) Directions: Translate the underlined sentences of the following passage into Chinese. Remember to write the answers on the Answer Sheet. In such a changing, complex society, formerly simple solutions to informational needs become complicated. (91) Many of life’s problems which were solved by asking family members, friends or colleagues are beyond the capability of the extended family to resolve. Where to turn for expert information and how to determine which expert advice to accept are questions facing many people today. In addition to this, there is the growing mobility of people since World War II. (92) As families move away from their stable community, their friends of many years, and their extended family relationships, the informal flow of information is cut off, and with it the confidence that information will be available when needed and will be trustworthy and reliable. The almost unconscious flow of information about the simplest aspects of living can be cut off. Thus, things once learned subconsciously through the casual communications of the extended family must be consciously learned. Adding to social changes today is an enormous stockpile of information. (93) The individual now has more information available than any generation, and the task of finding that one piece of information relevant to his or her specific problem is complicated, time-consuming and sometimes even overwhelming. Coupled with the growing quantity of information is the development of technologies which enable the storage and delivery of more information with greater speed to more locations than has ever been possible before. Computer technology makes it possible to store vast amounts of data in machine readable files, and to program computers to locate specific information. (94) Telecommunications developments enable the sending of messages via television, radio, and, very shortly, electronic mail, to bombard people with multitudes of messages. Satellites have extended the power of communications to report events at the instant of occurrence. Expertise can be shared worldwide through teleconferencing, and problems in dispute can be settled without the participants leaving their homes and / or jobs to travel to a distant conference site. Technology has facilitated the sharing of information and the storage and delivery of information, thus making more information available to more people. In this world of change and complexity, the need for information is of greatest importance. (95) Those people who have accurate, reliable up-to-date information to solve the day-to-day problems, the critical problems of their business, social and family life, will survive and succeed. “Knowledge is power” may well be the truest saying and access to information may be the most critical requirement of all people. Section B Chinese-English Translation (5 points) Directions: Translate the following sentences into English. Remember to write your answers on the Answer Sheet. 96(毫无疑问,有了计算机的帮助,人们能生活得更容易。 97(我们应该保护森林,使其免受破坏和污染。 98(国家的发展取决于年轻一代的素质,这是众人皆知的事实。 99(我们认为计算机在英语教学中会得到广泛应用。 100(为了不让父母为他担心,他有时不得不说些谎话。 Part VIII Writing (30 minutes, 30 points) Task I: Directions: There is an error in an English magazine that you feel must be corrected. Write a letter to the editor to 1) point out the mistake; 2) suggest the correction; 3) express your interest in the magazine. You should write about 120 words. Do not sign your own name at the end of the letter. Use “Li Ming” instead. You do not need to write the address. Now write the letter on the Answer Sheet. Task II: Directions: Look at the following picture and write an article, which should meet the following two requirements: 1) Interpret the message conveyed by the picture; 2) Make your comments on What is represented. Now-write a composition of about 150 words on the Answer Sheet. 参考答案 Part I Listening Comprehension (25 minutes, 30 points) Section A Short Conversations ( 6 points) 1. D 2. A 3. D 4. C 5. D 6. B Section B Long Conversation (4 points) 7. B 8. B 9. A 10. A Section C News Items (10 points) 11. B 12. A 13. C 14. A 15. A 16. B 17. B 18. C 19. B 20. C Section D Passages ( 10 points ) Passage One 21. D 22. B 23. A 24. C Passage Two 25. B 26. C 27. C 28. B 29. D 30. A Part II Vocabulary and Structure (5 minutes, 10 points) 31. C 32. B 33. A 34. C 35. A 36. A 37. D 38. B 39. D 40. C Part III Situational Dialogues (5 minutes, 5 points) 41. C 42. B 43. B 44. A 45. D Part IV IQ Test (5 minutes, 5 points) 46. D 47. B 48. D 49. C 50. A Part V Reading Comprehension (25 minutes, 40 points) Section A Multiple Choice (5 points) 51. C 52. B 53. A 54. D 55. B Section B Yes / No / Not Given ( 5 points ) 56. N 57. NG 58. Y 59. Y 60. N Section C Short Answer Questions (20 points) 61. swimming 62. garbage, sewage and medical waste 63. discover / see / find 64. eat (too much) fish 65. the ocean pollution / the pollution of oceans / the problem of ocean pollution 66. too cold 67. they can feel / sense their movements (in the air) 68. be very dangerous / hurt human beings 69. kill cockroaches 70. how and where cockroaches live, and difficulty of controlling them Section D Summary (10 points) 71. dangerous 72. culture / society 73. name 74. drops 75. nerves Part VI Cloze (10 minutes, 15 points) 76. around 77. benefits 78. sooner 79. money 80. alone 81. career 82. disagree 83. Moreover 84. argued 85. Furthermore 86. cultures 87. strange 88. outweighed 89. Unfortunately 90. believe Part VII Translation (15 minutes, 15 points) Section A English-Chinese Translation (10 points) 91. 许多原来通过请教家人、朋友或同事便能解决的生活问题,现在却超出了大家庭的能力范围。 92. 随着家庭搬离了稳定的社区和多年的朋友,失去了与大家庭的亲戚们的联系,非正式的信息交流便被切断,同时在需要时就能得到可靠、可信信息的信心也随之丧失了。 93. 今天,一个人可获得的信息比任何时代的人都多,而要找到一条与自己问题相关的信息既复杂又耗时,有时候甚至困难万分。 94. 电信技术的发展使人们有可能通过电视、无线电以及简短的电子邮件铺天盖地般发送大量信息。 95. 那些拥有准确、可信、最新信息并用来解决日常问题、工作中的重要问题以及社会和家庭问题的人就能生存下去并取得成功。 Section B Chinese-English Translation (5 points) 96. There is no doubt that people can live a much easier life with the help of computers. 97. We should protect forests from being destroyed and polluted. 98. It is a well-known fact that the development of a country depends on the quality of the younger generation. 99. We believe that computers will be widely used in the teaching of English. 100. Sometimes he had to tell lies in order not to make his parents worry about him. 2007年全国大学生英语竞赛初赛试题 Part I Listening Comprehension (25 minutes, 30 points) Section A (6 points) Directions: In this section, you will hear 6 short conversations. At the end of each conversation, a question will be asked about what was said. Both the conversation and the question will be read only once. After each question, there will be a pause. During the pause, you must read the three choices marked A, B and C, and decide which is the best answer. Then mark the corresponding letter on the Answer Sheet with a single line through the centre. 1. A. He hasn’t found a suitable one. B. He hasn’t enough money. C. He prefers his old one. 2. A. In ten minutes. B. In fifteen minutes. C. In twenty minutes. 3. A. Have dinner with Mary. B. See a film with Mary. C. Do his homework. 4. A. Rainy. B. Sunny. C. Cloudy. 5. A. He has to go to the bank. B. He has missed the train. C. His train has been delayed. 6. A. ?315. B. ?350. C. ?375. Section B (4 points) Directions: In this section, you will hear one long conversation. The conversation will be read only once. At the end of the conversation, there will be a one-minute pause. During the pause, you must read the four questions, each with three choices marked A, B and C, and decide which is the best answer. Then mark the corresponding letter on the Answer Sheet with a single line through the centre. 7. What office is the student looking for? A. Accounting. B. Economic History. C. Economics. 8. When was the orientation meeting held? A. Yesterday. B. Last Friday. C. A week ago. 9. What is the rule about attendance at lectures? A. It is optional. B. It is necessary. C. It is difficult to enforce. 10. How often does the student have to attend tutorials? A. Once every other week. B. Three times a week. C. Once a week. Section C (10 points) Directions: In this section, you will hear 10 short news items. After each item, there will be a pause. During the pause, you must read the question and then the three choices marked A, B and C, and decide which is the best answer. Then mark the corresponding letter on the Answer Sheet with a single line through the centre. 11. When did the bomb go off in a popular market in central Baghdad? A. Late in the evening. B. At mid-morning. C. Late in the afternoon. 12. What’s the purpose of the Amber Alert program? A. To report the number of children missing in the U.S. every year. B. To help find the children who are believed to have been abducted. C. To find out the reasons why children are kidnapped in the U.S. 13. How many villages have been involved in the inter-communal fighting in Chad? A. Fewer than 8. B. About 10. C. Over 20. 14. What is the best tactic when you’re caught up by a rolling wave of snow? A. To create space around you. B. To outrun the avalanche. C. To leave the ski resort instantly. 15. What’s the news item mainly talking about? A. Development of medical technology. B. Health care in California. C. Health insurance in the U.S. 16. How many people did bird flu kill in Indonesia in two weeks? A. 5. B. 2. C. 35. 17. What’s the function of the new drug produced by Pfizer? A. Reducing dogs’ weight. B. Increasing dogs’ appetite. C. Controlling dogs’ population. 18. What will soon begin in the southern Senegalese towns? A. Planting new agricultural crops. B. Biofuel production. C. Fuel recycling. 19. When did Ponti begin making films? A. In 1913. B. In 1931. C. In 1938. 20. What’s the main reason Democrats could regain control of Congress after 12 years? A. Nancy Pelosi has become their leader. B. Americans are dissatisfied with the Iraq war. C. They support sending more troops to Iraq. Section D (10 points) Directions: In this section, you will hear a short passage. There are ten missing words or phrases in it. Fill in the blanks with the exact words you hear on the tape. Remember to write the answers on the Answer Sheet. Every traveller has a tale to tell about bad driving. These are usually exaggerated (21) __________ of life on the main streets of cities where the motorists are crazy and traffic lights are treated as colourful (22) __________. It all happened on a Sunday April morning. I (23) __________ my intention to turn left into a side road and paused as the oncoming traffic cleared. But as I drove across the road, an elderly man driving a battered Renault 12 chose to overtake me (24) __________ pass on the inside. There was a fearful bang and my car was pushed sideways. I felt groggy and bruised and was taken to hospital in the back seat of a police car. There my spleen was removed. Fortunately, though, my recovery was steady and (25) __________. But that’s not the end of the tale. Yes, I had taken out insurance and it covered my medical expenses, (26) __________ while I recovered and my flights home. However, as always, there was a catch to it. I had (27) __________ hired my car from a backstreet firm called Kavis that had been recommended by the hotel front desk. I was assured that I had full insurance cover, but Kavis (28) __________ my credit card ?800 on the day of the accident. My solicitor took advice on the form I had signed. Being in a hurry at the time, I hadn’t (29) __________ to get it translated. “Mr. Balmer was stupid to sign this document,” it read. I (30) __________ with a smile. After all, I am still alive! Part II Vocabulary and Structure (10 minutes, 15 points) Directions: There are 15 incomplete sentences in this part. For each blank there are four choices marked A, B, C and D. Choose the one that best completes the sentence. Then mark the corresponding letter on the Answer Sheet with a single line through the centre. 31. If I lose my new watch my parents will be very annoyed ______ me. A. against B. with C. for D. about 32. I don’t think ______ possible to master a foreign language without much memory work. A. this B. that C. its D. it 33. Does brain power ______ as we get older? Scientists now have some surprising answers. A. descend B. decline C. reduce D. collapse 34. I don’t believe in Brown’s adventure at all. It sounds so ______. A. incredible B. credible C. incredulous D. credulous 35. We’ll never get to the station on time ______ we run as fast as we can. A. in case B. even if C. provided that D. if only 36. Oil sales volume in local urban and rural areas rose by 24% and 50% ______, over July 2006. A. separately B. individually C. independently D. respectively 37. The shipping authorities followed the ______ of the unidentified submarine on their radar screens. A. channel B. way C. course D. direction 38. I set off as soon as I got the news about my grandpa’s illness. Unfortunately, he ______ at the hospital before I ______. A. has died; went there B. had died; arrived C. died; had been there D. was dying; had gone 39. The committee has made rules ______ all its members are supposed to work. A. by which B. so that C. now that D. for which 40. They were married for fifteen years but have now ______ up and live apart. A. split B. divided C. divorced D. separated 41. We wouldn’t have missed the train if we ______ to the station. A. didn’t walk B. wouldn’t run C. hadn’t walked D. weren’t running 42. No matter what measure they took, in no way ______. A. could the outflowing tide be controlled B. the outflowing tide could be controlled C. could the outflowing tide control D. the outflowing tide could control 43. —Did you get any information from Peter? —He ______ an expert, but he doesn’t seem to know much. A. believes to be B. is supposed to be C. thought to be D. is claimed being 44. —The radio’s terribly loud. Could you turn it down a little? —Sorry! ______ —Yes, and something else - wouldn’t it be an idea to buy your own soap? A. Is it disturbing you? B. I forgot where I put my soap this morning. C. A football match was broadcast live on it. D. Could you repeat what you said? 45. —Can I book a room from now until Friday? —_________________ —What’s the price? —$128.75 not counting the service. A. Definitely. Go see it yourself. B. Yes, our hotel is quite near to the station. C. Of course. Would you like to follow me? D. You can have Room 33, overlooking the sea. Part III IQ Test (5 minutes, 5 points) Directions: There are 5 IQ Test questions in this part. For each question there are 4 choices marked A, B, C and D. Mark your answer on the Answer Sheet with a single line through the centre. 46. Which figure completes the sequence? 47. A man has 29 socks in his drawer, 9 identical blue, 8 identical grey and 12 identical black. The lights have fused and he is completely in the dark. How many socks must he take out to make certain that he has a pair of each colour? A. 21 B. 14 C. 23 D. 24 48. Which comes next? A, 1A, 111A, 311A,? A. 1312A B. 13211A C. 1231A D. 11231A 49. Find the weight to balance the scales. A. 2kg B. 4kg C. 3kg D. 5kg 50. What is the missing number? A. 748 B. 754 C. 745 D. 784 Part IV Reading Comprehension (25 minutes, 40 points) Section A (6 points) Directions: There is one passage in this section with 6 questions. For each question, there are four choices marked A, B, C and D. You should decide on the best choice. Then mark the corresponding letter on the Answer Sheet with a single line through the centre. Questions 51-56 are based on the following passage. considered the leader of a new breed of Indian “A Unique Experience in Indian Cuisine” cuisine in Hampshire,” he added. Opening night at the Gandhi Restaurant “By far the best curry we have had in the brought cries of praise and delight from Portsmouth area,” was the comment of Havant customers when they sampled the unique diners, Mr. and Mrs. Jim Cairns of Denvilles. cuisine on Monday night. “We enjoyed the different menu and found Opened by County Councillor, Mr. Tony the advice of the staff, explaining how each dish Peaston, the Gandhi offers the discerning diner was prepared, very helpful for deciding our authentic Indian dishes, many available for the choice of menu,” they said. first time in Hampshire. Such glowing comments reflect the The secret lies in the preparation - only exclusive nature of the dishes available at the authentic Indian herbs and spices are used to Gandhi Restaurant. individually prepare each special dish, following ancient recipes, many handed down through Original Recipes generations of Indian chefs. You can choose from a menu which offers High Standard curries and tandoories, knowing each one is specially cooked for you, with individual care “I’ve travelled extensively and dined at and attention, according to strict original many Indian restaurants throughout the country, recipes, by a top London chef, formerly of but rarely have I tasted Indian food of such a Covent Garden. high standard,” extolled Councillor Peaston. And after you have sampled the spicy “Whilst Gandhi himself was a leader of delights of your main course, you can select men, the Gandhi Restaurant could be from a choice of original Indian sweets to temper your palate. Relaxing in the comfortable surroundings of the restaurant, you can have a hot towel to freshen yourself or clean your fingers between courses - another touch of Indian living. The Gandhi Restaurant, situated at 139 Kingston Road, Portsmouth, is fully licensed and open seven days a week. You can pop in for a traditional Indian lunch between 12 and 2:30 pm or enjoy a languid evening meal, when the restaurant is open from 6 pm to midnight. But take care to book in advance, as demand for this cuisine is expected to be high, so avoid disappointment by telephoning Portsmouth 811966. As proof of confidence in your enjoyment, the Gandhi is offering a 10 per cent discount on the cost of your meal, when you produce this advertisement within three months of the opening. The staff at the Gandhi look forward to serving you with your first taste of truly authentic Indian cuisine in this area- and they know you will come back again and again. 51. “A Unique Experience in Indian Cuisine” here means that the Gandhi restaurant __________. A. is the only genuine Indian restaurant in Hampshire B. offers customers its own special style of cooking C. makes special food for native Indian customers D. is trying out Indian recipes for the first time 52. How does the food preparation contribute to the unique, experience? A. It varies from customer to customer, depending on the price. B. The methods used are secret and known only to the chef. C. It uses genuine Indian ingredients and follows ancient recipes. D. It follows methods described by traditional Indian authors. 53. According to the article, in what way does the restaurant live up to its name? A. It will soon be as famous throughout the world as Gandhi himself. B. The owner considers himself to be a leader of the local Indian community. C. It has had a considerable influence on other Hampshire restaurants. D. It sets new standards of excellence for other Indian restaurants to follow. 54. Where did the top chef of the Gandhi Restaurant use to work? A. In London. B. In Hampshire. C. In Denvilles. D. In Portsmouth. 55. Hot towels are provided __________. A. because customers feel cold B. to make customers feel more relaxed C. mainly for hygienic reasons D. to give customers the true feel of Indian life 56. Customers can obtain a cheaper meal by __________. A. bringing the advertisement with them B. making a phone call to the restaurant C. providing proof of their enjoyment D. contributing to the cost of advertising Section B (7 points) Directions: In this section, there is one passage followed by 7 statements. Go over the passage quickly and mark the answers on the Answer Sheet. For questions 57- 63, mark Y (for Yes) if the statement agrees with the information given in the passage; N (for No) if the statement contradicts the information given in the passage; NG (for Not Given) if the information is not given in the passage. Questions 57-63 are based on the following passage. Eleven days ago, as he felt himself being crushed from head to toe, Andrew Jepson was certain he was about to die. Yesterday the 26-year-old construction worker was able to stroll around his parents’ home and tell for the first time how he survived being run over by a four-ton road roller. The accident happened as Mr. Jepson worked at a building site at Heathrow and the roller was reversing at 4 mph. “I was looking away and then I just felt it go over my leg”, he said. “I fell to the floor and felt this amazing pain in my leg as it was being crushed.” “When I felt it running over me I thought this is it, I’m dead. Nobody can survive this.” “There was absolutely nothing I could do. The pain was horrendous and all the air was being crushed out of my lungs.” “I couldn’t breathe. I thought every breath would be my last.” The machine took more than a second to roll over Mr. Jepson’s body. “It came over my head but I had my hard hat on which must have blocked most of the pressure,” he said. Workmates believed he would be dead but the uneven surface, foundation for a road, cushioned much of the weight of the roller and saved him. “Afterwards I tried to get straight up,” he said. “I was fully conscious. I was in shock but for some reason I thought, ‘If I stay down, I will die.’” “Luckily a labourer ran over and kept me down so that I would not do any more injury to myself.” “The driver rushed over. He was really upset. I said, ‘You are not to blame, it was just an accident.’” Mr. Jepson, a site engineer, suffered crushed ribs, bruising, cuts and a collapsed left lung but escaped permanent injury. An air ambulance arrived and doctors operated at the scene. He spent two days in intensive care but six days later was back at his parents’ home near Spalding, Lincolnshire. “It is like being born again,” said Mr. Jepson, who has amazed doctors with his recovery. “When I came around from the anaesthetic I just cried with joy. Being able to walk and breathe again is the most wonderful thing in the world.” Mr. Jepson’s 62-year-old father, John, said, “When we heard he had been under a roller I feared the worst. We prayed all the way to the hospital. To see him now is just wonderful.” Mr. Jepson’s employer, construction firm Laing, said the accident was still being investigated. Statements: 57. The accident happened two weeks ago. 58. Mr. Jepson was paving the road when the accident happened. 59. The roller was going backwards when it hit Mr. Jepson. 60. A workman ran immediately to the scene of the accident. 61. An operation was performed at the scene of the accident. 62. The company has decided who is responsible for the accident. 63. The investigation of the accident lasted for almost two years. Section C (7 points) Directions: You are going to read a magazine article about sand. Seven sentences have been removed from the article. Choose from the sentences (A-H) the one which fits each gap (64-70). There is one extra sentence which you do not need to use. Mark your answers on the Answer Sheet. Sand: as children we play on it and as adults we relax on it. It is something we complain about when it gets in our eyes on a windy beach, and praise when it is made into sand castles. But we don’t often look at it. If we did, we would discover an account of a geological past and a history of sea life that goes back thousands and, in some cases, mil- lions of years. Sand covers not just seashores, but also ocean beds, deserts and mountains. 64 And it is a major element in manufactured products too - concrete is largely sand, while glass is made of little else. Well, it is larger than fine dust and smaller than shingle. In fact, according to the 65most generally accepted scheme of measurement, grains can be called sand if their diameter is greater than 0.06 of a millimetre and less than 0.6 of a millimetre. Depending on its age and origin, a particular sand can consist of tiny stones or porous grains through which water can pass. They have come from the breaking down of rocks, or 66from the dead bodies of sea creatures, which collect on the bottom of the oceans, or even from volcanic eruptions. If it is a dazzling white, its grains may come from nearby coral, from crystalline 67quartz rocks or from gypsum, like the white sand of New Mexico. On Pacific Islands, jet black sands form from volcanic minerals. Other black beaches are magnetic and are mined for iron ore. It washes rock into streams and rivers and down to the sea, leaving be- hind softer materials. By the time it reaches the sea, the hardest rocks remain but every- thing else has 68 been broken into tiny particles of 0.02 millimetre diameter or less. The largest pieces fall to the bottom quickly, while smaller particles float and settle only slowly in deeper water, which is why the sandy beach on the shoreline so often turns to mud further out. If the individual fragments still have sharp edges, you can be sure they were formed fairly recently. This is the case on the island of Kamoama in Hawaii, where a beach was 69created after a volcanic eruption in 1990. Molten lava spilled into the sea and exploded into glassy droplets. It seems that when the poet William Blake saw infinity in a grain of sand he was not far wrong. Sand is an irreplaceable industrial ingredient which has many uses. Sand cushions our land from the force of the sea, and geologists say it often does a better job protecting our shores than the most advanced coastal technology. 70 A. These may have the shape of stars or spirals, their edges rough or smooth. B. It is one of the most common substances on earth. C. In addition, it has one vital function which you might never even notice. D. Rain is an important force in the creation of beaches. E. In the great slow cycle of the earth, sand that was once rock can turn to rock again. F. What exactly is sand? G. Colour is another clue to the origins of sand. H. It can be difficult to date the sand on a beach accurately but it is possible to get a general idea of whether or not the sand is “young” or “old”. Section D (10 points) Directions: In this section, there is one passage followed by 5 questions. Read the passage carefully, then answer the questions in as few words as possible (not more than 10 words). Remember to write the answers on the Answer Sheet. Questions 71-75 are based on the following passage. Rare among American actors, Depp has made a name for himself effortlessly switching between mainstream Hollywood movies and more “out of the ordinary” projects. Talking about his choice of roles, he once said, “With any part you play, there is a certain amount of yourself in it. There has to be, otherwise it’s not acting. It’s lying.” Highlights of a richly diverse career include Edward Scissorhands, Sleepy Hollow and Pirates of the Caribbean. Depp dropped out of school at sixteen to concentrate on a career in music, playing the guitar (he played with more than twenty bands). However, his musical career rifled to take off, and he found himself selling pens over the phone to pay the bills. His lucky break came when makeup artist Loft Allison, to whom he was briefly married, introduced him to Nicolas Cage. Although at first they did not like each other, they later became good friends and Cage persuaded him to try acting. Depp signed on with Cage’s agent, and made his feature film debut in Wes Craven’s horror film Nightmare on Elm Street, in which the character he played was eaten by his bed. After that he had his first screen leading role in Private Resort. Depp went on to achieve teen idol status in the TV series 21 Jump Street, but after four seasons, he wanted out, with the hope of making the transition to the big screen. He starred in Cry-Baby, followed by Tim Burton’s Edward Scissorhands, after which he went on to win considerable critical acclaim in Ed Wood, a reunion with Burton. Depp made his feature directorial debut with The Brave in 1997, a film he also co-wrote and starred in. Premiering at the Cannes Film Festival, the film also featured Marlon Brando, but earned mostly negative reviews, with most critics blaming its weak script. Sleepy Hollow teamed him with director Burton yet again, before he starred in Ted Demme’s Blow, and appeared in the thriller From Hell, about Jack the Ripper. OFF screen, his good looks and “bad boy” image (he was once arrested for attacking intrusive paparazzi with a wooden plank) have earned him a lot of media attention. He was voted one of the fifty most beautiful people in the world by People magazine in 1996. He has also had his fair share of celebrity romances; when his engagement to Edward Scissorhands co-star Winona Ryder ended, he had a tattoo (one of at least eight), which said “Winona Forever”, altered by laser to get rid of the last two letters of her name. His relationship with model Kate Moss also ended abruptly in 1998, when he started dating French singer-actress Vanessa Paradis. They are now married and have two children, Lily-Rose Melody and Jack. More recent work has included Pirates of the Caribbean with Geoffrey Rush and Once Upon a Time in Mexico. Questions: 71. What does Johnny Depp think of acting a role without having any part of oneself in it? 72. Who inspired Depp to start his film career? 73. Why did many critics negatively review Depp’s film The Brave? 74. How many films has Depp made with director Tim Burton? 75. What have brought Depp a lot of media attention off screen? Section E (10 points) Directions: In this section, there is one passage followed by a summary. Read the pas- sage carefully and complete the summary below by choosing a maximum of three words from the passage to fill in the spaces 76-80. Remember to write the answers on the Answer Sheet. Questions 76-80 are based on the following passage. Lack of culture, or rather an excess of the wrong sort of culture, is often considered to be synonymous with disadvantage. Most commonly associated with low cultural standards are low levels of reading, and some thirteen per cent of all twenty-three-year-olds feel they have trouble with reading and writing. One way of compensating such disadvantaged young people is thought to be to provide them with the culture they lack: in particular, high quality reading material. Whereas forty to fifty per cent of young people aged sixteen to twenty rarely read a book, the majority appear to read comics. In 1991 sales of Viz, a UK comic, exceeded one million copies per issue, making it the fourth best-selling periodical in Britain. The reading of comics, however, is not restricted to young people: by 1992 it was estimated that two out of three men aged eighteen to fifty-three read Viz. The number of imitators this comic has spawned, including Zit, Gas, Brain Damage and Swiz, indicates the extent of the influence it wields. The reading of comics was traditionally regarded by the educational establishment with considerable suspicion. Whereas the received arts were always assumed to exert an improving or civilizing influence, comics were thought to “rot children’s brains”, to lower educational standards and to threaten morality. They were, and are, assumed to be an inferior cultural form, their readers assumed to come from the lower social classes, to be low educational attainers and to be easily led astray. Over the past decade, perceptions of comics have shifted. Since the 1970s, the comic format has been commonly used to represent the interests of various disenfranchised groups — community groups, the unemployed, welfare recipients — who became more conscious of a climate conditioned by other contemporary movements such as civil fights, consumerism, self-help and de-institutionalization. As cultural signifiers, comics have become the subject matter of academic courses in cultural and media studies. Indeed, young people’s cultural activities, grounded in the commercial rather than the subsidized sector, are beginning to merit the attention of the arts establishment. Summary: Low cultural standards, such as 76 of reading, a difficulty experienced by many young adults, are often associated with disadvantage. While around half of sixteen to twenty-year-olds rarely read books, most will read comics. Although many comics in Britain are 77 and have lots of readers, the educational establishment still considers them to be an 78 , appealing only to the lower levels of society. However, attitudes are beginning to change as the format has been adopted to 79 of disenfranchised groups. Certain comics have been included in the courses of 80 studies. Young people’s cultural activities are beginning to attract the attention of the arts establishment. Part V Cloze (10 minutes, 10 points) Directions: There are 10 blanks in the passage. Use the words (phrases) given in the box to fill in the blanks, changing the form where necessary. Use only one word (phrase) in each blank. There are two extra words (phrases) which you do not need to use. Remember to write the answers on the Answer Sheet. anxious about, strong, suggest, instruct, which, especial, whether, apply to, all, both, great, form Working out to music can improve the coordination of your mind and body, (81) __________ you are football crazy or keen on tennis. The (82) __________ that rock or pop music might ever play a part in sports training would have been regarded as a joke not so long ago. But today modem music is increasingly filling the gym as well as the front room. The idea of exercise to music is not new. For years, (83) __________ in eastern Europe, the benefits of sportsmen and sportswomen having (84) __________in ballet and classical dance, with their stress on total body control and balance, have long been recognised. Figure-skating and ice-dance are usually performed to music and can be said to be specialised (85) __________ of this type of exercise. But ballet and classical dance can be (86) __________ other sports that are also pleasing to the eye, such as gymnastics and skiing, (87) __________ of which demand high standards of balance, coordination and suppleness. In western Europe and North America, a far (88) __________ interest has been shown in working out to classical music. Even sports which seem to demand muscular (89) __________ more than any other physical requirement have taken up exercise to music as a valuable addition to their own specialised training schemes. Devotees of soccer, rugby, and rowing now regularly train to music; even those who take part in weightlifting, (90) __________ demands enormous physical strength, and participants in athletics field events, find that exercise to music is beneficial and makes their movements more fluid. Part VI Translation (15 minutes, 20 points) Section A (10 points) Directions: Translate the underlined sentences in the following passage into Chinese. Remember to write the answers on the Answer Sheet. (91) An active lifestyle and a healthy, fish-rich diet are not only good for your heart, they may also help tackle the memory loss associated with old age, two leading neuroscientists said. As people live longer, finding ways of halting the decline in mental agility is becoming increasingly important, said Professor Ian Robertson, director of the Institute of Neuroscience at Trinity College Dublin. (92) “The biggest threat to being able to function well and properly is our brains,” he told journalists. “There is very strong evidence, particularly in the over-50s, that the degree to which you maintain your mental faculties depends on a handful of quite simple environmental factors,” he said. (93) Those who remained physically fit, avoided high stress levels and enjoyed a rich and varied social life are better equipped to stay alert as they age. Mental stimulation, learning new things and simply thinking young also help. A new survey compiled for the University of Kent and the charity Age Concern showed ageism was rife in Britain where people, on average, see youth as ending at 49 and old age beginning at 65. But Robertson said such attitudes were not helpful given the number of 80-year- olds who remain “sharp as pins”. (94) “If you start to think of yourself as old when you are 60, which is no longer justified, you will behave old,” he said. Research conducted by his Trinity College colleague, Professor Marina Lynch, showed healthy eating was another key requirement for staying on the ball. (95) Lynch said new research showed fish oils may reduce the cell inflammation that triggers a decline in memory. Section B (10 points) Directions: Translate the following sentences into English, using the words given in the brackets. Remember to write your answers on the Answer Sheet. 96(这决不是解决人口问题的最好办法。(by no means) 97(首先,我要感谢那些为取得这项成果而辛勤工作的所有人。(bring about) 98(越来越多的新能源被开发出来代替煤和石油。(exploit) 99(有必要让人们充分认识违反交通规则的危害。(aware of) 100(看到古代的生活完整地保存下来,我们是多么高兴啊! (preserve) Part VII Writing (30 minutes, 30 points) Task I (10 points) Directions: You are in England helping to organise a course for foreign students which begins next week. This morning you received a message from Katarina Tabacek, one of the students who has reserved a place on the course. She wants to bring a friend with her on the course. Look at the description of the course below and the notes you have made and write to Katarina explaining why her friend cannot come on the course. Write a letter of between 80-120 words in an appropriate style on the Answer Sheet. Do not write your address or name. Task II (20 points) Directions: Read the following extract of a newspaper article on personal freedom. Then write an article (between 110-150 words) responding to the points raised and expressing your own views. Week after week we seem to hear about some reckless adventure that has gone wrong. Ina recent Sydney to Hobart ocean yacht race, the boats sailed into violent storms. Most of the sailors were rescued, but only as a result of a massive rescue operation which cost a huge amount of money and put rescuers’ lives at risk. Similar tragedies have occurred in other dangerous sports, costing the taxpayer thousands and wasting the emergency services’ time. Isn’t it time to ban dangerous sports and protect people who put themselves and others at risk? Or would a ban restrict our freedom too much? Please write on the Answer Sheet. 2007年全国大学生英语竞赛初赛赛题(C类)参考答案 Part I Listening Comprehension (25 minutes, 30 points) Section A (6 points) 1. A 2. C 3. B 4. B 5. C 6. B Section B (4 points) 7. C 8. A 9. B 10. C Section C (10 points) ll. B 12. B 13. C 14. A 15. B 16. A l7. A 18. B 19. C 20. B Section D (10 points) 21. accounts 22. decorations 23. signalled 24. rather than 25. complete 26. accommodation 27. foolishly 28. charged 29. bothered 30. paid up Part II Vocabulary and Structure (10 minutes, 15 points) 31. B 32. D 33. B 34. A 35. B 36. D 37. C 38. B 39. A 40.A 41. C 42. A 43. B 44. A 45. D Part III IQ Test (5 minutes, 5 points) 46. B 47. C 48. B 49. A 50. B Part IV Reading Comprehension (25 minutes, 40 points) Section A (6 points) 51. B 52. C 53. D 54. A 55. D 56. A Section B (7 points) 57. N 58. NG 59. Y 60. Y 61. Y 62. N 63. NG Section C (7 points) 64. B 65. F 66. A 67. G 68. D 69. H 70. C Section D (10 points) 71. Lying. / Not acting, but lying. 72. Nicolas Cage. 73. They thought its script was weak. 74. Three. 75. His good looks and "bad boy" image. Section E (10 points) 76. low levels 77. best-selling periodicals 78. inferior cultural form 79. represent the interests 80. cultural and media Part V Cloze (10 minutes, 10 points) 81. whether 82. suggestion 83. especially 84. instruction 85. forms 86. applied to 87. both 88. greater 89. strength 90. which Part VI Translation (15 minutes, 20 points) Section A (10 points) 91(两位著名的神经专家认为,积极的生活方式和健康的、富含鱼类的饮食不仅对心脏有好 处,还有助于攻克由衰老引起的记忆力减退。 92(他告诉记者说:“对我们肌体正常运转构成最大威胁的是我们的大脑。” 93(那些身体健康、不为压力困扰、享受多姿多彩生活的人,在上了年纪后,思维更能保持 敏锐。 94(他说:“尽管60岁已不再算老,但如果你60岁就开始认为自己老了,那么你的举止就 会象老年人一样。 95(林奇说最新的研究显示:鱼油能降低细胞炎症,而细胞炎症正是会导致记忆力衰退。 Section B (10 points) 96. This is by no means the best way to solve the population problem. This is by no means the best solution to the population problem. By no means is this the best way to solve the population problem. 97. First (of all), I'd like to thank all those who have worked hard to bring about the result. 98. More and more new energy resources have been exploited to replace coal and oil. 99. It is necessary to make people fully aware of the danger of violating traffic rules / regulations. 100. What a delight it is for us to see ancient life perfectly preserved! How delighted we are to see ancient life so well preserved! How happy we are to see ancient life preserved so well! 2009年全国大学生英语竞赛初赛试题 Part I listening Comprehension (25 minutes,30 marks) Section A (5 marks) In this section, you will hear five short conversations. Each conversation will be read only once .After each conversation, there will be a pause. During the pause, read the question and the three choices marked A, B and C, and decide which is the best answer .Then mark the corresponding letter on the answer sheet with a single line through the centre. Where does this conversation probably take place? A.In an engine room . B .in a car. C.In a factory. 2.Who is the man talking to ? A.A lifeguard . B.A travel agent . C.A gymnasium manager. 3.How will the correct price be determined ? A. By calling the company. B.By asking the sales representative. C.By looking at the price list. 4.Why is Sue upset according to the conversation? A.Someone broke into her car. B.She had to pay too much for parking. C.She was given a ticket . 5.What do the two speakers probably do for a living. A.Office clerks. B.Printers. C.Archiects. Section B (10 marks) In this section, you will hear two long conversations. Each conversation will be read only once. After each conversation, there will be a one-minute pause. During the pause, read the five questions, each with the three choices marked A, Band C, and decide which is the best answer. Then mark the corresponding letter on the answer sheet with a single line through the centre Conversation One 6.Why did the man watch the video ? A.It was required by Professor Stephen. B.He wanted to learn about heart attacks. C.He had to take part in a discussion. 7.In the man’s opinion, what is the video about? A. Men’s health. B.Stress. C.Health care for women . 8.What did the man learn from the video? A.Women are under more stress than men . B.Women have more heart attacks than men . C.Women who have heart attacks get less care than men . 9.At what ages are women most likely to have a heart attack? A.40 to 50 . B.60 to 65 . C.45 to 60. 10.What does the woman think about the video? A.It sounds puzzling. B.It sounds very interesting . C.It sounds very dull. Conversation Two 11.How long did the man wait before his call was answered? A.Twenty minutes. B.Twelve minutes . C.Twenty-four minutes . 12.Why did the man phone Interserve Customer Service? A.He wanted to know what the recorded message means . B.He didn’t know which button to choose. C.He wanted to know if he can use his e-mail address while overseas. 13.What does the man think about the menu of options? A.Its meaning isn’t ambiguous. B.It is far from clear . C.It is satisfactory. 14.Which department would the man have spoken to if had chosen button two? A.Accounts and Billing. B.General Enquiries. C.Accounts and Service. 15.What will the operator do next ? A.Make an official complaint. B.Improve the menu of options. C.Make a note of the man’s complaint. Section C (5 marks ) In this section, you will hear five short news items .Each item will be read only once. After each item ,there will be a pause ,During the pause ,read the question and the three choices marked A,B and C, and decide which is the best answer .Then mark the corresponding letter on the answer sheet with a single line through the centre. 16.How does rotational grazing reduce the need for pesticide treatments? A.By increasing the use of natural fertilizer. B.By limiting the need for chemical fertilizers. C.By reducing the growth of weeds. 17.Which news agency votes for the top ten news stories? A.Reuters. B.The Associated Press C.United Press International. 18.Which counties spent billions of dollars to develop new oil fields ? A. Syria, Brazil and Russia. B.Brazil,Iran and Saudi Arabia . C.Saudi Arabie ,Brazil and Russia. 19.Why has the number of camels in India been decreasing in recent years? A.Because they are dying from traveling too far . B.Because they are short of food. C.Because they are infected with a disease. 20.What is a symptom of multiple sclerosis? A.Loss of the ability to hear. B.Loss of the ability to see well. C.Loss of the ability to speak clearly. Section D(10 marks) In this section, you will hear a short passage which will be read only once. There are 10 missing words or phrases. Fill in the blanks with the exact words or phrases you hear on the tape .Remember to write the answers on the answer sheet . The fourth component of the Swedish social system is that its policies ensure full employment and increase the mobility of labour-that is ,the ability to change jobs without (21)______income. Swedes have a strong work ethic .This ,combined with state-funded programmers that(22)_______unemployment, which Sweden prefers to high unemployment rates and large welfare (23)______.So, there are a large number of (24)_______training programmes that unemployers par for (25)___________to workers in jobs that have become unnecessary. There are also subsidies for workers who must change jobs because of changes in the labour market . Sweden introduced the fifth and final component of its social system at the end of the 20th century. This component corrects some of the problems created by the four (26)________mentioned components. It includes new rules that are designed to improve job stability , such as training programmes to help employees learn new skills as their jobs change and become more (27)______,Also ,the government has reduced social(28)_________and welfare payments . So, in conclusion, the Swedes are happy with their system. Sweden has achieved more in terms of social equality, economic(29)________than many other economies. As a result ,most Swedes aren’t interested in any more reform of the economic system .However, because it is (30)_________in international markets, Sweden must continue to manage its social policies so that it remains competitive in the international marketplace. Part II Vocabulary and Structure(10 minutes,15 marks) 31.The businessmen discussed the contract_____________but never actually signed anything. A.at length . B.at sea . C.at randon. D.at will. 32.Accustomed to the coffee from Columbia________Governor John said that ________coffee should always be strong. A./;a. B.the;a. C.a;a. D./;/. 33._________a well-balanced diet,________adequate sleep ,is needed for good health. A.Alongside;with. B.Not only;but. C.Attached;to . D.Due to;so 34._________very familiar with inspection reports, I realized that some pages of this one were missing. A.To be B.That was C.Being D.So was 35.If you have not signed a contract, you are under no _________ to pay them any money. A.responsibility B.liability C.circumstance D.obligation 36.As the market was _________ goods, the economy became more balanced , and inflation went down. A.speculated on B.subscribed for C.saturated with D.submitted to 37.Surveys show that the majority of passengers are pleased that an agreement has been reached to forbid smoking on _________ flights within the continental United States. A. economical B. commercial C. global D. internal 38.The new CEO asked that all inter-office communications ________ in writing rather than in person , _________ possible. A.have been made; whoever B.would be made; whichever C.be made ; whenever D.stage bare of scenery 39.Written to be performed on a __________,Thornton Wilder’s play Our Town depicts life in a small New England community. A.stage scenery of bare B.bare of stage scenery C.scenery bare of stage D.stage bare of scenery 40.____________ school policy , the students had a holiday between Chritmas and the New Year. A.On behalf of B.With regard to C.In line with D.In case of 41.To sell the house , we made a __________ to the buyer by agreeing to put a new roof on it. A.consideration B.commission C.concession D.confirmation 42.Never before _________ available for quick and easy acess in so many different fields of study. A.so much free information were B.has so much free information been C.were so much free information D.so much free information has been 43.The computer company announced that the new solfware __________ in the first half of next year. A.be released B.will have released C.would have released D.was going to be released 44.Tina: A group of us are going boeling on Tuesday night. Would you like to join us? Joe : I really would , but unfortunately I have a tennis lesson. Tina : Oh, that’s too bad. _________ Joe : Yes, for sure. Thanks for the offer. A.How do you deal with that ? B.Maybe another time then ? C.When will you come back ? D.Don’t you really like bowling ? 45.George : Hey, Agnes . How did the Human Resources meeting go ?I couldn’t make it because I was out at a sales conference. Agnes : Oh, George . __________ It turned into a major argument. George : Really ? A .Glad you’re back now. B.You’re lucky to have missed it. C.Nice to see you again. D.You’re just on time. Part III Reading Comprehension (20 minutes, 35 marks) Section A (5 marks ) In this section , there is one passage followed by five question. For each question, there are four choices marked A, B, C and D .You should decide on the best choice, and then mark the corresponding letter on the answer sheet with a single line through the centre. Questions 46 to 50 are based on the following passage. When e-mail first came into general use about fifteen years ago, there was a lot of talk about the imminent arrival of the paperless office. However, it seems that e-mail has yet to revolutionize office communications .According to communicate ions analyst Richard Metcalf, some offices have actually seen an increase in paper as increase in paper as a result of e-mail.” Information in the form of e-mail messages now floods our computer screens. These messages can be distributed in the hundres.For those secretaries whose books ask them to print out all their e-mails and leave them in their intrays, this means using up a great deal of paper every month ,”Metcalf says. Metcalf has found that because some e-mails get lost in cyberspace, as are increasingly likely to be asked by clients and colleagues to send all important documents both by e-mails and by fax or “snail mail”-though the post .This highlightsafuther postential problem with e-mail in today’s offices-it is taking up time rather than saving it. “With e-mail, communication is much easier ,but there is also more room for misunderstrandings,”says psaychologist Dr David Lewis Generally ,much lsee care is take with e-mails than with letters or faxes and the sender will probably print the the documents and reread it before putting it in an envelope or sending it by fax. More worrying is still the increase misuse of e-mail for sending “flame-mail” –abusive or inappropriate e-mail messages .Recent research in several companies suggests that aggressive communications like this are on the increase .E-mail has become the perfect medium for expressing workplace frustration because is so instant. E-mail can also be a problem in order ways .Staff all too often make the mistake of thinking that the condense of an e-mail ,like things said over the phone ,are private and not permanent. But it is not only possible for an employer to read all your e-mails, it is also perfectly legal-mail messages can be traced back to their origin for a period of at last two years , so you might want to rethink e-mailing your frustrations about your job to your friends. The advice is to keep personal e-mails out of the office . It goes without saying that e-mail exists to make life easier, and if used correctly, it is an invaluable tool for business of all sizes. But perhaps, for the time being ,the fact that in the business word 70persent of all documents are still in paper form is not such a bad thing after all. 46.Why has the promise of the paperless office not come true in many offices? A.People write more memos than they used to. B.Maybe secretaries keep paper copies of everything their bosses send and receive . C.Many managers prefer to read their messages on paper . D.Staff leave messagers lying around their offices. 47.What dose Richard Metcalf say about e-mail in Paragraph Two? A.It is not an appropriate channel for sending important information B.It increases the amount of paperwork done in offices. C.It is not popular with many secretaries. 48.How does David Lewis feel about e-mail messages? A.Many of them are not well written . B.Many of them are too long . C.Many of them are not printed out. D.Many of them are never read 49.Why should employees not use company e-mail systems for personal messages? A.Because company e-mail systems are not easy to use. B.Because the people the messages sent to can’t. C.Because they allow people to express their anger immediately. D.Because e-mail is not a private means of communication. 50.What dose the writer conclude about e-mail in the last paragraph? A.It has already made life a lot easier for many business. B.It is not being used enough in business today. C.It will never replace written communication. D.It is really useful when people use it properly. Section B(10 marks) In this section, there is one passage followed by five incomplete sentences. Read the passage carefully, and then complete the sentences in a maximum of there words for each blank. Remember to write the answers on the answer sheet. Questions 51 to 55 are based on the following passage. It is well known that some cultures prefer sons to daughters. In contrast, surveys have revealed that some people would rather have a daughter than a son. Apparently, each of these preference has its explanation in the local culture . Around the world , people who prefer sons have similar reasons .In many countries , sons are valued because they can work and contribute to the family income . Also , sons will be able to support their parents when the parents are old . In some cultures , sons are important because they enable the family name to continue for another generation . This is because the wife in these cultures takes her husband’s family name , and more or less “belongs ”to the husband’s family . In addition , having a daughter can sometimes cause difficult for a family . In India , for example , having a daughter can be a burden because the family has to pay a dowry when the girl gets married . Generally ,people think traditional cultures are the ones in which boys are preferred . Research seems to support this belief . However , according to a survey conducted in 2000, people in the Czech Republic , Lithuania , Portugal , and the Caribbean prefer to have daughters . The researchers said that parents in these societies do not need to have the same economic situation that exists in other countries , so they do not need to have a son to help make money . Instead , families in these countries want daughters for emotional reasons . Many mothers think they will have a better relationship with a daughter than a son , for example . In addition ,they think daughters will take better care of them when they are old . In Japan , a similar trend has emerged . This might come as a surprise to some , because only one generation ago , the Japanese favored sons . In 1982 , over half of Japanese couples surveyed would want a boy if they could have only one child . A survey done in 1997 , on the other hand , found that 75 percent wanted a girl ! Some Japanese couples say they want to have daughters because they think daughters are cuter and easier to raise . Japanese couples also worry about being taken care of when they are old . If they have to live with their children when they are very old , many Japanese parents would want to live with a daughter . “A sons wife would be a stranger and harder to ask for help ,” one Japanese mother said . Critics warn that preference for a daughter is just as bad as preference for a son . “Instead of valuing children for the social roles they are expected to fill , parents must learn to value children as individuals ,” critics say . when that is the case, parents will be equally pleased with a son or daughter . Questions: 51. The reasons that parents prefer to have sons are to continue____ , to Help earn money for the family , and to support the parents in their old Age . 52. Japan’s preference for daughters now is quite____ because not long ago The preference was for sons . 53. In some developing countries , parents value boys more than girls mainly for _____ . 54 . In India , parents receive _______ when their sons get married . 55. Critics maintain that children should be treated by their parents as ___. Section C (10 marks ) In this section ,there is one passage followed by five questions . Read the passage carefully and then answer each of the questions in a maximum of 10 words . Remember to write the answers on the answer sheet . Questions 56 to 60 are based on the following passage. At least 600,000 jobs could disappear in the UK this year , according to a report by a personnel managers’ professional body . The Chartered Institute of Personnel and Development says even those who escape redundancy will face pay freezes .It says that while total unemployment will not hit three million , the time between New Year and Easter will be the worst for job losses since 1991 . According to official statistics , there were 1.86 million people out of work in the UK in October , 2008. This figure was the highest since 1997 , taking the overall unemployment rate to 6 percent . David Frost of the British Chamber of Commerce says , “It is the worst year . Companies are in survival mode .” The CIPD , which represents managers and personnel staff , issued its gloomy forecast a day after children’s retailer Adams joined a growing list of well known chains applying for bankruptcy protection .Chief economist John Philpott said , “In the face of some skepticism , the CIPD warned that 2008 would the UK’s worst year for jobs in a decade . It was , but in retrospect it will be seen as merely the slow –motion prelude to what will be the worst year for jobs in almost two decades . The CIPD’s annual barometer forecast is that the UK economy will shed at least 600,000 jobs in 2009 .Overall ,the 18-month period from the start of the recession in mid-2008 until the end of 2009 will witeness the loss of around three quarters of a million jobs , equivalent to the total net rise in employment in the preceding three years .” Mr Philpott said job losses were likely to continue into 2010 ,taking the final toll to about one million . The CIPD also surveyed 2,600 workers and found that more than one in four did not expect a pay rise next year ,while others feared wage cuts . The institute’s reward adviser ,Charles Cotton ,said ,”Employees are realistic about their pay prospects . Against this backdrop ,employers will need to work hard to find new ways to motivate their employees to perform .” “Financial incentives and targeted investment in training and development could be effective ways to do this ,” he added . “More than ever , this is a time where organizations need to engage in an open and straightforward communication with their staff , clearly explaining the reasons for any difficult measures that will affect them ,” he said . “This will help preserve staff loyalty and engagement even during times when unpopular decisions need to made .” Questions: 56. According to the passage , which will be the worst year for job losses In the UK ? 57. What kind of organization is the Chartered Institute of Personnel and Development ? 58. What are many British companies now struggling for ? 59. What does the CIPD advise British employers to do under the present circumstances? 60. Why does Charles Cotton suggest that employers communicate with their straight—forwardly ? Section D (10 marks) In this section, there is one passage followed by a summary. Read the passage carefully, and then complete the summary below by choosing a maximum of three words from the passage to full in each of the blanks. Remember ton write the answers on the answer sheet. Questions 61 to 65 are based on the following passage. Everyone knows about pollution in the environment. Water, air, and land are all polluted. This means that pollution is everywhere .Now, scientists are looking inside our bodies to find out about internal pollution. In 2003, the Environmental Working Group studied nine people to measure the chemicals in there bodies. These nine people had an average of 53 cancer-causing chemicals in their bodies .They also had average of 62 chemicals that can harm babies in pregnant women .Even though a lot of chemicals were found in human bodies , the chemicals were found in small amounts .The amounts were small enough that they were probably not hurting the people . However, scientists are worried because most of these chemicals were created by humans. Most of these chemicals did not exist 75b years ago. This proves that we have not only polluted the world – we have polluted our own babies! How dose this pollution get into our bodies? We come into contact with many chemicals everyday .for example, everyone use soap, skin lotion, and shampoo. However, few people know that these products contain harmful chemicals, some of which may cause memory loss. Chemicals known as DEA(diethanolamine) and TEA(triethanolamine) may seep into the skin and stop us from absorbing a helpful nutrient called “choline”. Choline is a nutrient that plays a crucial role in the memory cell making progress in the developing brain. Some people worry that every time we wash our hair, we are decreasing our memories. Everyone knows they should brush their teeth regularly, but recent research has shown that some toothpastes can cause cancer. Fluoride is a key ingredient in many toothpastes. However, studies show that fluoride dose not really protect our teeth. In fact, fluoride has been connected with bone cancer and other diseases. While we are trying to save our smiles, we could be damaging our health. Women are probably exposed to more chemicals then men because they use more beauty products. Recently, harmful chemicals are called “phthalates”. Studies on animals have shown that phthalates can damage the liver, the kidneys, and the lungs. Cosmetic companies say the amounts of phthalates in their products are safe. But consumer protection groups disagree. They say some women try to look beautiful, the more they could be harming their health. In a futuristic story by Ray Bradbury, a man found a pristine stream on a new planet. When he drank from the stream, he died! Why? His body was so polluted that pure water was a position to him! Perhaps that is only science fiction, but it reminds us to take care of our bodies. We must find ways to reduce the pollution we absorb. Summary: Like our polluted environment, our bodies are no longer pollution-free. Scientists now know that there is pollution in our bodies. On (61) , we have 53 cancer-causing chemicals in our bodies. Even fluoride (62) with bone cancer and other diseases. Some common cosmetics contain small amounts of chemicals, which seep into our (63) when we use these products. They prevent us from absorbing the (64) our bodies need. So, it is (65) that we find ways to reduce the amount of chemicals in our bodies. We should remind our friends and families to take care of their bodies. Part IV Cloze(15 minutes,15 marks) Read the passage and fill in each blank with one word. Choose the word in one of the following three ways: according to the context , by using the correct from of the given word ,or by using the given letters of the word. Remember to write the answers on the answer sheet. According to a group called the Voice Function, everyone has a singing voice as well as a(66)——(speak) voice somewhere inside them. This, they say, should be (67) enc ——from an early age because it provides the best, and the (68) ——(cheap), basis on which to build an understanding of music. The teaching of the Hungarian (69) (compose) Zoltan Kodaly are responsible for this idea. He observed that songs can become a key part of the relationship (70)_______a mother and her child almost from birth . This is (71) as______ true of traditional societies, (72)_______those of West Africa , where some small children are able to sing literally(73)hun______ of songs , all of which have been learn by heart . However many modern children first come to understanding of music (74)_______they learn to play an instrument (75)_______although some teaching of the theory of music is usually a part of this , their relationship with the music on the page is often a mechanical one. The (76)____(believe) of the Voice Foundation is that a natural feeling for rhythm. harmony and musical structure .the very qualities we appreciate in the greatest musicians. can only be achieved through the (77)____(explore) of the voice from the beginning of a person’s life .The foundation has , therefore ,set itself the task of (78) _____a singing-centered musical education(79)pro_____that could benefit junior all(80)_____the world. Part V Translation (15minutes, 20 marks) Section A (10 marks) Translate the underlined sentences of the following passage into Chinese. Remember to write the answer sheet. Embroidery (刺绣) is a brilliant pearl in Chinese art .(81) From the magnificent Dragon Robes worn by emperors to today’s fashions, embroidery adds a great deal of pleasure to our culture and our lives. The oldest embroidery on record in China dates from the Shang Dynasty. Embroidery in this period symbolized social status. (82) It was not until later on, as the national economy developed, that embroidery entered the lives of the common people. After the Zhou Dynasty, the Han Dynasty witnessed a leap in both techniques and styles of embroidery. Embroidered objects ranged from the sun, the moon, stars, mountains dragons, and phoenixes to tigers, flowers and grasses, clouds and geometric patterns. Auspicious words were also fashionable. Both historic records and products of the time prove this. The cultural relics found in the Mawangdui Han Tomb are the best evidence of this unprecedented development in embroidery. In addition, embroidery unearthed from the Mogao Caves in Dunhuang , the ancient tombs in Turpan and northern Inner Mongolia ,further strengthen this observation . (84) The Chinese word for embroidery is “xiu”, a picture or embroidery in five colours .It implies beautiful and magnificent things. Embroidery was an elegant task for ladies who were forbidden to go out of their homes. (85)It was a good pastime to which they could devote their intelligence and passion . Imagine a beautiful young lady embroidering a dainty pouch .Stitch by stitch, she embroiders a pair of love birds for her lover .It’s a cold winter day and the room is filled with the aroma of incense. What a touching and beautiful picture! Section B (10marks) Translate the following sentences into English, using the words or structures required in the brackets .Remember to write the answers on the answer sheet. 86.她先天残疾,但他从未屈服于任何困难。(倒装句) 87.无论什么时候说什么事情,你至少要让别人听得懂。(whenever) 88.对于一个公司来说,能跟上市场的发展的变化是很重要的。(keep pace with) 89.如果你通过旅行社预定你的行程,你可能会得到很大的折扣 。(discount) 90(当初要是投资电信业,我们现在会很富有。(虚拟语气) Part VI IQ Test (5 minutes,5 marks) There are five IQ questions in this part .Write the answers on the answer sheet. 91. If all the letters of the alphabet were written out backwards ,which would be the next but one letter after the third vowel? 92. The top two scales are in perfect balance. How many spades (黑桃) will be needed to balance the bottom set? 93. If FRTZZ is the code for ESSAY ,what is the code for PAPER, 94. A, B, C,D ,E and F are six seaside resorts. A,C and F have a fun pair and a caravan park ,whereas C has a caravan park only .The others have neither .There are amusements at all the resorts except B and C. Which resort without a caravan park has amusements but no promenade, On the left are some shapes and the codes that go with them .You must decide how the code letters go with the shapes. Which code should replace the question mark on the right? XM YN ? ZM Part VII Writing (30 minutes, 30 marks) Task I (10 marks) You must organize a two-day debating contest which will take place on May 1-2 .Write an e-mail to your classmate Tom:: , informing him that you’re working on a schedule for the Contest and that you will send it by e-mail tomorrow , asking him to get in touch with some teachers to act as judges-you need to know their names by Friday , asking him to come up with some ideas by tomorrow morning You should write no fewer than 80 words on the answer sheet. Task II (20 marks) You have received this letter from English –speaking Penfield .Read the letter and write your report about fashion industries. We’re doing a project at college on fashion industries in different countries .It would be nice to include a first-hand report form someone living abroad .The report should describe how fashions for young people have changed recent years. You should write no fewer than 120 words on the answer sheet. 参考答案及作文评分标准 Part I Listening Comprehension (25 minutes, 30 marks) Section A (5 marks) 1. B 2. B 3. C 4. A 5. C Section B (10 marks) 6. A 7. B 8. C 9. C 10. B 11. A 12. C 13. B 14. A 15. C Section C (5 marks) 16. C 17. B 18. C 19. B 20. C Section D (10 marks) 21. losing 22. minimise 23. expenditures 24. vocational 25. available 26. previously 27. complex 28. insurance 29. security and freedom 30. heavily involved Part II Vocabulary and Structure (10 minutes, 15 marks) 31. A 32. D 33. B 34. C 35. D 36. C 37. B 38. C 39. D 40. C 41. C 42. B 43. D 44. B 45. B Part III Reading Comprehension (20 minutes, 35 marks) Section A (5 marks) 46. B 47. A 48. A 49. D 50. D Section B (10 marks) 51. the family name 52. surprising 53. economic reasons 54. dowries 55. individuals Section C (10 marks) 56. 2009. 57. It’s a professional body representing managers and personnel staff. 58. Survival. 59. Find new ways to motivate their employees to perform. 60. To help preserve staff loyalty and engagement. Section D (10 marks) 61. average 62. has been connected / is connected 63. skin 64. nutrients 65. crucial Part IV Cloze (15 minutes, 15 marks) 66. speaking 67. encouraged 68. cheapest 69. composer 70. between 71. especially 72. like / including 73. hundreds 74. when 75. and 76. belief 77. exploration 78. developing / designing 79. program(me) 80. over Part V Translation (15 minutes, 20 marks) Section A (10 marks) 81. 从皇帝身上华丽的龙袍到当今的时装,刺绣给我们的文化和生活增添了许多乐趣。 82. 直到后来,随着国家经济的发展,刺绣才进入平常百姓的生活。 83. 马王堆汉墓出土的文物是这个时期刺绣空前发展的最好见证。 84. 汉语“刺绣”一词中的“绣”,意思是由五种颜色组成的图画或刺绣,它暗示着美丽壮观的 事物。 85. 倾注了智慧和热情,刺绣确实是大家闺秀用来打发时间的一种很好的消遣。 Section B (10 marks) 86. She was disabled from birth, but never did she give in to any difficulty. 87. Whenever you say anything, you must at least make yourself understood. 88. It is important for a business to keep pace with changes in the market. 89. If you book your trip through a travel agency, you’re likely to get a big discount. 90. Had we invested in the telecommunications industry, we would be quite rich by now. Part VI IQ Test (5 minutes, 5 marks) 91. G. 92. Four spades. 93. QZQDS. 94. E resort. 95. ZN. 2010年全国大学生英语竞赛初赛试题 Part I Listening Comprehension (25 minutes, 30 marks) Section A (5 marks) In this section, you will hear five short conversations. At the end of each conversation, a question will be asked about what was said. Both the conversation and the question will be read only once. After each question, there will be a pause. During the pause, read the three choices marked A, B and C, and decide which is the best answer. Then mark the corresponding letter on the Answer Sheet with a single line through the center. 1What is the woman probably doing now? A.She is writing an essay. B.She is studying for a test. C.She is shopping for shoes. 2.How did the woman feel according to the conversation? A.She was relaxed B.She was pleased C.She was disappointed 3.What does the woman say about her presentation? A.It’s far from being ready B.She got a lot of information from the internet C.She needs another week to get it ready 4.Where will the company probably hold the stuff party? A B C 5.Which picture shows the corrections to the man’s name? Section B (10 marks) In this section, you will hear two long conversations. Each conversation will be read only once. At the end of each conversation, there will be a one-minute pause. During the pause, read the five questions, each with three choices marked A, B and C, and decide which is the best answer. Then mark the corresponding letter on the Answer Sheet with a single line through the center. 6. What does the man want to talk to Ann about? A.A holiday trip to Yellowstone Park B.A research project in Yellowstone Park C.A lecture by a professor who visited Yellowstone Park A B C Section B (10 marks) In this section, you will hear two long conversations. Each conversation will be read only once. At the end of each conversation, there will be a one-minute pause. During the pause, read the five questions, each with three choices marked A, B and C, and decide which is the best answer. Then mark the corresponding letter on the Answer Sheet with a single line through the center. Conversation one 6. What does the man want to talk to Ann about? A.A holiday trip to Yellowstone Park B.A research project in Yellowstone Park C.A lecture by a professor who visited Yellowstone Park 7. According to the man, why is the buffalo population increasing in Yellowstone Park? A. a lot of buffalo have come from neighbouring areas. B. Fewer buffalo are dying of disease C. It is easier now for the buffalo to find food in winter. 8. Why does the man think Ann would be interested in going to Yellowstone ? A. She has been studying animal diseases B. She is eager to visit Yellowstone Park C. She needs the money to continue her studies 9. What did the woman plan to do in July? A. Work on her thesis B. Have a holiday C. Study the buffalo population 10. Where will the woman most probably spend the coming summer? A. At the University of Wyoming B. At Yellowstone National Park C. At her friend’s home in Wyoming. Conversation Two 11. What was Matt Ryan’s first job? A. Doing holiday relief work at a television channel B. Working part-time as a model C. Taking pictures for a television station 12. What did Matt find interesting about the sixties? A. The increasing number of comic books B. The rapid advances in technology C. The American space programme 13. Why were Matt’s models once used on the news? A. They presented better images than the real pictures B. The spacecraft camera got damaged and failed to take any pictures C. The television studio was trying some new ideas 14. What was the name of the programme that marked the beginning of Matt’s TV career? A. Strange Creatures. B. Time Traveller. C. Bright Star. 15. What did Matt do for the programme? A. He made models. B. He acted the part of a monster. C. He filmed it. Section C (5 marks) In this section, you will hear five short news item. Each item will be read only once. After each item, there will be a pause. During the pause, read the question and the three choices marked A, B and C, and decide which is the best answer. Then mark the corresponding letter on the Answer Sheet with a single line through the center 16. How many times has the National Hockey League allowed its players to take part in the Winter Olympics? A. Five times. B. Three times. C. Twice. 17. Where was the Africa Cup of Nations held? A. In Togo. B. In South Africa. C. In Angola. 18. What has caused a large number of people to flee their homes in northern Yemen? A. An armed conflict. B. Lack of food. C. Freezing temperatures. 19. Who is Emtiaz Sooliman? A. A search and rescue specialist. B. Head of a South African foundation. C. Leader of a local civic group. 20. What did the second stimulus bill passed by the House of Representatives aim at? A. Promoting international trade. B. Boosting employment. C. Expanding the private sector. Section D (10 marks) In this section, you are required to fill 10 blanks, each with NO MORE THAN THREE WORDS after listening to a short passage. The passage will be read twice. Remember to write the answers on the answer sheet. Children’s Toys ? Most popular wooden toy: (21) ? The wooden toy is for (22) ? Output per (23) :4,000 items ? Average (24) time per box (25) minutes ? The number of boxes in (26) :1,000 ? Date of the coming dispatch (27) ? Current number of (28) :20 ? Number of staff working on the (29) :40 staff ? All staff do: (30) work Part II Vocabulary and Structures ( 15 marks ) There are 15 incomplete sentences in this part. For each blank there are four choices, marked A, B C and D. Choose the one that best completes the sentence, then mark the corresponding letter on the Answer Sheet with a single line through the center. 31. When Ian was injured, Harry was chosen as last-minute for the rugby team. A. preference B. diversification C. alternative D. replacement 32.—Hello, Mr. Brown, I’m ringing about our component delivery. It’s not arrived yet and it’s already three o’clock in the afternoon. —Let’s see...it’s reach you on Tuesday afternoon. A. owing to B. likely to C. due to D. subject to 33. Among the last groups of people to accept the new model were religious groups, who still the idea that the earth was the center of the universe. A. clung to B. applied to C. adapted to D. contributed to 34. Although apparently rigid, bones exhibit a degree of elasticity that enables the skeleton to considerable impact. A. escape B. overwhelm C. withstand D. suppress 35.—Would you like me to go to the dentist with you? —No, you with me. A. need not to go B. need not go C. do not need go D. not need go 36. Lance returned to cycling and training only five months after he was diagnosed with cancer. A. aggressively B. drastically C. exactly D. initially 37. Great minds generally look at life in a way to themselves. A. peculiar B. confined C. similar D. unusual 38. They called in an electrician he could put a finger on the cause of the short circuit. A. to hope B. to be hoping C. hoping D. to have hoped 39. The resistance experienced when one body moves over another, it is in contact, is called frictional force. A. to which B. where C. with which D. while 40. Above all, they want to study a question: Are humans actually aware of the world they live in? A. contrary B. fundamental C. solemn D. progressive 41.—Tina, I hear you had a good journey to the Maldives last week, How was it? —I enjoyed the beautiful scenery, but the hotel was satisfactory. A. not anything B. nothing from C. nothing but D. anything but 42.At the beginning of the 20th century, people made coffee a cloth bag full of coffee grounds into boiling water. A. by dumping B. to dump C. for dumping D. that dumped 43. If you Susan recently, you’d think the photograph on the right was strange. A. shouldn’t contact B. hadn’t contact C. weren’t to contact D. didn’t contact 44. Beata: I’ve put the job advertisement in the newspaper, Mr. Trim. Trim: Good. Beata: Well, it was a bit more than the ,10 that they quoted us. Trim: As long as it wasn’t ,10 a day. A. How to schedule it? B. How much was it? C. How often was it put there? D. How about the newspaper? 45. Woman: Who is Jackie Tow? I have a parcel here for him. Jackie: That’s me. Woman: Yes, put your name here. Jackie: Thank you. It must be the new pair of shoes I bought online. A. This is the receipt for it. B. Do you have the sender’s address? C. I have to check the package D. Do I have to sign for it? Part III Cloze (10 marks) Read the following passage and fill in each blank with one word. Choose the word in one of the following three ways: according to the context, by using the correct form of the given word, or by using the given letters of the word. Remember to write the answers on the answer sheet. Where The Wild Thing Are “I didn’t set out to make a children’s movie,” says Being john Malkovich director Spike Jonze , “I set out to make a movie about childhood.” Indeed, like the recent alternative children’s book (46) adapt , Fantastic Mr Fox, this is more like an adult film (47) children’s clothing ,or rather in Jim Henson monster suits. When rambunctious (无法无天) nine-year-old Max feels (48) ig by his busy single mum and her new boyfriend, and runs away (49) home , he finds himself on an island populated by huge, hairy, scary Wild Things. Here, he gets himself crowned king, and he and the monsters fight and play, and throw mud at each other (which the younger viewers will love). They return home. That’s it .And that’s your problem. As a film , I can’t (50) de it’s disappointing. Despite whimsical (异想天开) imaginative and heart-tugging moments, Jonze and co-writer Dave Eggers inevitably lose the wonderful subtlety (精妙之处) of Maurice sendak’s well-loved 338-word picture book just by (51) (spin) it out into a full-length feature. Enough already! We get this dysfunctional group of neurotic Wild Things (52) rep Max’s child’s eye view of grown-ups as comprehensible giants, both terrifying and loving. (53) , even the repetitive action and non-subtleties (54) ,make this feel more like family therapy than a fairy (55)t can’t destroy the film’s haunting magic. Part IV Reading Comprehension (40marks) Read the following passages carefully and answer the questions given. Remember to write the answers on the answer sheet. Section A (10 marks) HOLDAYS IN WALES 1. Rhos –Ddu Country Cottages Ynys, Criccieth LL532 OPB Delightful hideaway cottages with private fishing. Comfortably furnished with antiques, old oak beams, log fires, giving the aura and grace of a bygone age, but with all the essentials of the 20th century –sauna, jacuzzi , four poster bed ,snooker table… A holiday venue one rarely finds –but often dreams of. For Enquiries contact: Mrs A Jones Rhandir, Boduan, Pwllheli Gwynedd, LL53 8UA 2. Five Star Cottage Talhenbont hall, Talhenbont, Criccieth, Gwynedd Enq: Roger & Gillian Good Lovingly restored stone cottage and hunting lodge in 70 acre wooded country estate with river. A luxurious and carefree holiday for the discerning. Available throughout the year. Woodland walks and wildlife.Free tennis, riding and fishing .One mile from coast, five miles Snowdonia . Under the personal supervision of the owners. 3.Windsor Flats 12 Marine Terrace, Criccieth,Gwynedd,LL52 OEF On sea front close to Criccieth Castle.Comfortable, clean,fully equipped,completely private flats,in picturesque village central for Snowdonia and the Llyn Peninula. Climbing, sailing, fishing, walking, tennis and golf closeby. Colour TV,payphone.Bed linen supplied. Short breaks available out of season.Write or phone for brochure. 4.Dwyach Cottages Cticciech, Gwynedd Enquiries: Mrs S Edwards Pen-y-Bryn, Chwilog, Pwllheli, Gwyhedd,LL53 6SX Enjoy a peaceful holiday in beautifully situated farmhouse or single storey cottage. This is an area of unrivalled natural beauty, the haunt of buzzards and woodpeckers. Cottages are superbly equipped to make your holiday relaxed and memorable. Dishwasher , washer/dryer, microwave, linen, children’s play area, barbecue and farm trail. 5.Bron Afon Self Catering Borth-y-Gest, Phorthmadog, Gwynedd, LL49 9TU Situated only minutes from the beach with fabulous views of garden, sea and mountains. The accommodation is quiet and private. An ideal base for touring , walking, climbing, fishing, the slate mines, castles, Portmeirion, Porthmadog leisure centre or just relaxing on the beach. Bed & Breakfast also available. Question 56 to 58: Decide whether the following statements are true(T) or false(F) according to the advertisements. 56.To have a holiday in a delightful hideaway cottage ,you should contact Mrs S Edwards. 57.In the Cticcieth Five Star Hotel , free tennis ,game fishing, riding and golf are available. 58.Short breaks are available out of season in the Llyn Peninsula. Question 59 to 60:Answer the following questions briefly according to the advertisements. 59. If you plan to have a holiday with your children, what is the best place? 60.What will you enjoy most in Bron Afon Self Catering? Section B(10 marks) The giant panda,the creature that has become a symbol of conservation ,is facing extinction. The major reason is loss of habitat, which has contitued despite the establishment of 14 panda reserves . Deforestation , mainly carried out by farmers clearing land to make way for fields as they move higher into the mountains,has drastically contracted the mammal’s range. The panda has disappeared from much of central and eastern China, and is now restricted to the eastern flank of the Himalayas. Satellite imagery has shown the seriousness of the situation ; almost half of the panda’s habitat has been cut down or degraded since 1975.Worse ,the surviving panda population has also become fragmented; a combination of satellite imagery and ground surveys reveals panda “islands” in patches of forest separated by cleared land. The population of these islands has become isolated because the animals are loath to cross open areas .Just putting a road through panda habitat may be enough to split a population in two. The minuscule size of the panda populations worries conservationists. The smallest groups have too few animals to be viable, and will inevitably die out .The larger populations may be viable in the short term, but will be susceptible to genetic defects as a result of inbreeding. In these circumstances, a more traditional threat to pandas – the cycle of flowering and subsequent withering of the bamboo that is their staple food – can become literally species-threatening. The flowering prompts pandas to move from one area to another, thus preventing inbreeding in otherwise sedentary populations. In pandas, however, bamboo flowering could prove catastrophic(灾难性的)because the pandas are unable emigrate. The latest conservation management plan for the panda, prepared by China’s Ministry of Forestry and the World Wide Fund for Nature, aims primarily at maintaining panda habitats and ensuring that populations are linked wherever possible. This plan will change some existing reserve boundaries, establish 14 new reserves and protect or replant corridors-of forest between panda islands. Other measures include better control of poaching, reducing the degradation of habitats outside reserves, and reforestation. The plan is ambitious. Implementation will be expensive and will require participation by individuals ranging from villagers to government officials. Question 61 to 65: Complete the summary with words from the passage, changing the form where necessary, only one word for each blank. The survival of the giant panda is being seriously (61) .This is largely because the overall size of their habitat has been reduced. As a result, pandas are more prone to problems and are unable to (62) around freely, following the growth cycles of (63) plant. A new plan aims to protect existing panda (64) and to join some of them together. This plan also involves reforestation and the creation of new reserves. To succeed, everyone, (65) both the government and individuals, will have to cooperate. Section C (10 marks) Honda has developed a way to read patterns of electric currents on a person’s scalp as well as changes in cerebral blood flow when a person thinks about four simple movements-moving the right hand, moving the left hand, running and eating. 67 In a video shown at Tokyo headquarters, a person wearing a helmet sat still but thought about moving his right hand – a thought that was picked up by electrodes attached to his head inside the helmet. 68 Honda said the technology wasn’t quite ready for a live demonstration because of possible distractions in the person’s thinking. Another problem is that brain patterns differ greatly among individuals, and so about two to three hours of studying them in advance are needed for the technology to work. The company, a leader in robotics, acknowledged the technology was still at a basic research stage, with no immediate practical applications in the works. 69 Japan boasts one of the leading robotics industries in the world, and the government is pushing to develop the industry as a road to growth. Research on the brain is being tackled around the world, but Honda said its research was among the most advanced in figuring out ways to read brain patterns without having to hurt the person, such as embedding (植入) sensors into the skin . Honda has made robotics a centerpiece of its image, sending Asimo to events and starring the walking, talking robot in TV ads. 70 “Our products are for people to use. It is important for us to understand human behavior, ” he said.“We think this is the ultimate in making a machine move.” Question 66:Choose the best answer according to the passage. 66. What does Honda say about its robot Asimo? A. It can detect a person’s way of thinking. B. It can imitate complex human movements. C. It is still in its experimental stage. D. It is ready for market distribution. Question 67 to 70: Choose the most appropriate of the following paragraphs that fit into questions 67 to 70 in the passage. A. “I’m talking about dreams today,” said Yasuhisa Arai, executive at the Honda Research Institute in Japan, the company’s research unit. “Practical uses are still way into the future.” B. Among the challenges for this brain technology is making the reading-device smaller so it can be portable, according to Honda. C. After several seconds, Asimo, programmed to respond to these brain signals, lifted its right arm. D. Honda succeeded in analysing these thought patterns, and then relayed them as wireless commands to Asimo, its human-shaped robot. Section D (10marks) Being an introvert is a bad thing, right? Well, a lot of people seem to think so, judging by the number of articles I’ve read about how to “cure” introversion. In response to these articles, I wrote The Introverts Strike Back, in which I argued that introverts can’t become extraverts, and they shouldn’t particularly want to. However, I’m not here to debate whether it’s better to be an introvert to an extravert. The fact is, we all have to interact with both types of people every day. Regardless of which type you are, you can greatly improve your relationships by learning to get along better with people of the other type. Here are some tips for getting started. For Introverts: *Indicate to others when you’re busy. When an extravert sees you reading, writing, or maybe just thinking, he might assume that the only reason you are doing this is because you don’t have someone to talk to. So he thinks he’s doing you a favor by striking up a conversation, when he’s actually interrupting. To prevent this, be sure to give an indication that you’re in the middle of something and don’t want to socialise right now. This can be a visual sign or verbal. I know one person who tended to get a lot of visitors at work, and while he was actually an extravert, the frequent visits were slowing him down too much. He put a sign on his door saying “If I don’t make eye contact or respond to you, I apologize. I’m not trying to be rude, I just have a lot of work to do. Thank you for understanding.” While I don’t think many people need to go that far, it certainly worked! *Realise that extraverts often need to talk. Because extraverts are more in touch with the external world, for them talking is something as necessary as breathing. They might think out loud by bouncing their thoughts off other people, and they might need to chat in order to boost their energy. For an introvert, this can be the most difficult part of dealing with an extravert. The same conversation that energises the extravert also drains the introvert. However, keeping in mind that the extravert is not being intentionally malicious, the introvert has at least two options for handling this in a polite way. They can patiently participate in the conversation, and then, when it’s over, recharge by being alone. Or they can cut off the conversation early by mentioning something else they need to be doing, or even by saying, “I’d like to help, but I’m not sure that I’m the right person for you to be talking to.” Of course, sometimes a conversation can be very enjoyable for an introvert, in which case this isn’t a problem. For Extraverts: *Ask if someone is busy before spending time with them. If someone appears to be lonely, they might not be. Even if they’re just sitting there and don’t seem to be doing anything, they could be deep in thought and not want to be interrupted. If you need something, try to ask for it up front. Otherwise, look for clues that they might not feel like talking right now, such as lack of eye contact. If they seem uninterested, don’t take it personally. You just don’t know what you’re interrupting. *Understand what a draining effect a conversation can have on someone. No matter how fabulous a person you are, keep in mind that introverts simply prefer their internal world to the external world. They might start off with a fully changed battery, but while they’re engaged in conversation, that battery is steadily draining. How long it lasts depends on various factors, but be sure to keep an eye out for when they’re starting to lose interest. Be more to the point with introverts, and save most of your chatting for extraverts who will appreciate it more. Questions 71 to 74: Complete each of the following sentences with NO MORE THAN FIVE WORDS according to the passage. 71. Introverts are advised to to prevent interrupting when they are in the middle of something. 72. For extraverts, having a conversation with people can be as important as 73. An important clue that someone might not feel like talking with you right now is 74. It is necessary to remember that introverts tend to focus more on Question 75: Choose the best answer according to the passage. 75. What is the passage mainly about? A. Whether it is better to be an introvert or an extravert. B. How to get along with both introverts and extraverts. C. How to avoid embarrassment in conversations. D. One should be neither an introvert nor an extravert. Part V Translation (20marks) Section A (10 marks) Translate the underlined sentences in the following passage into Chinese. Remember to write the answers on the answer sheet. A soaring dropout rate is causing the United Stated to lose ground educationally to rivals abroad and is trapping millions of young American at the very margins of the economy. (76)The Obama administration acknowledges the problems in its new budget, which includes a ,50 million prevention programme, but solving this problem will require a lot more money and a comprehensive national strategy. The alarming scope of the dropout crisis is laid out by the Center for Labor Market Studies at Northeastern University in Boston and the Alternative Schools Network in Chicago. (77)Their study, which examines data from the 12 largest states, finds that 16 percent of people from the ages of 16 to 24 have dropped out. (78) The problem is especially pronounced among men, who make up more than 60 percent of those who change school nationally. The dropout problem hits minorities really hard. (79) Many of this country’s large urban high schools are rightly called “dropout factories” because more students leave school than graduate. According to the study, state dropout rates are highest in the South, where Gerorgia (22.1 percent), Florida (20.1 percent) and Texas (18.5 percent) lead the way. (80) The dropout crisis presents a clear danger to national prosperity, but at the moment, states and localities are struggling to contain it with little help or guidance from the federal government. Congress, which is just waking up to this issue, can improve the situation by the putting its money and muscle behind proven programmes that have been shown to re-engage young people who have dropped out, and that keep at-risk children on track to complete their educations. Section B (10 marks) Translate the following sentences into English, using the words given in brackets. Remember to write the answers on the answer sheet. 81.石油价格的飞速上涨对世界经济产生了很大的影响。( affect ) 82.奥林匹克运动会上最重要的不是取胜而是参与。( not„but„) 83.早知道你没有钥匙,我就不把门锁上了。( would not have) 84.花这么多的时间和精力来安排这样一个活动,值得吗,( worthwhile ) 85.这些邮票由于其独特的艺术设计吸引了众多集邮者。( appeal to ) Part VI IQ Test (5 marks) There are five IQ test questions in this part. Remember to write the answers on the answers sheet. 86. What number comes next? 87. When David is twice as old as he is now he will be four times as old as his daughter Jane will be in five years time. If in 1990, four years ago,he was four times as old as his daughter, in what year was she born? 88. The cost of hiring a private rail carriage is shared equally by all the passengers who paid an exact number of pounds which was less than ,100 each. The carriage has seats for 50 passengers and the total bill amounts to ,1887. How many seats were not occupied? 89. In how many ways can the word TURN be read? Start at the central letter T and move in any direction. N N R N N R U R N N R U T U R N N R U R N N R N N 90. What is the largest number of regions this ring could be divided into using three straight lines? Part VII Writing ( 30 marks ) Task I (30 marks) You have been asked to organize a special event as part of the school’s English Book Week.Suggestions include a book sale, poetry readings and an opportunity to meet the author of one of your English course books. Write a notice for the school notice board to advertise the event. ? Describe the kind of event you are planning ? State exactly where and when it will be held ? Ask for volunteers to help with preparations ? Say how you can be contacted You should write no fewer than 100 words on the answer sheet. Task II (20 marks) You are a college student. Recently, a national newspaper has been running a special weekly insert on careers, and you are going to contribute to an article entitled “Key Considerations for the School Leaver”. Please state the most important issues facing young people when they decide on a career path. You should write no fewer than 120 words on the answer sheet. 2010年全国大学生英语竞赛C类试题参考答案及作文评分标准 Section A (5 marks) 1. B 2. C 3. A 4. C 5. C Section B (10 marks) 6. B 7. C 8. A 9. A 10. B 11. A 12. C 13. B 14. C 15. A Section C (5 marks) 16. C 17. C 18. A 19. B 20. B Section D (10 marks) 21. train sets 22. the under-fives 23. month 24. packaging 25. five 26.storage 27. November 3rd 28. drivers 29. production lines 30. shift Part II Vocabulary and Structure (15 marks) 31. D 32. C 33. A 34. C 35. B 36. D 37. A 38. C 39. C 40. B 41. A 42. A 43. B 44. B 45. D Part III Cloze (10 marks) 46.adaptation 47. in 48. ignored 49. from 50.deny 51. spinning 52. representing 53.However 54. that/which 55. tale Part IV Reading Comprehension (40 marks) Section A (10 marks) 56. F 57. F 58. T 59. Dwyfach Coggages 60. The beach Section B (10 marks) 61. threatened 62. move 63. bamboo 64. habitats 65. including Section C (10 marks) 66. C 67. D 68. C 69. A 70. B Section D (10 marks) 71. give an indication signal 72. breathing 73. lack of eye contace 74. their internal world 75. B Part V Translation (20 marks) Section A (10 marks) 76. 奥巴马政府在新预算中承认了这个问题,该预算包括了一个五千万美元的预防性 方案 气瓶 现场处置方案 .pdf气瓶 现场处置方案 .doc见习基地管理方案.doc关于群访事件的化解方案建筑工地扬尘治理专项方案下载 .。 77. 他们的研究参照了12个最大的州的调查数据,发现16-24岁的人中有16%的人辍学。 78.这个问题在男性中尤为明显,辍学的人占到了全国转学人数的60%还多。79.全国各城市 内一些大的高中被恰当地称为 “辍学工厂”,因为辍学学生多于毕业学生。 80.辍学危机明显给国民经济造成威胁,但在缺少联邦政府帮助和指导情况下,各州各地现 仍在努力遏制这种危机。 Section B (10 marks) 81. The rapid increase in the price of oil has affected the world economy. 82. The mose important thing in the Olympic Games is not to win but to take part. 83. If I had known you didn’t have a key, I wouldn’t have locked the door. 84. Is it worthwhile to spend so much time and energy arranging this activity? 85. These astamps appeal to many stamp collectors because of their unique artistic designs. Part VI IQ Test (5 marks) 86. 8.6 (there are two alternate sequences, +1.65 and +1.92) 87. in 1983 88. 13 seats (37 people each paid,51) 89. 28 90. Nine regions 2011年全国大学生英语竞赛初赛试题 Part I Listening Comprehension (30 marks) Section A (5 marks) In this section, you will hear five short conversations. Each conversations will be read only once. At the end of each conversation, there will be a twenty-second pause, read the question and the there choices marked A,B and C ,and decide which is the best answer.Then mark the corresponding letter on the answer sheet with a single line through the centre. 1. What does the man want to do? A. Get something to eat now. B. Find a quiet place that shows games. C. Watch the next game with the woman. 2. Why does not the man have a MySpace account? A. He is not skilled at using computer. B. All of the instruction are in English C. The woman won not teach him. 3. How long does the woman plan to try teleworking A. For a few days. B. For a few weeks. C. For a few months. 4. What does the man hope will happen? A. The price of cell phone novels will go down. B. The novel’s author will writer longer stories. C. The woman will tell him ho the story ends. 5. what is the woman going to do next? A. turn on her computer. B. Go for a walk with peter. C. Visit her new neighbors. Section B (10 marks) In this section, you will hear two long conversations. Each conversation will be read only once. At the end each conversation, there will be a one minute pause. During the pause, read the questions, each with three choices marked A, B and C, and decide which is the best answer. Then mark the corresponding letter on the answer sheet with a single line through the centre. Conversation one 6. What did Jack do over the summer? A. He studied very hard. B. He took a summer class. C. He visited one of his teachers. 7. What does Jack think of Ms Wellington as a teacher? A. Easy-going. B. Tough. C. Interesting. 8. Why is Ms Wellington’s class hard? A. Her exams are difficult. B. She does not give students the help they need. C. She makes do lots of work. Conversation two 9. Why is Mrs. Griffin going to the city where the hotel is located? A. He is on holiday. B. He’s on a business trip C. He is going to a conference. 10. How many times has Mrs. Griffin stayed at the Sunrise Hotel? A. Twice. B. Once. C. Three times. 11. Where is Mrs. Griffin form? A. Canada. B. New Zealand. C. Australia. 12. What is Mrs. Griffin’s passport number? A.87647489 B.87637289 C.87637489 13. What kind of room does Mrs. Griffin want? A. A single room for two nights. B. A double room for two nights. C. A single room for one night. 14. When will Mrs. Griffin arrive at Sunrise Hotel? A. at 9:15 pm B. at 9:35 pm C. at 10:00 pm 15. What food will be put into Mrs. Griffin’s room? A. a sandwich with fries. B. a cheese sandwich. C. a burger with chips. Section C (5Marks) 16. What does the Associated Press ask editor and news directors to do? A. vote for the top stories of the year. B. describe the oil spill in the Gulf of Mexico C. writes about the 11 workers killed in the explosion 17. Where are the doctors and technology experts from? A. New York. B. London C. Tokyo 18. For how long does President Obama agree to extend the tax cuts? A. for four years B. for three years C. for two years. 19. How many people in the world don’t have enough to eat,according to the report? A. more than one billion. B. some six hundred million. C. nearly nine hundred million 20. What have astronomers recently discovered? A. there are unknown plants in older galaxies. B. there are many galaxies in the universe C. there are a lot more red dwarf in older galaxies Section D (10 marks) In the section, you will hear a short passage. There are 10 missing words phrases. Fill in the blanks with the exact words or phrases you hear. Remember to write the answer on the answer sheet What do you do if you don’t get your first choice university? This ____ faces thousands of British every year. Many such_____ turn to Clearing, a service that helps find university places for students at the last moment. If they don’t have the marks to get into their____, Clearing tells them about places available at other university, though they might have to read a difficult subject. This year has seen a record number of people applying to university. This, combined with the _____________________,an uncertain job market, and budget cuts at university, product even more of a scramble for places than usual. Some sources say six students have applied for each remaining___________________________ place The British University Admissions Service, UCAS, says up to a quarter of this year’s university applicants-almost 190000 people-have not been admitted into a____________________________. That is an increase of over 46000 students from last year. Faced with these figure, some British students might consider an interesting alternative:_____________________________. The University of Nottingham for is offering place at its campuses in Ningbo, near Shanghai, and Kuala Lumpur, Malaysia. Students at these institutions can earn University of Nottingham degrees, according, engineering and English. Similarly, the University of Bolton says it has unlimited places at its campus in the United Arab Emirates. To deal with these problems, the UK’s Higher Education Minister, David Willet’s, is encouraging students who have not made the grade to consider alternatives to university, such as _______________________and studying at home. “There are arrange of options available, “he says. “people can reapply next year, so they should consider spending this year in a way that will add positively to their CVs. Getting _____________________or other skills will strengthen their chances next year.” Some commentators say, though that rising university costs, poor long-term_______________________, and a drop in graduate recruitment mean this the worst time to be a university student in the UK. Part Two Vocabulary and Structure (15 marks ) There are 15 incomplete sentences in this section. For each blank there are four choices marked A, B, C and D. Choose the one that best completes the sentence. Then mark the corresponding letter on the answer sheet with a single line through the centre. 31. After four days of talks, we are glad to announce that the union and management have reached an______. The agreement is fair and benefits both sides. A. accord B. accomplishment C. identity D. undertaking 32. As the clerk______prepared my milk shake, I wondered how long she had been working there ,mindlessly making ice cream treats in a set order of steps. A. logically B. methodically C. graphically D. synthetically 33. As a boy he wanted to be a fireman. As a high school student, he thought he'd like to become a teacher. Now he______to be nothing more than a janitor. A. Assumes B. Prescribes C. Aspires D. Presumes 34. Regardless of what caused it, I an grateful that have finally reached a point in my life_______I can appreciate my strengths, accept my weaknesses and try to be comfortable with everything in between. A. Why B. Where C. Which D. What 35. ______information provided by members of the public, the police would have a much move difficult job. A. Supposing B. Provided the C. If it were not for D. On condition that 36. Peter Brown was a painstaking writer;______, he once spent half a day on the composition of a single sentence. A. On the other hand B. Nevertheless C. Moreover D. For example 37.----What an I going to do about a present for Carol? ----You______some flowers. A. Might have sent her B. Must have sent her C. Could send her D. Would send her 38. Without the air holding in some of the sun's heat, the earth______cold at night, too cold for us to live on. A. Will be freezing B. Would be freezing C. An be frozen D. Would be frozen 39. The students in our university each______an English dictionary. That is to say, each of the students in our university______an English dictionary. A. Have; has B. Have; have C. Has; have D. Has; has 40. Here's your kitchen. I hope you enjoy cooking here. Is there______else that you need? A. Something B. Anything C. Nothing D. Everything 41. David______his business partner over plans to reduce the workforce. A. Came down to B. Broke down to C. Fell out with D. Went along with 42. ______is this piece of equipment to be removed from the building. A. On no account B. Absolutely C. Scarcely D. Not at all 43. Helen' s parents were______that she was still on the job., but she had resigned. A. In doubt B. Of the opinion C. Under the impression D. With suspicion 44.----I don't think I will ever, in my life, win a lottery of five million dollars. ----Well, ______. Anything can happen. A. You made it B. You're kidding C. What you say D. You can never tell 45.-----How did you find the concert in the Grand Theatre last night? -----______ but the conductor was perfect. A. I couldn't agree more B. I didn't think much of it C. I was crazy about it D. I really liked it Part Three Cloze(15 marks ) I have been reading a lot on my iPad recently, and I have some (46)_____ (complain) not about the iPad itself but about the state of digital reading generally. Reading is a subtle thing, and its subtleties are artifacts of a venerable medium: words printed in ink on paper. Glass and pixels aren't the same. When I read a physical book, I don't have to look anywhere else to find out how much I've read. The iPad e---reader, iBooks tries to create the (47) illu_____ of a physical book. The pages seem to turn, and I can the edges of those that remain, but it's fake. There are always exactly six unturned pages, no matter (48)_____ I am in the book. Also, there is a larger problem. Books in their digital format look vastly less "finished", or less genuine than real books. You can vary their font and type size, but this only makes them (49)_____(resemble) word---processed---no matter how (50)_____(wretch) or wonderful they are---will never look as good as Robert Hass's poems in the print edition of The Apple Trees at Olema. But your poems can look almost exactly as ugly---as "e---book---like" ---as the Kindle version of that collection. All the e---book I've read have been ugly---books by Chang---rae Lee, Alvin Kernan, and Stieg Larsson---though the texts have been wonderful. I didn't grow up reading texts. I grew up reading books, and this(51)_____(differ) is important. When it comes to digital editions, the(52)_____(assume) seems to be that all books(53)are_____(create) equal. However, nothing could be further from the truth. In the mass migration from print to digital, we're seeing a profusion of digital books---many of them out of copyright---that look new and even "HD," but which may well have been supplanted by more accurate editions and better translations. We need a digital readers' guide---a place where readers can find(54)_____ whether the book they're about to download is the best available edition. (55)Fi_____, two related problems. I already have a personal library, but most of the books I've read have come from(56)_____(lend) libraries. Barnes & Noble has released an e---reader that allows short---term (57) _____(borrow) of some books. The entire idea behind Amazon's Kindle and Apple's iBooks assumes that you cannot read a book unless you own it first and that only you can read it unless you want to give your reading device to someone else. This goes against the social value of reading, the collective knowledge and (58)_____(collaborate) discourse that comes from access to (59)_____or our culture in general. Part Four Reading Comprehension (40 marks). Section A (10 marks ). Questions 61 to 65 are based on the following passage. Not keen on reading? Do you have trouble finding a novel that arouses your interest? Why not follow Ammon Shea's example and start reading a dictionary? Mr Shea owns over 1,000 dictionaries and he reads them for fun. He recently spent a year reading all 20 volumes of the Oxford English Dictionary. The dictionary contains more than 20,000 pages and over 59 million words. As he read from A to Z, he noted down interesting words in a ledger. This includes words such as "happify," meaning to make someone happy and "tripudiate", which means to dance, skip or leap for joy. Mr Shea also kept a diary about this experience, which has since become a best---selling book. Why did he do this? He claims it was fun. "I've always enjoyed reading dictionaries . They are far more interesting than people give then credit for," he said. It appears that it was not his goal to sound more intelligent by using longer and more complex words. "I'm not against long, fancy or obscure words, but I'm opposed to using then for their own sake," he said. In fact ,as a result of reading so many new words , Mr Shea often forgot everyday vocabulary. He wrote, "My head was so full of words that I often had trouble forming simple sentences." Mr Shea is not alone in his love of reading dictionaries.Elaine Higgleton, a representative of Collins Cbuild dictionaries, explained that thousands of crossword puzzle and Srabble fans read dictionaries for fun and to improve their games. Ms Higgleton did however note that, "It's probably not the best way to learn English ,and you'd learn more than you need." It is not known how many of the 59 million words Ms Shea remembers, but he has certainly made history with his eccentric hobby. Questions 61 to 65. Decide whether the following statements are True or False. 61. Mr Shea has read 1.000 dictionaries. 62. Mr Shea spent one month reading the Oxford English Dictionary. 63. In Mr Shea's opinion,people don't give dictionaries enough credit for being interesting. 64. Mr Shea thinks it is important t be able to use long and complicated words in everyday conversation. 65. Elaine Higgleton thinks that reading a dictionary is the best way to learn English. Section B (10 marks) Questions 66 to 70are based on the following passage. Surfing is something people often get hooked on after trying it a few times. For many surfers it is much more than a hobby---they would probably agree with the American professional surfer Kelly Slater when he said,"Once you're in, you're in. There's no getting out." "Surfing", of course, refers to riding on ocean waves using a surfboard. Many surfers stand up on their boards, which requires god balance and is therefore difficult for most beginners to learn, but some lie down and "bodyboard" The history of surfing probably began with the Polynesian people of the Pacific Islands. One of the first white people to see anyone surfing was the British explorer Captain Gook, when his ship arrived in Hawaii in 1779. He watched many Hawaiians riding waves on large pieces of wood, and reported that, "Surfing seems to give them a feeling of great pleasure. " When surfing started to become very popular in the United States in the 1950's and 60s, surfers used large wooden boards (often more than three metres long) that were quite heavy. Boards today are shorter and also much lighter, because they are made of artificial materials instead of wood. For anyone who wants to try surfing. The only essentials are waves and a board. There are a few other things, however, that most surfers find important; a cord t attach one of their ankles to the board and therefore stop it from being carried a long way away when they fall off'; wax, which they put on the surface of the board to help their feet stick to it; and a wetsuit to help them keep warm in cold water. The south---west of English is an example of a place where surfers usually need wetsuits, even in summer. Surfing has been a professional sport for many yeara and the very best surfers are able to make a living from it. Most of the best professional surfers in the last 30 years, both men and women, have been American or Australian, but surfers from Brazil, Peru and South Africa have also won important competitions. Questions 66 to 70 Answer the following questions with the information given in the assage in a maximum of 10 words for each question. 66. Why do most beginners find it difficult t stand up on a surfboard? 67. In what part of the world did surfing probably begin? 68. When did surfing start to become very popular in the United States? 69. What do surfers use wax for? 70. According to the passage, in what part of the world do surfers usually need wetsuits? Section C (10 marks) Questions 71 to 75 are based on the following passage. The latest human development report from the United Nations Development Programmed (UNDP) contains some good news, but also a very serious warning about the threat posed y climate change. The report, published annually since 1990, seeks to asses “human development” around the world, and calculates a “Human Development Index (HDI) for 169 counties. The HDI is based on average income, life expectancy and level of education in a country. Not surprisingly, rich counties tend to have higher HDIs than poor counties, but there are interesting variations in human development among countries with similar levels of economic development, because some have better health and education systems than others. According to the 2010 report, the county with the highest level of human development is Norway, followed by Australia, New Zealand, the United States and Ireland. Most of the lowest HDIs belong to counties in sub-Saharan Africa. Almost all counties around the worlds have higher HDIs now than in 1990, despite the fact that since the 2008 financial crisis, the total number of people living in extreme poverty has increased. The report concludes that most people are healthier, live longer, are better educated and have access to more goods and services. Even in countries with severe economic problems, people’s level of health and education as generally improved. Although sub-Saharan African countries are at the bottom of the pile in terms of human development, some of them have made significant progress since 1990. The report is critical, however, of the fact economic inequality has increased significantly in the last twenty years, both within and between countries. The greatest threat to improving HDIs in the future, according to the report, is climate change. Economic growth increases average incomes in a country through increasing production and consumption. However , if this leads to greater emissions of greenhouse gases, as has always been the case in the past, global warning will probably accelerate, and cause severe environmental problems in some parts of the world hat will threaten the livelihoods of huge numbers f people. The progress of the last twenty years, therefore, might not be sustainable. The only solution, according to the report, I to break the link between economic growth and greenhouse gas emissions-which, needless to say, is easier, said than done. Questions 71 to 75 Complete the following sentences with information given in the passage in a maximum of 10 words for each blank. 71. The concept of “human development” is based on the following three factors: _______, _____________and_____________. 72. Some countries with similar levels of economic development have quite different HDIs because they have_____________. 73._________________has caused the number of people living in extreme poverty to increase since 2008. 74. The report says that ______________ is the greatest threat to increasing HDIs in the future. 75. The report says the link between____________ and ____________needs to be broken. Section D (10 marks) Questions 76 to 80 are based on the following passage It is natural for young people to be critical of their parents at times and to blame them for most of the misunderstanding between them. They have always complained, more or less justly, that their parents are out of touch with modern ways; that they are possessive and dominant; that they do not trust their children to deal with crises: that they talk too much about certain problems-and that they have no sense o humor, at least parent-child relationships. I think it is true that parents often underestimate their teenage children and also forget how they felt themselves when were young. Young people often irritate their parents with their choices in clothes, hairstyles, entertainers and music. This is not their motive. They feel cut off from the adult world into which they have not yet been accepted, so they create a culture and society and their own. Then, if it turns out that their music, entertainers, vocabulary, clothes or hairstyles irritate their parents, this gives them additional enjoyment. They feel they are superior, at least in a small way, and that they are leaders in style and taste. Sometimes teenagers are resistant and proud because they do not want their parents to approve of what they do. If they did approve, it looks as if the teenager is betraying his own age group. All this is assuming that the teenager is the underdog: he can not win but at least he can keep his honor. This is a passive way of looking at things. It is natural enough after years of childhood, when children were completely under their parent’s control, but it ignores the fact that when they become teenagers, children are beginning to be responsible for themselves. If you plan to control your life, co-operation should be a part of that plan. You can charm other people, especially your parents, into doing things the way you want. You can also impress people with your of responsibility and your initiative, so that they will give you the authority to do what you want to do. Questions 76 to 78 Choose the best answer according to the passage. 76 the first paragraph is mainly about_____________. A teenagers’ criticism of their parents B misunderstanding between teenagers and their parents C the dominance of parents over their children D teenagers’ ability to deal with crises 77 teenagers have strange clothes and hairstyles because they___________ A have a strong desire to be leaders in style and taste B want to prove their existence by creating a culture of their own C have no other way to enjoy themselves D want to irritate their parent 78 teenagers do not want their parents to approve of what they do because they _______________. A have already been accepted into adult world B feel that they are superior to adult world C want to win adults over to their culture D don’t want to appear to be disloyal to their own age group Question 79 to 80 Translate the sentences in the passage into Chinese 79 I think it is true that parents often underestimate their teenage children and also forget how they felt themselves when were young 80 You can charm other people, especially your parents, into doing things the way you want. Part V translate (10 mark) Translate the following sentences into English, using the words given brackets. Remember to write your answer on the answer sheet. 81 他以牺牲健康为代价获得财富。(at the cost of ) 82 我不知道有多少学生涉及到这次事件。(be involved in ) 83 为孤儿捐款,请按以下地址或电话同我们联系。(contact) 84 目前,在韩国和日本, 汉语已成为第二外语,仅次于英语。(second to) 85 要不是下雨,我们的旅行本来是很愉快的。(but for) Part VII writing (30 marks) Task I (10marks) It’s the beginning of a new year. Write describing resolutions you make to try to change your life and why you Li Ming instead Write a note of about 100 words in an appropriate style on the answer sheet. Task II (20marks) Write about the following topic. Success is something most people try to achieve. What does success mean to different people, and what is your view or success? Give reasons for your answer and include any relevant example from your own knowledge or experience. Write your article in about 140 words in an appropriate style on the answer sheet.
本文档为【[英语考试]03-11年08由于格式原因除外全国大学生英语竞赛c类初赛试卷及答案完全整理个人收录_保证全!】,请使用软件OFFICE或WPS软件打开。作品中的文字与图均可以修改和编辑, 图片更改请在作品中右键图片并更换,文字修改请直接点击文字进行修改,也可以新增和删除文档中的内容。
该文档来自用户分享,如有侵权行为请发邮件ishare@vip.sina.com联系网站客服,我们会及时删除。
[版权声明] 本站所有资料为用户分享产生,若发现您的权利被侵害,请联系客服邮件isharekefu@iask.cn,我们尽快处理。
本作品所展示的图片、画像、字体、音乐的版权可能需版权方额外授权,请谨慎使用。
网站提供的党政主题相关内容(国旗、国徽、党徽..)目的在于配合国家政策宣传,仅限个人学习分享使用,禁止用于任何广告和商用目的。
下载需要: 免费 已有0 人下载
最新资料
资料动态
专题动态
is_594905
暂无简介~
格式:doc
大小:1MB
软件:Word
页数:0
分类:生活休闲
上传时间:2017-10-14
浏览量:41